final version 1989 to 2010 vn tst

177

Upload: dang-nhan

Post on 27-Jun-2015

347 views

Category:

Documents


26 download

TRANSCRIPT

Page 1: Final Version 1989 to 2010 VN TST
Windows XP
Typewriter
Viet Nam Team Selection Test Collection From 1989 to 2010
Page 2: Final Version 1989 to 2010 VN TST

Mục lục

1 Đề thi chọn đội tuyển toán 3

1.1 Đề thi chọn đội tuyển toán năm học 1989 - 1990(Ngày thi: 16, 17/5/1990) . . . . . . . . . . . . . . . . . . . 3

1.2 Đề thi chọn đội tuyển toán năm học 1990 - 1991(Ngày thi 8, 9/5/1991) . . . . . . . . . . . . . . . . . . . . . 4

1.3 Đề thi chọn đội tuyển năm học 1991 - 1992(Ngày thi 19, 20/05/1992) . . . . . . . . . . . . . . . . . . . 6

1.4 Đề thi chọn đội tuyển toán năm học 1992 - 1993(Ngày 4, 5/05/1993) . . . . . . . . . . . . . . . . . . . . . . 7

1.5 Đề thi chọn đội tuyển toán năm học 1993 - 1994(Ngày 18, 19/05/1994) . . . . . . . . . . . . . . . . . . . . . 8

1.6 Đề thi chọn đội tuyển toán năm học 1994 - 1995(Ngày 5, 6/5/1995) . . . . . . . . . . . . . . . . . . . . . . . 9

1.7 Đề thi chọn đội tuyển toán năm học 1995 - 1996(Ngày 17, 18/5/1996) . . . . . . . . . . . . . . . . . . . . . . 11

1.8 Đề thi chọn đội tuyển toán năm học 1996 - 1997(Ngày 16, 17/5/1997) . . . . . . . . . . . . . . . . . . . . . . 12

1.9 Đề thi chọn đội tuyển toán năm học 1997 - 1998(Ngày 13, 14/5/1998) . . . . . . . . . . . . . . . . . . . . . 13

1.10 Đề thi chọn đội tuyển năm học 2001 - 2002(Ngày thi 7, 8/5/2002) . . . . . . . . . . . . . . . . . . . . . 14

1.11 Đề thi chọn đội tuyển toán năm học 2003 - 2004(Ngày 7, 8/5/2004) . . . . . . . . . . . . . . . . . . . . . . . 15

2 Đáp án tuyển sinh 182.1 Đáp án chọn đội tuyển năm học 1991 - 1992 . . . . . . . . . 18

2.2 Đáp án chọn đội tuyển năm học 1992 - 1993 . . . . . . . . . 242.3 Đáp án chọn đội tuyển năm học 1993 - 1994 . . . . . . . . . 342.4 Đáp án chọn đội tuyển năm học 1994 - 1995 . . . . . . . . . 45

2.5 Đáp án chọn đội tuyển năm học 1995 - 1996 . . . . . . . . . 512.6 Đáp án chọn đội tuyển năm học 1996 - 1997 . . . . . . . . . 59

1

Windows XP
Typewriter
Chaper I
Windows XP
Typewriter
Chapter I
Windows XP
Typewriter
Page 3: Final Version 1989 to 2010 VN TST

2 MỤC LỤC

2.7 Đáp án chọn đội tuyển năm học 1997 - 1998 . . . . . . . . . 662.8 Đáp án chọn đội tuyển năm học 2001 - 2002 . . . . . . . . . 762.9 Đáp án chọn đội tuyển năm học 2003 - 2004 . . . . . . . . . 81

Page 4: Final Version 1989 to 2010 VN TST

Chương 1

Đề thi chọn đội tuyển toán

1.1 Đề thi chọn đội tuyển toán năm học 1989

- 1990

(Ngày thi: 16, 17/5/1990)

Bài 1: Trong mặt phẳng cho đa giác lồi M0,M1, . . . ,M2n (n > 1) mà 2n+1đỉnh M0,M1, . . . ,M2n nằm (theo thứ tự ngược chiều quay của kim đồnghồ) trên một đường tròn (C) bán kính R. Giả sử có điểm A bên trong đa

giác lồi đó sao cho các góc M0AM1, M1AM2, . . . , M2n−1AM2n, M2nAM0 đềubằng nhau, (và bằng 360

2n+1độ). Giả sử A không trùng với tâm của (C) và

gọi B là điểm nằm trên đường tròn (O) sao cho đường thẳng AB vuông gócvới đường kính đi qua A.

Chứng minh:

2n + 11

AM0+ 1

AM1+ · · · + 1

AM2n

< AB <AM0 + AM1 + · · · + AM2n

2n + 1< R

Bài 2: Cho bốn số thực dương a, b,A,B. Xét dãy số thực x1, x2, x3, x4, . . .xác định bởi:

x1 = a, x2 = b

xn+1 = A 3√

x2n + B 3

√x2

n−1 (n = 2, 3, 4, . . .)

Chứng minh rằng tồn tại giới hạn limn→∞ xn và hãy tính giới hạn ấy.Bài 3: Chứng minh rằng không tồn tại hàm số f(x) xác định với mọi

số thực x và thoả mãn f(f(x)) = x2 − 2 với mọi x.Bài 4: Xét tập hợp T gồm hữu hạn số nguyên dương thoả mãn hai điều

kiện:

3

Page 5: Final Version 1989 to 2010 VN TST

4 Chương 1. Đề thi chọn đội tuyển toán

1. Với hai phần tử bất kỳ của T thì ước số chung lớn nhất và bội sốchung nhỏ nhất của chúng cũng là những phần tử của T .

2. Với mỗi phần tử x của T , có phần tử x′ của T sao cho x và x′ nguyêntố cùng nhau và bội số chung nhỏ nhất của chúng là số lớn nhất củaT .

Với mỗi tập hợp T như thế, ký hiệu l(T ) là số phần tử của nó. Tìm sốl(T ) lớn nhất, biết rằng l(T ) nhỏ hơn 1990.

Bài 5: Cho tứ diện mà mỗi cặp cạnh đối diện đều có tích độ dài bằng l.Gọi các góc giữa các cạnh đối diện đó là α, β, γ và gọi các bán kính của cácđường tròn ngoại tiếp các mặt của tứ diện là R1, R2, R3, R4. Chứng minh:

sin2 α + sin2 β + sin2 γ >l√

R1R2R3R4

Bài 6: Có n em học sinh (n > 3) đứng thành một vòng tròn và luônquay mặt vào cô giáo ở tâm vòng tròn. Mỗi lần cô giáo thổi còi thì có haiem nào đó đứng sát cạnh nhau đổi chỗ cho nhau, còn các em khác khôngdời chỗ. Tìm số M bé nhất để sau M lần thổi còi, bằng các đổi chỗ nhưnói ở trên một cách thích hợp, các học sinh đứng được thành vòng tròn saocho: Hai em bất kỳ lúc ban đầu đứng sát cạnh nhau thì lúc kết thúc cũngđứng sát cạnh nhau, nhưng trong hai em đó, tạm gọi là A và B, nếu A lúcban đầu đứng bên tay trái của B thì lúc kết thúc A đứng bên tay phải củaB.

1.2 Đề thi chọn đội tuyển toán năm học 1990

- 1991

(Ngày thi 8, 9/5/1991)

Bài 1: Trong mặt phẳng xét tập hợp S gồm n điểm phân biệt (n > 3) thoảmãn ba điều kiện sau:

1. Khoảng cách giữa hai điểm bất kỳ thuộc S đều không vượt quá 1 đơnvị dài.

2. Mỗi điểm A thuộc S có đúng hai điểm "kề với nó", nghĩa là hai điểmthuộc S có cùng khoảng cách bằng 1 đến điểm A.

3. Với hai điểm tuỳ ý A,B thuộc S gọi A′ và A′′ là hai điểm kề với A,

gọi B′ và B′′ là hai điểm kề với B thì A′AA′′ = B′BB′′.

Page 6: Final Version 1989 to 2010 VN TST

1.2. Đề thi chọn đội tuyển toán năm học 1990 - 1991 (Ngày thi 8, 9/5/1991) 5

Hỏi có tồn tại tập hợp S như thế khi n = 1991 không và khi n = 2000không? Vì sao?

Bài 2: Cho dãy số thực dương a1, a2, . . . , an với n lớn hơn 2 và a1 khácan, là dãy không giảm (nghĩa là ak 6 ak+1 với k = 1, 2, . . . , n − 1) hoặc làdãy không tăng (nghĩa là ak > ak+1 với k = 1, 2, . . . , n − 1), và cho các sốthực dương x, y thoả mãn x

y> a1−a2

a1−an. Chứng minh rằng:

a1

a2x + a3y+ · · · + ak

ak+1x + ak+2y+ · · ·+

· · · + an−2

an−1x + any+

an−1

anx + a1y+

an

a1x + a2y> n

x + y

Bài 3: Cho dãy số thực dương x1, x2, . . . , xn, . . . xác định bởi:

x1 = 1, x2 = 9, x3 = 9, x4 = 1

xn+4 = 4√

xnxn+1xn+2xn+3 với n > 1

Chứng minh rằng dãy số trên có giới hạn. Tìm giới hạn đó.Bài 4: Gọi T là hình tứ diện tuỳ ý thoả mãn hai điều kiện sau:

1. Mỗi cạnh có độ dài không vượt quá 1 đơn vị dài.

2. Mỗi mặt là một tam giác vuông.

Ký hiệu s(T ) là tổng bình phương diện tích bốn mặt của hình tứ diệnT . Tìm giá trị lớn nhất của s(T ).

Bài 5: Với mỗi số tự nhiên n, định nghĩa số f(n) như sau: f(1) = 1và khi n > 1 thì f(n) = 1 + a1p1 + · · · + akpk, trong đó n = p1 . . . pk là sựphân tích thành thừa số nguyên tố của n (các số nguyên tố p1, . . . , pk đôimột khác nhau và a1, . . . , ak là số nguyên dương). Với mỗi số tự nhiên s,đặt fs(n) = f(f(. . . (f(n)) . . .)), trong đó ở vế phải có đúng s lần chữ f .

Chứng minh rằng với số tự nhiên a cho trước, có số tự nhiên s0 để vớimọi số nguyên s > s0 thì tổng fs(a) + fs−1(a) không phụ thuộc vào s.

Bài 6: Cho tập hợp X gồm 2n số thực đôi một khác nhau (n > 3). Xétmột tập hợp K gồm một số cặp số thực (x, y) với x, y thuộc X, x khác y,mà K thoả mãn hai điều kiện sau:

1. Nếu cặp số (x, y) thuộc K thì cặp số (y, x) không thuộc K.

2. Mỗi số x thuộc X có mặt nhiều nhất trong 19 cặp số của K.

Chứng minh rằng ta có thể phân chia tập hợp X thành 5 tập hợp conkhông rỗng và đôi một không giao nhau x1, x2, x3, x4, x5 sao cho với mỗii = 1, 2, 3, 4, 5 thì số cặp số (x, y) thuộc K mà x và y cùng thuộc Xi khôngvượt quá 3n.

Page 7: Final Version 1989 to 2010 VN TST

6 Chương 1. Đề thi chọn đội tuyển toán

1.3 Đề thi chọn đội tuyển năm học 1991 -

1992

(Ngày thi 19, 20/05/1992)

Bài 1: Cho hai số tự nhiên n và m (n > 1). Hãy tìm số nguyên dương knhỏ nhất có tính chất sau: Trong k số nguyên tuỳ ý a1, a2, . . . , sk mà ai −aj

(i 6= j và i, j chạy từ 1 đến k) không chia hết cho n, luôn tồn tại hai sốap, as (p 6= s) thoả mãn m + ap − as chia hết cho n.

Bài 2: Cho đa thức f(x) với hệ số thực và có bậc lớn hơn hoặc bằng 1.Chứng minh rằng với mỗi số c > 0, tồn tại số nguyên dương n0 thoả mãnđiều kiện sau: Nếu đa thức P (x) với hệ số thực có bậc lớn hơn hoặc bằngn0, và có hệ số của số hạng bậc cao nhất bằng 1 thì các số nguyên x mà|f(P (x))| 6 c không vượt quá bậc của P (x).

Bài 3: Cho tam giác ABC có BC = a,CA = b,AB = c (a 6= b 6= c).Trong mặt phẳng ABC lấy các điểm A′, B′, C ′ sao cho:

1. Các cặp điểm A và A′, B và B′, C và C ′ hoặc đều ở cùng phía hoặcđều ở khác phía theo thứ tự đối với các đường thẳng BC,CA,AB.

2. Các tam giác A′BC,B ′CA,C ′AB là các tam giác cân đồng dạng.

Hãy xác định các góc A′BC theo a, b, c để các độ dài AA′, BB′, CC ′

không phải là ba độ dài của ba cạnh một tam giác.(Tam giác được hiểu theo nghĩa thông thường: ba đỉnh của nó không

thẳng hàng).Bài 4: Trong mặt phẳng cho một họ hữu hạn hình tròn thoả mãn: hai

hình tròn bất kỳ hoặc ở ngoài nhau hoặc tiếp xúc ngoài với nhau và mỗihình tròn không tiếp xúc với quá 6 hình tròn khác. Giả sử mỗi hình trònkhông tiếp xúc với 6 hình tròn khác đã được đặt ứng với một số thực nàođó. Chứng minh rằng không có quá một cách đặt ứng với mỗi hình tròn cònlại một số thực bằng trung bình cộng của 6 số ứng với 6 hình tròn tiếp xúcnó.

Bài 5: Tìm tất cả các cặp số nguyên dương (x, y) thoả mãn phươngtrình

x2 + y2 − 5xy + 5 = 0

.Bài 6: Trong một hội thảo khoa học tất cả các đại biểu tham dự biết

tổng cộng 2n ngôn ngữ n > 2. Mỗi người biết đúng 2 ngôn ngữ và bất cứhai người nào cũng biết chung nhiều nhất một ngôn ngữ. Biết rằng với mộtsố nguyên k thoả mãn 1 6 k 6 n− 1 đều có không quá k − 1 ngôn ngữ màmỗi ngôn ngữ này có không quá k người biết. Chứng minh rằng ta có thể

Page 8: Final Version 1989 to 2010 VN TST

1.4. Đề thi chọn đội tuyển toán năm học 1992 - 1993 (Ngày 4, 5/05/1993) 7

chọn ra một nhóm 2n đại biểu biết tổng cộng 2n ngôn ngữ và mỗi ngôn ngữcó đúng 2 đại biểu trong nhóm biết.

1.4 Đề thi chọn đội tuyển toán năm học 1992

- 1993

(Ngày 4, 5/05/1993)

Bài 1: Gọi hình chữ nhật kích thước 2× 3 (hoặc 3× 2) bị cắt bỏ một hìnhvuông 1× 1 ở một góc là hình chữ nhật khuyết đơn (xem hình 1). Gọi hìnhchữ nhật kích thước 2× 3 (hoặc 3× 2) bị căt bỏ hai hình vuông 1× 1 ở haigóc đối diện là hình chữ nhật khuyết kép (xem hình 2). Người ta ghép mộtsố hình vuông 2 × 2, một số hình chữ nhật khuyết đơn và một số hình chữnhật khuyết kép với nhau sao cho không có hai hình nào chờm lên nhau,để tạo thành một hình chữ nhật kích thước 1993 × 2000. Gọi s là tổng sốcác hình vuông 2× 2 và hình chữ nhật khuyết kép cần dùng trong mỗi cáchghép hình nói trên. Tìm giá trị lớn nhất của s.

Bài 2: Cho dãy số {an} được xác định bởi:

a1 = 1 và an+1 = an +1

√an

với n = 1, 2, 3, . . .

Hãy tìm tất cả các số thực α sao cho dãy {un} xác định bởi un = aαn

nvới

n = 1, 2, 3, . . . có giới hạn hữu hạn khác 0 khi n → +∞.

Bài 3: Xét các số thực x1, x2, x3, x4 thoả mãn:

1

26 x2

1 + x22 + x2

3 + x24 6 1

Tìm giá trị lớn nhất và nhỏ nhất của biểu thức:

A = (x1 − 2x2 + x3)2 + (x2 − 2x3 + x4)

2 + (x2 − 2x1)2 + (x3 − 2x4)

2

Page 9: Final Version 1989 to 2010 VN TST

8 Chương 1. Đề thi chọn đội tuyển toán

Bài 4: Gọi H, I,O theo thứ tự là trực tâm, tâm đường tròn nội tiếp vàtâm đường tròn ngoại tiếp của một tam giác. Chứng minh rằng 2.IO > IH.Hỏi dấu bằng xảy ra khi nào?

Bài 5: Cho số nguyên k > 1. Với mỗi số nguyên n > 1, đặt

f(n) = k.n(1 − 1

p1

)(1 − 1

p2

) . . . (1 − 1

pr

)

trong đó p1, p2, . . . , pr là tất cả các ước số nguyên tố phân biệt của n. Tìmtất cả các giá trị k để dãy {xm} xác định bởi x0 = a và xm+1 = f(xm),m =0, 1, 2, 3, . . . là dãy bị chặn với mọi số nguyên a > 1.

Bài 6: Xét n điểm A1, A2, . . . , An (n > 2) trong không gian, trong đókhông có 4 điểm nào đồng phẳng. Mỗi cặp điểm Ai, Aj(i 6= j) được nối vớinhau bởi một đoạn thẳng.

Tìm giá trị lớn nhất của n sao cho có thể tô tất cả các đoạn thẳng đóbằng hai màu xanh, đỏ thoả mãn ba điều kiện sau:

1. Mỗi đoạn thẳng được tô bằng đúng một màu.

2. Với mỗi i = 1, 2, . . . , n số đoạn thẳng có một đầu mút là Ai mà đượctô màu xanh không vượt quá 4.

3. Với mỗi đoạn thẳng Ai, Aj được tô màu đỏ đều tìm thấy ít nhất mộtđiểm Ak (k khác i, j) mà các đoạn thẳng AkAi và AkAj đều được tômàu xanh.

1.5 Đề thi chọn đội tuyển toán năm học 1993

- 1994

(Ngày 18, 19/05/1994)

Bài 1: Given a parallelogram ABCD. Let E be a point on the side BC andF be a point on the side CD such that the triangles ABE and BCF havethe same are. The diagonal BD intersects AE at M and intersects AF atN . Prove that.

a) There exists a triangle, three sides of which are equal to BM,MN,ND.b) When E,F vary such that the length sides of MN decreases, the

radius of the circumcircle of the abovementioned triangle also decreases.Bài 2: Consider the equation

x2 + y2 + z2 + t2 −Nxyzt− N = 0

where N is a given positive integer.

Page 10: Final Version 1989 to 2010 VN TST

1.6. Đề thi chọn đội tuyển toán năm học 1994 - 1995 (Ngày 5, 6/5/1995) 9

a) Prove that for an infinite number of values of N , this equation haspositive integral solution (each such solution consists of four positive integersx, y, x, t).

b) Let N = 4k(8m+7) where k,m are non-negative integers. Prove thatthe considered equation has no positive integral solution.

Bài 3: Let be given a polynomial P (x) of degree 4, having 4 positiveroots. Prove that the equation

1 − 4x

x2P (x) + (1 − 1 − 4x

x2)P ′(x) − P ′′(x) = 0

has also 4 positive roots.Bài 4: Given an equilateral triangle ABC and a point M in the plan

(ABC). Let A′, B′, C ′ be respectively the symmetric through M of A,B,C.a) Prove that there exists s unique point P equidistant from A and B′,

from B and C ′ and from C and A′.b) Let D be the midpoint of the side AB. When M varies (M does not

coincide with D), prove that the circumcircle of triangle MNP (N is theintersection of the lines DM and AP ) passes through a fixed point.

Bài 5: Determine all function f : R → R satisfying

f(√

2x) + f((4 + 3√

2)x) = af((2 +√

2)x)

for all x.Bài 6: Calculate

T =1

n1!n2! . . . n1994!(n2 + 2n3 + 3n4 + · · · 1993n1994)!

where the sum is taken over all 1994-upple of natural numbers (n1, n2, . . . , n1994)satisfying

n1 + 2n2 + 3n3 + · · · + 1994n1994 = 1994

1.6 Đề thi chọn đội tuyển toán năm học 1994

- 1995

(Ngày 5, 6/5/1995)

Bài 1. Cho tam giác ABC với BC = a,CA = b,AB = c. Lấy sáu điểmphân biệt A1, A2, B1, B2, C1, C2 không trùng với A,B,C sao cho các điểmA1, A2 nằm trên đường thẳng BC; các điểm B1, B2 nằm trên đường thẳngCA; các điểm C1, C2 nằm trên đường thẳng AB. Gọi α, β, γ là các số thựcxác định bởi

−−−→A1A2 =

α

a

−−→BC,

−−−→B1B2 =

β

b

−→CA,

−−−→C1C2 =

γ

c

−→AB.

Page 11: Final Version 1989 to 2010 VN TST

10 Chương 1. Đề thi chọn đội tuyển toán

Xét các đường tròn ngoại tiếp các tam giác AB1C1,AB2C2, BC1A1, BC2A2,CA1B1, CA2B2 và gọi dA, dB, dC theo thứ tự là các trục đẳng phương củacặp đường tròn đi qua A, cặp đường tròn đi qua B, cặp đường tròn đi quaC. Chứng minh rằng dA, dB, dC đồng quy khi và chỉ khi

aα + bβ + cγ 6= 0.

Bài 2. Tìm tất cả các số nguyên k sao cho có vô số giá trị nguyên n ≥ 3để đa thức

Pn(x) = xn+1 + kxn − 870x2 + 1945x + 1995

có thể phân tích được thành tích của hai đa thức với hệ số nguyên có bậclớn hơn hay bằng 1.

Bài 3. Tìm tất cả các số nguyên a, b, n lớn hơn 1 thoả mãn điều kiện

(a3 + b3)n = 4(ab)1995.

Bài 4. Trong không gian cho n điểm (n ≥ 2) mà không có bốn điểm nàođồng phẳng và cho 1

2(n2 − 3n + 4) đoạn thẳng mà tất cả các đầu mút của

chúng nằm trong số n điẻm đã cho. Biết rằng có ít nhất một đoạn thẳngmà sau khi bỏ nó đi (giữ nguyên các đầu mút) thì sẽ tồn tại hai điểm phânbiệt mà không phải là hai đầu mút của một đường gấp khúc nào.

Hãy tìm số k lớn nhất sau cho có k đoạn thẳng tạo thành đường gấpkhúc khép kín mà mỗi đỉnh của nó là mút của đúng hai đoạn thẳng thuộcđường gấp khúc đó.

Bài 5. Với mỗi số nguyên không âm n đặt f(n) là số nguyên không âmlớn nhất sao cho 2f(n) là một ước số của n + 1. Cặp số nguyên không âm(n, p) được gọi là đẹp nếu 2f(n) > p. Hãy tìm tất cả các bộ ba số nguyênkhông âm (n, p, q) sao cho các cặp số (n, p), (p, q), và (n + p + q, n) đều làcác cặp số đẹp.

Bài 6. Cho hàm số thực

f(x) =2x3 − 3

3(x2 − 1).

1. Chứng minh rằng tồn tại hàm số g(x) liên tục trên R và có đồng thờicác tính chất sau

f(g(x)) = x, ∀x ∈ R; g(x) > x ∀x ∈ R.

2. Chứng minh rằng tồn tại số thực a > 1 để dãy {an}, n = 0, 1, 2, ...,được xác định bởi a0 = a, an+1 = f(an) ∀n ∈ N là dãy tuần hoàn vớichu kỳ dương nhỏ nhất bằng 1995.

Page 12: Final Version 1989 to 2010 VN TST

1.7. Đề thi chọn đội tuyển toán năm học 1995 - 1996 (Ngày 17, 18/5/1996) 11

1.7 Đề thi chọn đội tuyển toán năm học 1995

- 1996

(Ngày 17, 18/5/1996)

Bài 1. Trong mặt phẳng cho 3n điểm (n > 1) mà không có ba điểm nàothẳng hàng và khoảng cách giữa hai điểm bất kỳ không vượt quá 1. Chứngminh rằng có thể dựng được n tam giác đôi một rời nhau và thoả mãn đồngthời các điều kiện sau

1. Mỗi điểm trong 3n điểm đã cho là đỉnh của đúng một tam giác;

2. Tổng diện tích của n tam giác nhỏ hơn 12.

Hai tam giác được gọi là rời nhau nếu chúng không có điểm nàochung nằm bên trong cũng như nằm trên cạnh tam giác.

Bài 2. Với mỗi số nguyên dương n, gọi f(n) là số nguyên lớn nhất đểsố

[n−12

]∑

i=0

(2i + 1

n

)3i chia hết cho 2f(n).

Tìm tất cả các số nguyên dương n mà f(n) = 1996.Bài 3. Xét các số thực a, b, c. Tìm giá trị nhỏ nhất của biểu thức

f(a, b, c) = (a + b)4 + (b + c)4 + (c + a)4 − 47(a4 + b4 + c4).

Bài 4. Cho ba điểm A,B,C không thẳng hàng. Với mỗi điểm M củamặt phẳng (ABC) gọi M1 là điểm đối xứng của M qua đường thẳng AB,gọi M2 là điểm đối xứng của M1 qua đường thẳng BC và gọi M ′ là điểmđối xứng của M2 qua đường thẳng CA. Hãy xác định tất cả các điểm Mcủa mặt phẳng (ABC) mà khoảng cách MM ′ bé nhất. Gọi khoảng cáchđó là d. Chứng minh rằng với mỗi điểm M của mặt phẳng (ABC) khi tathực hiện liên tiếp ba phép đối xứng qua ba đường thẳng chứa ba cạnh củatam giác ABC theo thứ tự khác (so với thứ tự trên) để được điểm M ′′ thìkhoảng cách bé nhất của MM ′′ cũng bằng d.

Bài 5. Người ta muốn mời một số em học sinh tới dự một buổi gặp mặt,mà trong số đó mỗi em chưa quen với ít nhất là 56 em khác, và với mỗicặp hai em chưa quen nhau thì đều có ít nhất một em quen với cả hai emđó. Hỏi số học sinh được mời dự buổi gặp mặt nói trên có thể là 65 em haykhông?

Page 13: Final Version 1989 to 2010 VN TST

12 Chương 1. Đề thi chọn đội tuyển toán

Bài 6. Hãy tìm tất cả các số thực a sao cho dãy số {xn}, n = 0, 1, 2, ...,xác định bởi

x0 =√

1996, xn+1 =a

1 + x2n

với n = 0, 1, 2, , ...

có giới hạn hữu hạn khi n → ∞.

1.8 Đề thi chọn đội tuyển toán năm học 1996

- 1997

(Ngày 16, 17/5/1997)

Bài 1. Cho tứ diện ABCD với BC = a,CA = b,AB = c,DA = a1,DB =b1,DC = c1. Chứng minh rằng có điểm P duy nhất thoả mãn

PA2+a21+b2+c2 = PB2+b2

1+c2+a2 = PC2+c21+a2+b2 = PD2+a2

1+b21+c2

1

và với điểm P đó ta luôn có PA2 + PB2 + PC2 + PD2 ≥ 4R2, trong đó Rlà bán kính mặt cầu ngoại tiếp tứ diện ABCD. Tìm điều kiện cần và đủvới độ dài các cạnh của tứ diện để bất đẳng thức trên trở thành đẳng thức.

Bài 2. Ở một nước có 25 thành phố. Hãy xác định số k bé nhất sao chocó thể thiết lập các đường bay (dùng cho cả đi lẫn về) giữa các thành phốđể hai điều kiện sau được đồng thời thoả mãn

1. Từ mỗi thành phố có đường bay trực tiếp đến đúng k thành phố khác;

2. Nếu giữa hai thành phố không có đường bay trực tiếp thì tồn tại ítnhất một thành phố có đường bay trực tiếp đến hai thành phố đó.

Bài 3. Hãy tìm số thực α lớn nhất sao cho tồn tại vô hạn số tự nhiên(an), n = 1, 2, 3, ..., thoả mãn đồng thời các điều kiện sau

1. an > 1997n với mọi n ∈ N∗;

2. với mỗi n ≥ 2 đều có un ≥ aαn, trong đó un là ước số chung lớn nhất

của họ tất cả các số ai + ak mà i + k = n.

Bài 4. Cho hàm số f : N → Z thoả mãn các điều kiện f(0) = 2, f(1) =503 và f(n + 2) = 503f(n + 1) − 1996f(n) với mọi n ∈ N.

Với mỗi số k ∈ N∗ lấy số nguyên s1, s2, ..., sk sao cho si ≥ k với mọii = 1, 2, ..., k. Với mỗi số si (i = 1, 2, ..., k) lấy một ước nguyên tố p(si) nàođó của f(2si). Chứng minh rằng với số nguyên dương t ≤ k, ta có

k∑

i=1

p(si)...2t khi và chỉ khi k

...2t.

Page 14: Final Version 1989 to 2010 VN TST

1.9. Đề thi chọn đội tuyển toán năm học 1997 - 1998 (Ngày 13, 14/5/1998) 13

Bài 5. Hãy xác định tất cả các cặp số thực a, b sao cho với mọi n ∈ N∗

và với mọi nghiệm thực xn của phương trình

4n2x = log2(2n2x + 1)

ta luôn cóaxn + bxn ≥ 2 + 3xn.

Bài 6. Cho các số nguyên dương n, k, p với k ≥ 2 và k(p + 1) ≤ n. Chon điểm phân biệt cùng nằm trên một đường tròn. Tô tất cả n điểm đó bởihai màu xanh, đỏ (mỗi điểm tô bởi một màu) sao cho có đúng k điểm đượctô bởi màu xanh và trên mỗi cung tròn mà hai đầu mút là hai điểm màuxanh liên tiếp (tính theo chiều quay của kim đồng hồ) đều có ít nhất p điểmđược tô bởi màu đỏ.

Hỏi có tất cả bao nhiêu cách tô màu khác nhau?(Hai cách tô màu được gọi là khác nhau nếu có ít nhất một điểm được

tô bởi hai màu khác nhau trong hai cách đó).

1.9 Đề thi chọn đội tuyển toán năm học 1997

- 1998

(Ngày 13, 14/5/1998)

Bài 1. Cho hàm số f(x) xác định trên R sao cho với mọi số thực dương ctồn tại đa thức hệ số thực Pc(x) thoả mãn

|f(x) − Pc(x)| ≤ cx1998 với mọi x ∈ R.

Chứng minh rằng f(x) là một đa thức với hệ số thực.Bài 2. Trong mặt phẳng cho đường tròn (C) bán kính R chứa và tiếp

xúc với đường tròn (C ′) bán kính R2. Xét họ H các đường trong bên trong

(C), bên ngoài (C ′), tiếp xúc với (C) và (C ′). Với mỗi số nguyên n ≥ 3 vàcác số dương p1, pn, chứng minh rằng hệ thức

(p1 − pn)2 = (n − 1)2(2(p1 + pn) − (n − 1)2 − 8)

là điều kiện cần và đủ để có n đường tròn phân biệt (C1), (C2), ..., (Cn) củahọ H mà (Ci) tiếp xúc ngoài với (Ci−1) và (Ci+1) (i = 2, 3, ..., n − 1), ở đó(C1) có bán kính R

p1, (Cn) có bánh kính R

pn.

Bài 3. Cho các số nguyên dương m > 3. Giả sử p1, p2, ..., pk là tất cảcác số nguyên tố không vượt quá m. Chứng minh rằng

k∑

i=1

( 1

pi+

1

p2i

)> ln(lnm).

Page 15: Final Version 1989 to 2010 VN TST

14 Chương 1. Đề thi chọn đội tuyển toán

Bài 4. Tìm tất cả các đa thức P (x) hệ số nguyên với hệ số bậc cao nhấtbằng 1, có tính chât: Tồn tại vô số các số vô tỉ α để P (α) đều là số nguyêndương.

Bài 5. Giả sử d là ước dương của 5 + 19981998. Chứng minh rằng d cóthể biểu diễn dưới dạng d = 2x2 + 2xy + 3y2, ở đó x, y là các số nguyên khivà chỉ khi d chia cho 20 có dư 3 hoặc 7.

Bài 6. Trong một cuộc hội thảo có n, n ≥ 10 người tham dự. Biết rằng

1. Mỗi người quen với ít nhất[

n+23

]người tham dự.

2. Hai người bất kỳ A và B nếu không quen nhau thì quen nhau giántiếp, nghĩa là có k (k ≥ 1) người A1, A2, ..., Ak sao cho A quen A1, Ai

quen Ai+1, (i = 1, 2, ..., k − 1) và Ak quen B.

3. Không thể xếp n người thành một hàng ngang sao cho hai người cạnhnhau bất kỳ đều quen nhau.

Chứng minh rằng có thể chia n người thành hai nhóm: nhóm thứ nhất xếpđược quanh một bàn tròn sao cho hai người cạnh nhau bất kỳ đều quennhau, còn nhóm thứ hai gồm người đôi một không quen nhau.

1.10 Đề thi chọn đội tuyển năm học 2001 -

2002

(Ngày thi 7, 8/5/2002)

Bài 1. Tìm tất cả các tam giác ABC có BCA là góc nhọn và đường trung

trực của đoạn thẳng BC cắt các tia Ax và Ay, là các tia chia góc BAC

thành ba phần bằng nhau (BAx = xAy = yAC) tại các điểm M và N thoảmãn:

AB = NP = 2HM

trong đó: H là hình chiếu vuông góc của A trên BC và M là trung điểmcủa đoạn thẳng BC.

Bài 2. Người ta ghi lên bảng số nguyên dương N0. Hai người A và Bchơi trò chơi trò chơi sau: Người A xoá số N0 rồi ghi lên bảng số N1 ∈{N0 − 1; [N0/3]}. Tiếp theo người B xoá số N1 rồi ghi lên bảng số N2 ∈{N1 − 1; [N1/3]}. Đến lượt mình người A lại thực hiện phép toán trên đốivới N2;...Trò chơi cứ tiếp tục cho đến khi trên bảng xuất hiện số 0. Ngườighi số 0 đầu tiên được coi là thắng cuộc, người còn lại bị coi là thua cuộc.Hỏi ai, người A hay người B, là người có cách chơi để chắc chắn thắng nếu:

1) N0 = 120

Page 16: Final Version 1989 to 2010 VN TST

1.11. Đề thi chọn đội tuyển toán năm học 2003 - 2004 (Ngày 7, 8/5/2004) 15

2) N0 = (32002 − 1)/23) N0 = (32002 + 1)/2Bài 3. Cho số nguyên dương m có một ước nguyên tố lớn hơn

√2m+1.

Hãy tìm số nguyên dương M nhỏ nhất sao cho tồn tại một tập hợp gồmhữu hạn số nguyên dương đôi một khác nhau thoả mãn đồng thời các điềukiện sau:

i) m và M tương ứng là số nhỏ nhất và số lớn nhất trong T .ii) Tích tất cả các số thuộc T là một số chính phương.Bài 4. Cho số nguyên dương n ≥ 2 và cho bảng ô vuông kích thước

n × 2n (bảng gồm n hàng và 2n cột). Người ta đánh dấu một cách ngẫunhiên n2 ô vuông con của bảng. Chứng minh rằng với mỗi số nguyên k mà1 < k ≤ [n/2] + 1, luôn tồn tại k hàng sao cho bảng ô vuông kích thướck × 2n, được tạo nên từ k hàng đó, có không ít hơn

k!(n − 2k + 2)

(n − k + 1)(n − k + 2) . . . (n − 1)

cột chỉ gồm các ô được đánh dấu.([a] ký hiệu số nguyên lớn nhất không vượt quá a).Bài 5. Hãy tìm tất cả các đa thức P (x) với hệ số nguyên sao cho đa

thức

Q(x) = (x2 + 6x + 10)[P (x)]2 − 1

là bình phương của một đa thức với hệ số nguyên.Bài 6. Chứng minh rằng tồn tại số nguyên m ≥ 2002 và m số nguyên

dương đôi một khác nhau a1, a2, . . . , am sao cho số

m∏

i=1

a2i − 4

m∑

i=1

a2i

là số chính phương.

1.11 Đề thi chọn đội tuyển toán năm học

2003 - 2004

(Ngày 7, 8/5/2004)

Bài 1. Xét tập hợp S gồm 2004 số nguyên dương phân biệt a1, a2, . . . , a2004,có tính chất: Nếu với mỗi i = 1, 1, . . . , 2004, ký hiệu f(ai) là số các số thựcthuộc S nguyên tố cùng nhau với ai thì d(ai) < 2003 và f(ai) = f(aj) vớimọi i, j ∈ {1, 2, . . . , 2004}.

Page 17: Final Version 1989 to 2010 VN TST

16 Chương 1. Đề thi chọn đội tuyển toán

Hãy tìm số nguyên dương k nhỏ nhất sao cho trong mỗi k tập con củamột tập S tuỳ ý có tính chất nêu trên đều tồn tại hai số phân biệt mà ướcsố chung lớn nhất của chúng khác 1.

(k - tập con là tập con có k phần tử).Bài 2. Hãy xác định tất cả các số thực α mà ứng với mỗi α, có một và

chỉ một hàm số f xác định trên tập hợp R, lấy giá trị trong R và thoả mãnhệ thức.

f(x2 + y + f(y)) = (f(x))2 + αy

với mọi x, y thuộc R.(R ký hiệu tập hợp các số thực).Bài 3. Trong mặt phẳng, cho hai đường tròn (O1) và (O2) cắt nhau tại

hai điểm A và B. Các tiếp tuyến tại A và B của đường tròn (O1) cắt nhautại điểm K. Xét một điểm M (không trùng với A và B) nằm trên đườngtròn (O1). Gọi P là giao điểm thứ hai của đường thẳng MA và đường tròn(O2). Gọi C là giao điểm thứ hai của đường thẳng MK và đường tròn (O1).Gọi Q là giao điểm thứ hai của đường thẳng CA và đường tròn (O2). Chứngminh rằng:

1) Trung điểm của đoạn thẳng PQ nằm trên đường thẳng MC.2) Đường thẳng PQ luôn đi qua một điểm cố định khi điểm M di động

trên đường tròn (O1).((O) ký hiệu đường tròn tâm O).Bài 4. Cho dãy số (xn), n = 1, 2, 3, . . . xác định bởi

x1 = 603, x2 = 102 và xn+2 = xn+1+xn+2√

xn+1xn − 2 với mọi n ≥ 1

Chứng minh rằng:1) Tất cả các số hạng của dãy số đã cho đều là các số nguyên dương.2) Tồn tại vô số số nguyên dương n sao cho biểu diễn thập phân của xn

có bốn chữ số tận cùng là 2003.3) Không tồn tại số nguyên dương n mà biểu diễn thập phân của xn có

bốn chữ số tận cùng là 2004.Bài 5. Xét lục giác lồi ABCDEF . Gọi A1, B1, C1,D1, E1, F1 lần lượt là

trung điểm của các cạnh AB,BC,CD,DE,EF,FA. Ký hiệu p và p1 tươngứng là chu vi của lục giác ABCDEF và của lục giác A1B1C1D1E1F1. Giảsử lục giác A1B1C1D1E1F1 có tất cả các góc trong bằng nhau. Chứng minhrằng:

p ≥ 2√

3

3p1

Hỏi dấu đẳng thức xảy ra khi và chi khi nào?Bài 6. Cho S là một tập hợp gồm một số số nguyên dương mà số nhỏ

nhất và số lớn nhất trong S là hai số nguyên tố cùng nhau.

Page 18: Final Version 1989 to 2010 VN TST

1.11. Đề thi chọn đội tuyển toán năm học 2003 - 2004 (Ngày 7, 8/5/2004) 17

Với mỗi số tự nhiên n, ký hiệu Sn là tập hợp gồm tất cả các số tự nhiênmà mỗi số đều là tổng của nhiều nhất n số (không nhất thiết đôi một khácnhau) thuộc tập S. Quy ước 0 là tổng của 0 số thuộc S. Gọi a là số lớnnhất trong S. Chứng minh rằng tồn tại số nguyên dương k và số nguyên bsao cho

|Sn| = an + b

với mọi n > k.(|X| ký hiệu số phần tử của tập hợp X)

Page 19: Final Version 1989 to 2010 VN TST

Chương 2

Đáp án tuyển sinh

2.1 Đáp án chọn đội tuyển năm học 1991 -

1992

Bài 1. Trong trường hợp m chia hết cho n (kể cả khi m = 0 (nếu coi 0 làsố tự nhiên, chia hết cho n)), rõ ràng không có số nguyên k > 1 thoả mãnđề bài mà k ≤ n. Vậy k = n + 1.

Sau đây xét m > 0, m không chia hết cho n, (n > 1).với l ∈ Z. Xét

ϕ(l) : Zn = Z/nZ → Zn

x 7→ lm + x = lm + x

thì nó xác định tác động của nhóm cộng Z lên Zn. Nhóm dừng tại x gồmcác l ∈ Z mà lm = 0 tức là lm (một bội của m) là bội của n, vậy nhóm dừng

đó goòm các bội của BSCBN(m,n)m

= m.nd

. 1m

= nd

= n0, trong đó d = (m,n).

Từ đó mỗi quỹ đạo α của tác động nói trên có n0 phần tử, cụ thể là dãyxα thuộc α thì α = {ϕ(l)(xα) | l = 01, ..., n0 − 1} và Zn là hợp rời rạc củann0

= d quỹ đạo như thế.

Chú ý: do m không chia hết cho n nên n0 > 1. Vậy số N = d[n0

2]+1 > 1

và rõ ràng N ≤ n. Hãy chứng minh N bằng số k cần tìm.1) a1, a2, ..., aN là N phần tử phân biệt của Zn thì do có đúng d quỹ đạo

rời nhau nên có hơn [n0

2] phần tử ai đó nằm trong một quỹ đạo α nào đó và

do α có n0 phần tử, có ap, as thuộc α mà ϕ(1)(ap) = as, tức m + ap = as

hay m + ap − as = 0.2) Khi d = 1 và n0 = 2 hay 3 thì N = 2 rõ ràng có tính chất bé nhất

cần tìm, tức N = k.Trong các trường hợp khác thì N > 2 và lấy trong mỗi quỹ đạo α phần tử

xα thì tập hợp {ϕ(2l)(xα) | l = 0, 1, ..., [n0

2], α chạy qua tập các quĩu đạo}

18

Page 20: Final Version 1989 to 2010 VN TST

2.1. Đáp án chọn đội tuyển năm học 1991 - 1992 19

gồm N − 1 phần tử phân biệt của Zn mà không có hai phần tử khác nhaunào có hiệu bằng m. Vậy N có tính chất bé nhất đang xét.

Kết luận: m chia hết cho n (kể cả m = 0): k = n + 1. Còn các trườnghợp khác: đặt d = (m,n), n0 = n

dthì k = d[n0

2] + 1.

Bài 2. Do f(x) là đa thức có bậc không nhỏ hơn 1 nên |f(x)| → +∞khi |x| → +∞, vậy có x0 > 0 để |f(x)| > c với mọi x mà |x| > x0. Kí hiệun0 là số nguyên dương bé nhất thoả mãn n0!

2n0> x0. Hãy chứng minh n0 là

số cần tìm.Giả sử p(x) là đa thức có deg P = k ≥ n0 và có hệ số của số hạng bậc

k bằng 1. Với k + 1 số nguyên phân biệt tuỳ ý b1 < b2 < · · · < bk+1, theocông thức nội suy Lagrange, ta có

P (x) =k+1∑

i=1

P (bi)∏

j 6=i

(x − bj)

(bi − bj).

Tính chất hệ số của số hạng bậc cao nhất của P (x) bằng 1 cho

1 =

k+1∑

i=1

P (bi).∏

j 6=i

1

bi − bj

≤ max1≤i≤k+1

|P (bi)|.k+1∑

i=1

1

(bi − b1) · · · (bi − bi−1)(bi+1 − bi) · · · (bk+1 − bi)

≤ max1≤i≤k+1

|P (bi)|k+1∑

i=1

1

(i− 1)!(k + 1 − i)!(do bj − bl ≥ j − l ∀j > l)

≤ max1≤i≤k+1

|P (bi)|k∑

j=0

1

k!Cj

k =2k

k!

(max

1≤i≤k+1|P (bi)|

)

Từ đó

max1≤i≤k+1

|P (bi)| ≥k!

2k≥ n0!

2n0> x0

.Vậy có i ∈ {1, 2, ..., k + 1} để |f(P (bi))| > c, tức là số các số nguyên x

mà |f(P (x))| ≤ c không vượt quá k = deg P .Bài 3. Coi các tam giác cân A′BC,B ′CA,C ′AB có các đỉnh cân theo

thứ tự là A′, B′, C ′, (đỉnh cân đối diện với đáy).Coi tam giác ABC xác định hướng thuận trong mặt phẳng và đặt θ =

(AC ′, AB) = (BA′, BC) = (CB′, CA) (góc định hướng), ở đây −π2

< θ < π2,

θ 6= 0, và A′, B′, C ′ theo thứ tự thuộc các trung trực của BC,CA,AB.

1)−−→AA′ =

−→AB +

−−→BA′ =

−→AB +

−→f (

−−→BC),

−−→BB′ =

−−→BC +

−−→CB′ =

−−→BC +

−→f (

−→CA),

−−→CC ′ =

−→OA +

−−→AC ′ +

−→OA +

−→f (

−→AB), trong đó

−→f là tích, phép vị tự

Page 21: Final Version 1989 to 2010 VN TST

20 Chương 2. Đáp án tuyển sinh

(véctơ) hệ số 12 cos θ

với phép quay (véctơ) góc −θ. Vậy−−→AA′ +

−−→BB′ +

−−→CC ′ =

−→AB +

−−→BC +

−→CA +

−→f (

−−→BC +

−→CA +

−→AB) =

−→0 . Chú ý rằng

−−→AA′,

−−→BB′,

−−→CC ′

là những véctơ khác véctơ không (vì a, b, c khác nhau đôi một) nên suy ra

luôn có tam giác có cạnh dài AA′, BB′, CC ′ trừ khi và chỉ khi−−→AA′ song

song với−−→BB′.

2) Với hai véctơ−−→CM,

−−→ON trong mặt phẳng đã xác định hương, kí hiệu

−−→OM ×−−→

ON =

0 nếu−−→OM =

−→0 hay

−−→ON =

−→0

|−−→OM ||−−→ON | sin(OM,ON) nếu−−→OM 6 −→0 ,

−−→ON 6= −→

0 hay−−→ON =

−→0

Lấy hệ toạ độ Đềcac vuông góc định hương thuận trong mặt phẳng,−−→OM (x, y),

−−→ON (x′, y′) thì do sin(OM,ON) = sin((Ox,ON) − (Ox,OM)) tính được−−→OM ×−−→

ON = xy′− x′y.Từ đó dễ thấy (−−→OM +

−−→OM ′)×−−→

CN =−−→OM ×ON +

−−→OM ′ ×−−→

CN ,−−→OM × (

−−→ON +

−−→ON ′) =

−−→OM ×−−→

ON +−−→OM ×

−−→CN ′.

Trở lại bài toán: Dễ thấy từ định nghĩa−−→AA′ song song với

−−→BB′ khi và

chỉ khi−−→AA′ ×

−−→BB′ = 0.

Ta có

−−→AA′ ×

−−→BB′ = (

−→AB +

−−→BB′) × (

−−→BC +

−−→CB′) =

−→AB ×−−→

BC +−→AB ×

−−→CB′+

+−−→BA′ ×−−→

BC +−−→BA′ ×

−−→CB′

Tính từng số hạng của tổng này:−→AB×−−→

BC = 2S, S là diện tích của tam giác

ABC;−→AB × CB′ = c b

2cos θsin(θ − A), trong đó A = (AB,AC), để ý rằng

(AB,CB ′) = (AB,AC)+(AC,CA)+(CA,CB′) = A+π− θ;−−→BA′×−−→

BC =a

2cos θa sin θ;

−−→BA′ ×

−−→CB′ = a

2cos θb

2cos θsin C, trong đó C = (CA,AB), để ý

rằng (BA′, CB′) = (BA′, BC)+ (BC,CA)+ (CA,CB ′) = θ + π − C − θ =

π − C.

Page 22: Final Version 1989 to 2010 VN TST

2.1. Đáp án chọn đội tuyển năm học 1991 - 1992 21

Vậy

−−→AA′ ×

−−→BB′ = 2S +

bc

2 cos θsin(θ − A) +

a2 sin θ

2 cos θ+

ab

4 cos2 θsin C

=1

2S tan2 θ +

1

4(a2 + b2 + c2) tan θ +

3

2S.

Từ đó θ cần tìm là nghiệm của phương trình

S tan2 θ +1

2(a2 + b2 + c2) tan θ + 3S = 0.

Biệt thức của tam thức bậc hai đối với tan θ ở vế trái là

∆ =1

4(a2 + b2 + c2) − 12S2

=1

2{(a2 − b2)2 + (b2 − c2)2 + (c2 − a2)2} > 0.

(Để ý công thức Heron: 16S2 = 2(a2b2 + b2c2 + c2a2) − (a4 + b4 + c4). Vậyphương trình trên có hai nghiệm tan θ1 và tan θ2 phân biệt, âm

tan θ1, tan θ2 =1

4S{−(a2+b2+c2)±

√2√

(a2 − b2)2 + (b2 − c2)2 + (c2 − a2)2},

trong đó S = 14

√(a + b + c)(a + b − c)(a − b + c)(−a + b + c).

Coi các tam giác cân A′BC,B ′CA,C ′AB có đỉnh cân theo thứ tự là

B,C,A thì chúng minh theo đúng cách trên ta vẫn có−−→AA′+

−−→BB′+

−−→CC ′ =

−→0 ,

ở đây−→f là phép quay (véctơ) góc −θ (ở đây 0 < θ < π), còn

−−→AA′×

−−→BB′ =

4S−2S cos θ+ a2+b2+c2

2sin θ nên θ cần tìm phải là nghiệm của phương trình

lượng giácα sin θ + β cos θ = γ

trong đó, α = a2+b2+c2

2, β = −2S, γ = −4S. Gọi ϕ là góc mà sinϕ =

β√α2+β2

, cos ϕ = α√α2+β2

thì sin(θ+ϕ) = γ√α2+β2

= − 8S√16S2+(a2+b2+c2)2

, cho

hai nghiệm θ1, θ2 phân biệt.Nếu coi các tam giác cân A′BC,B ′CA,C ′AB có đỉnh cân theo thự tự

là C,A,B thì cũng chứng minh theo cách tương tự, trong kết quả vừa rồithay θ bởi π − 2θ, ở đây (−π

2< θ < π

2, θ 6= 0).

Bài 4. Kí hiệu A là tập hợp các hình tròn của họ tiếp xúc 6 hình trònkhác, B là tập các hình tròn còn lại.

Với mỗi hình tròn C0 của họ, kí hiệu L(C0) là tập giữa C0 và các hìnhtròn C của họ mà có dãy C1, C2, ..., Cm hình tròn của họ (m ≥ 1) để Ci tiếpxúc với Ci+1 (i = 0, 1, ..., n− 1), C = Cm.

Page 23: Final Version 1989 to 2010 VN TST

22 Chương 2. Đáp án tuyển sinh

1) Trong mỗi L(C0) phải có C ∈ B vì nếu mọi C ∈ L(C0) đều thuộc Athì xét hình tròn có bán kính bé nhất trong L(C0) suy ra mọi hình tròn của(C0) phải có bán kính bằng bán kính đó (*) và khi đó lưới các tâm các hìnhtròn của L(C0) không thể bị chặn, mâu thuẫn với họ hữu hạn hình tròn.

(*) suy ra từ: một hình tròn bán kính R không thể tiếp xúc ngoàivới sáu hình tròn ngoài nhau hay tiếp xúc ngoài nhau mà sáu bán kínhkhông nhỏ hơn R và có bán kính lớn hơn R. Thực vậy, nếu có hình vẽ bên:O1O2 = R1 + R2 + l, (R ≤ R1, R1 ≤ R2, 0 ≤ l) thì góc ϕ = (OO1, OO2)phải giữa 60◦ và 180◦ do

cos ϕ =(R + R1)

2 + (R + R2)2 − (R1 + R2 + l)2

2(R + R1)(R + R2)≤ 1

2.

2) Nếu có hai cách đặt f, g thỏa mãn đề bài thì f − g lấy giá trị 0 trênB và nó thỏa mãn điều kiện trung bình cộng của đề bài. Cần chứng minhf − g lấy giá trị 0 trên A. Giả sử có C ∈ A mà (f − g)(C) 6= 0; kíhiệu C0 là hình tròn của họ sao cho (f − g)(C0) = min

mọi C(f − g)(C) nếu

)f − g)(C) < 0 hay (f − g)(C0) = maxmọi C

(f − g)(C) nếu (f − g)(C) > 0,

thì C0 ∈ A và theo 1), trong L(C0) có dãy C1, ..., Cm (m > 1), Ci tiếpxúc Ci+1 (i = 0, 1, ...,m − 1), Ci ∈ A (i = 0, 1, ...,m − 1), Cm = C ∈ B.Do tính chất trung bình cộng của các số lơn hơn hoặc bằng a (trong cácsố nhỏ hơn hoặc bằng b) chỉ bằng a (theo thứ tự b) khi tất cả các số đóbằng a (theo thứ tự b) và do tính chất min, max nói trên , với C0 suy ra(f − g)(C0) = (f − g)(C1) = · · · = (f − g)(Cm−1) = (f − g)(C) = 0 vìC ∈ B), do đó có mâu thuẫn.

Chú ý: sau khi xét trường hợp (f − g)(C) < 0 có thể đưa trường hợp

(f − g)(C) > 0 vè trường hợp nó âm nhờ xét g − f thay cho f − g.

Bài 5. Dễ thấy rằng: Nếu (x, y) là nghiệm thì (y, x) là nghiệm. Nếukhông có nghiệm nguyên (x, x).

2) Xét ánh xạ f : R2 → R2, f((x, y)) = (x′ = 5x − y, y′ = x)a) dễ thấy rằng x′2 − 5x′y′ + y′2 = x2 − 5xy + y2.b) Kí hiệu S là tập các nghiệm nguyên dương thì dễ thấy rằng f(S) = S

Page 24: Final Version 1989 to 2010 VN TST

2.1. Đáp án chọn đội tuyển năm học 1991 - 1992 23

(để ý rằng phương trình có thể viết y(5x − y) = x2 + 5).c) Hãy chứng minh rằng với (x, y) ∈ S mà 1 < x < y thì f((x, y)) =

(x′, y′) có tính chất 1 ≤ x/ < x(= y/). Thực vậy, khi 1 < x < y = (x′, y′)có tính chất 1 < x < y mà x, y nguyên thì (y − x)2 + 5 = 3xy ≥ 3.2.3 nên(y−x)2 ≥ 13, vậy y−x ≥ 4. Khi đó nếu như 4x ≥ y thì 5xy = x2 +y2 +5 ≤x2 + 4xy + 5, do đó x2 + 5 ≥ xy hay x(y − x) ≤ 5, mà y − x ≥ 4 nên x = 1,trái với giả thiết x > 1. Vậy 4x < y, do đó x′ = 5x − y < x.

d) dễ thấy tập các nghiệm nguyên dạng (1, y) là S0{(1, 3), (1, 2)}. vậytừ c) và d) suy ra rằng nếu kí hiệu S1 = {(x, y) ∈ S | 1 < x < y}thì f(S1) ⊂ S1 ∪ S0 và với mọi (x, y) ∈ S1, có số nguyên dương k đểfk((x, y)) ∈ S0.

3) Đổi vai trò x với y trong 2) suy ra ánh xạ g : R2 → R2 : g((x, y)) =

(x′ = y, y′ = 5y − x) có tính chất g(S) ⊂ S và kí hiệu S1 = {(x, y) ∈ S |1 < y < x}, S0 = {(2, 1), (3, 1)} thì g(S1) ⊂ S1 ∪ S0, S0 ⊂ S, và với mọi

(x, y) ∈ S1, có số nguyên dương k để gk((x, y)) ∈ S0.4) Dễ thấy rằng g = f−1, do đó f |s, g |s là song ánh và để ý rằng

f(S0) = S0, suy ra S = {fk((1, 3)), fk((1, 2)) | k ∈ Z}. Chú ý rằng với mọik ∈ Z, fk là song ánh và khi k 6= 1 thì fk không có điểm bất động do nếu(x, y) ∈ S, f((x, y)) = (x′, y′) thì x′−y′ = 5x−y−x = 4x−y = x−y+3x >x − y, vậy các phần tử viết trên đây của S là đôi một khác nhau.

Có thể tính được (với mọi k ∈ Z)

{fk((1, 3)) = 1

α(λk+1

1 − λk+12 + 3(λk

2 − λk1), λ

k1 − λk

2 + 3(λk−12 − λk−1

1 ))

fk((1, 2)) = 1α(λk+1

1 − λk+12 + 2(λk

2 − λk1), λ

k1 − λk

2 + 2(λk−12 − λk−1

1 ))

trong đó λ1 = 12(5 + α), λ2 = 1

2(5 − α), α =

√21. Vài nghiệm

· · · 7→ (14, 67)f7→ (3, 14)

f7→ (1, 3)f7→ (2, 1)

f7→ (9, 2)f7→ (43, 9)

f7→ · · ·

· · · 7→ (9, 43)f7→ (2, 9)

f7→ (1, 2)f7→ (3, 1)

f7→ (14, 3)f7→ (67, 14)

f7→ · · ·Bài 6. Lập đồ thị G: đỉnh biểu diễn cho “ngôn ngữ", cạnh nối hai đỉnh

biểu diễn “người biết hai ngôn ngữ đó". Vậy G là đồ thị 2n đỉnh. Điều kiện“hai người biết chung nhiều nhất một ngôn ngữ" nói rằng G là đồ thị đơn.Điều kiện còn lại cho biết: với mỗi k nguyên 1 ≤ k ≤ n − 1 có không quák − 1 đỉnh, mỗi đỉnh có bậc nhỏ hơn hoặc bằng k (*).

Theo đề bài, cần chứng minh: từ tất cả các cạnh của G có thể ... 2ncạnh thuộc 2n đỉnh và mỗi đỉnh luôn thuộc đúng hai cạnh ... 2n cạnh đó.Để chứng minh điều này ta sẽ chứng minh:

Trong G tồn tại một đường đi khép kín có độ dài 2n và đi qua tất cảcác định của G (một đường đi như thế ta sẽ gọi là chu trình H. Ta chứng

Page 25: Final Version 1989 to 2010 VN TST

24 Chương 2. Đáp án tuyển sinh

minh điều này bằng phản chứng. Giả sử trong G không có chu trình H.Khi đó tập các đỉnh không kề nhau của G là không rỗng và hữu hạn. Bằngcách thêm dần hai cạnh nối hai đỉnh không kề nhau ta sẽ xây dựng đồ thị2n đỉnh G thoả mãn 1) (*), 2) trong G không có chu trình H 3) Khi thêm

cạnh nối hai đỉnh bất kì không kề nhau của G ta sẽ nhận được đồ thị cóchu trình H.

Xét G với v là đỉnh của G kí hiệu f(v) là bậc của v.

a) Từ 2) và 3) suy ra giữa hai đỉnh bất kì không kề nhau của G đều tồntại một đường đi nhận hai đỉnh ấy làm hai đầu mút, đi qua tất cả các đỉnhcủa G và có độ dài 2n − 1.

b) Nếu hai đỉnh v và v′ của G có f(v) ≥ n, f(v′) ≥ n thì v và v′ phải kềnhau.

Thật vậy, giả sử v và v′ không kề nhau thì có đường đi v1, v2, ..., v2n

(v1 ≡ v, v2n ≡ v′ đi qua tất cả các đỉnh của G và có độ dài 2n − 1. Giảsử f(v) = s ≥ n. Kí hiệu vi1, vi2, ..., vis (2 = i1 < i2 < · · · < is < 2n) làcác đỉnh kề với v1 ≡ v. Khi đó với mỗi j = 1, 2, ..., s các đỉnh v(ij)−1 không

kề với v2n ≡ v′ vì nếu ngược lại thì chu trình H trong G là v1v2...v(ij)−1v2n

v2n−1...vij mâu thuẫn với 2). Từ đó suy ra f(v′) ≤ 2n− (s− 1) ≤ n− 1 (do

s ≥ n), mâu thuẫn với f(v′) ≥ n. Vậy v, v′ phải kề nhau.

c) Từ b) suy ra tập v gồm các đỉnh v của G mà f(v) ≤ n − 1 là khôngrỗng, vậy có maxv∈V f(v) = m ≤ n−1. Lấy v1 mà f(v1) = m. Điều kiện (*)với k = n− 1 nói rằng có ít nhất 2n− (n− 1) + 1 = n + 2 đỉnh có bậc ≥ n,do với k = n− 1 nói rằng có ít nhất một trong các đỉnh này, chẳng hạn v2n,không kề với v1. Suy ra có đường đi v1, v2, ...v2n đi qua tất cả các đỉnh của Gvà có độ dài 2n−1. Kí hiệu vi1, vi2, ...vim (2 = i1 < i2 < · · · < im < 2n là cácđỉnh kề với v1 thì lập luận như ở b) chứng tỏ với mọi j = 1 → n ta có v(ij)−1

không kề với v2n (chú ý rằng điều kiện (*) với k = 1) chứng tỏ mọi đỉnh

của G có bậc ≥ 2. Áp dụng điều kiện (*) với k = m (2 ≤ m ≤ n − 1) suyra {v(i1)−1, v(i2)−1, ..., v(im)−1} phải có ít nhất một đỉnh vq có f(vq) ≥ m + 1. Từ định nghĩa của m suy ra f(vq) ≥ n. Như vậy, vq, v2n có f(vq) ≥ n,f(v2n) ≥ n mà không kề nhau, mâu thuẫn với b). Mâu thuẫn này cho tađiều phải chứng minh.

2.2 Đáp án chọn đội tuyển năm học 1992 -

1993

Bài 1. Trường hợp tổng quát khi đặt 1993 = 2m + 1 (dòng) và 2000 = 2n(cột) với m = 996 và n = 1000.

Gọi s là tổng số các hình vuông 2×2 và hình chữ nhật khuyết kép, gọi y

Page 26: Final Version 1989 to 2010 VN TST

2.2. Đáp án chọn đội tuyển năm học 1992 - 1993 25

là số các hình chữ nhật khuyết đơn. Ta có đẳng thức về diện tích các hình:

4s + 5y = 2n(2m + 1) (1)

Đánh dấu (×) vào các ô vuông có toạ độ (2r, 2t) với 1 6 r 6 m và1 6 t 6 n ta được m.n dấu (×). Dễ thấy trên hình:

+) Hình vuông 2×2 hoặc hình chữ nhật khuyết kép chứa đúng một dấu(×).

+) Hình chữ nhật khuyết đơn chứa một hoặc hai dấu (×). Từ đó đặt1 6 v 6 2, v nguyên, ta có bất đẳng thức sau về số dấu (×) trên hình

m.n = s.1 + y.v > s + y (2)

Từ (1) và (2) ta có 5m.n > 5(s + y) = 4s + 5y + s = 2n(2m + 1) + s hays 6 5m.n − 2n(2m + 1) = m.n− 2n = n(m − 2).

Áp dụng với m = 996 và n = 1000 có kết quả s 6 994000. Sự tồn tạicủa cách ghép với s = 994000 cho trên hình vẽ.

Page 27: Final Version 1989 to 2010 VN TST

26 Chương 2. Đáp án tuyển sinh

Bài 2. 1) Ta có

a3n+1 =

(an +

1√

an

)3

= a3n + 3an

√an + 3 +

1

an√

an

> a3n + 3an

√an +

9

4= (√

a3n +

3

2)2

⇒√

a3n+1 >

√a3

n +3

2⇒√

a3n >

3n

2∀n > 1 (1)

2) Mặt khác

an+1 =(√

an +1

2an

)2

− 1

4a2n

<(√

an +1

2an

)2

⇒ √an+1 <

√an +

1

2an

⇒√

a3n+1 <

√a3

n +3

2+

3

4.

1

an√

an+

1

8a3n

⇒√

a3n < 1 +

3n

2+

3

4

n−1∑

k=0

1

ak√

ak+

1

8

n−1∑

k=0

1

a3k

, ∀n > 1

Từ (1) suy ra với n > 1 thì

n−1∑

k=0

1

a3k

< 1 +n∑

k=1

1

a3k

< 1 +1

9

n∑

k=1

1

k3< K = const

n−1∑

k=0

1

ak√

ak< 1+

n∑

k=1

1

ak√

ak6 1+

√n

√√√√n∑

k=1

1

a3k

< 1+K ′√n với K ′ = const

Do vậy√

a3n <

3n

2+ K1 + K2.

√n ∀n > 1

ở đây K1,K2 = const3) Từ kết quả của 1) và 2) ta được:

3n

2< a

32n <

3n

2+ K1 + K2.

√n ∀n > 1

⇒ ∃ limn→∞

a32n

n=

3

2(2)

Page 28: Final Version 1989 to 2010 VN TST

2.2. Đáp án chọn đội tuyển năm học 1992 - 1993 27

Từ (1) ta có limn→∞ an = +∞. Do đó, kết hợp với (2) suy ra

limn→∞

aαn

n= +∞ với α >

3

2

limn→∞

aαn

n= 0 với α <

3

2

Vậy

limn→∞

aαn

n= L 6= 0 ⇔ α =

3

2.

Bài 3. 1) Tìm maxA. Viết lại A dưới dạng

A = 5(x21 + x2

4) + 6(x22 + x2

3) − 8(x1x2 + x2x3 + x3x4) + 2x1x3 + 2x2x4

Với mọi α > 0 ta có

−8x1x2 6 αx21 +

16

αx2

2 dấu ” = ” ⇔ αx1 = −4x2 (1)

−8x2x3 6 4(x22 + x2

3) dấu ” = ” ⇔ x2 = −x3 (2)

−8x3x4 6 αx24 +

16

αx2

3 dấu ” = ” ⇔ αx4 = −4x3 (3)

2x1x3 6α

4x2

1 +4

αx2

3 dấu ” = ” ⇔ αx1 = 4x3 (4)

2x2x4 64

αx2

2 +α

4x2

4 dấu ” = ” ⇔ αx4 = 4x2 (5)

Do vậy

A 6(5 +

4

)(x2

1 + x24) +

(10 +

20

α

)(x2

2 + x23)

Chọn α > 0 thoả mãn

5 +5α

4= 10 +

20

α⇔ α = 2(1 +

√5

. Khi đó

A 6 (5 +5α

4)(x2

1 + x22 + x2

3 + x24) 6 5 +

4=

5

2(3 +

√5)

Ta có A = 52(3 +

√5) ⇔ có đồng thời (1), (2), (3), (4), (5) và

∑4i=1 x2

i = 1

⇔ x1 = ±1

2

√1 − 1√

5, x2 = −1 +

√5

2x1, x3 =

1 +√

5

2x1, x4 = −x1

Page 29: Final Version 1989 to 2010 VN TST

28 Chương 2. Đáp án tuyển sinh

Vậy

maxA =5

2(3 +

√5)

2) Tìm minA: Lấy 1 < α < 2. Viết lại A dưới dạng

A = (x1 −αx2 +x3)2 +(x2−αx3 +x4)

2 +−2(4−α)x1x2− 4(2−α)x2x3−− 2(4 − α)x3x4 + 4(x2

1 + x24) + (5 − α2)(x2

2 + x23)

Do α < 2 nên ∀β > 0 ta có

−2(4 − α)x1x2 > −(4 − α)(βx2

1 +x2

2

β

)” = ” ⇔ βx1 = x2 (6)

−2(4 − α)x3x4 > −(4 − α)(βx2

4 +x2

3

β

)” = ” ⇔ βx4 = x3 (7)

−4(2 − α)x2x3 > −2(2 − α)(x22 + x2

3) ” = ” ⇔ x2 = x3 (8)

Suy ra

A > −2(4 − α)x1x2 − 4(2 − α)x2x3 − 2(4 − α)x3x4+

+ 4(x21 + x2

4) + (5 − α2)(x22 + x2

3)

> [4 − (4 − α)β](x21 + x2

4) + [1 + 2α − 4 − α

β− α2](x2

2 + x23)

Chọn β sao cho

4 − (4 − α)β = 1 + 2α − 4 − α

β− α2 (9)

Khi đó

A > [4 − (4 − α)β](x21 + x2

2 + x23 + x2

4) >1

2[4 − (4 − α)β]

Dấu ” = ” ở bất đẳng thức cuối cùng đạt được khi và chỉ khi có đồng thời

(I)

βx1 = x2

βx4 = x3

x2 = x3

x1 − αx2 + x3 = 0

x2 − αx3 + x4 = 0

x21 + x2

2 + x23 + x2

4 = 12

Page 30: Final Version 1989 to 2010 VN TST

2.2. Đáp án chọn đội tuyển năm học 1992 - 1993 29

Dễ thấy, các giá trị x1, x2, x3, x4 thoả mãn hệ trên là các giá trị có cùng dấu.Bởi vậy, để đơn giản ta sẽ chỉ xét xi = 0, i = 1, 4. Với điều kiện đó ta có

(I) ⇔

x2 = x3, x1 = x4

x1 = 1βx2

x1 = (α − 1)x2

2x21 + 2x2

2 = 12

x3 = x2, x4 = x1

x1 = (α − 1)x2

α − 1 = 1β

(10)

x1 = 1

2√

1+β2

Bây giờ ta sẽ xác định β nhờ vào (9) và (10). Thế (10) vào (9) ta được

4 +( 1

β− 3)β = 2 − 1

β2+

1

β

( 1

β− 3)

⇔β2 − β − 1 = 0 ⇔ β =1 +

√5

2(do β > 0)

Từ đó

x1 =1√

10 + 2√

5, x2 =

1 +√

5

2.

1√10 + 2

√5

Vậy

minA =7 − 3

√5

4

đạt được khi

x1 = x4 =1√

10 + 2√

5, x2 = x3 =

1 +√

5

2√

10 + 2√

5

Bài 4. (Hình học phẳng)

Từ hai đẳng thức quen thuộc (với a = BC, b = CA, c = AB)

−−→OH =

−→OA +

−−→OB +

−→OC (1)

Page 31: Final Version 1989 to 2010 VN TST

30 Chương 2. Đáp án tuyển sinh

vàa−→IA + b

−→IB + c

−→IC =

−→0 (2)

suy ra

(a + b + c)−→OI = a

−→OA + b

−−→OB + c

−→OC (3)

và(a + b + c)

−→IH = (b + c)

−→OA + (c + a)

−−→OB + (a + b)

−→OC (4)

Để ý rằng 2−−→OB.

−→OC = 2R2 − a2, 2

−→OC.

−→OA = 2R2 − b2, 2

−→OA.

−−→OB = 2R2 − c2

với R là bán kính đường tròn ngoại tiếp tam giác ABC, từ (3) và (4) suyra

(a + b + c)2−→OI2 = (a + b + c)2R2 − abc(a + b + c) (5)

(a + b + c)−→IH2 = 4(a + b + c)2R2 − (a + b + c)(a3 + b3 + c3 + abc) (6)

Gọi r là bán kính đường tròn nội tiếp 4ABC thì abca+b+c

= 2Rr nên từ (5)

và (6) có:

OI2 = R2 − abc

a + b + c= R(R − 2r)

IH2 = 4R2 − (a3 + b3 + c3 − 3abc) + 4abc

a + b + c= 4R(R − 2r) − (a2 + b2 + c2 − bc− ca − ab)

= 4OI2 − 1

2[(b− c)2 + (c − a)2 + (a− b)2]

Từ đó IH 6 2IO và dấu bằng xảy ra khi và chỉ khi a = b = c tức là 4ABClà tam giác đều.

Bài 5. Ký hiệu p(n) là ước nguyên tố lớn nhất của n. Ký hiệu α(n) làsố mũ của p(n) trong phân tích nguyên tố của n.

1) Trước hết, chứng minh nhận xét sau:Nhận xét: Nếu p(x0) > k thì tồn tại ít nhất một chỉ số m1 > 0 sao cho

p(xm1) < p(x0).Chứng minh: Giả sử x0 = pα1

1 pα22 · · · pαr

r với p1 < p2 < · · · < pr là tất cảcác ước nguyên tố của x0. Khi đó.

x1 = f(x0) = kpα1−11 pα2−1

2 · · · pαr−1r (p1 − 1)(p2 − 1) . . . (pr − 1) (1)

Vì pr > k > 1 nên từ (1) suy ra:+) Nếu α(x0) = αr > 1 thì p(x1) = p(x0) = pr và α(x1) = α(x0) − 1+) Nếu α(x0) = αr = 1 thì p(x1) < p(x0) = pr Từ đó suy ra p(xαr ) <

p(x0). Nhận xét được chứng minh.

Page 32: Final Version 1989 to 2010 VN TST

2.2. Đáp án chọn đội tuyển năm học 1992 - 1993 31

2) Từ nhận xét trên suy ra nếu p(x0) > k thì sẽ tồn tại chỉ số M saocho p(xM ) 6 k. Như thế, ∀a > 1 luôn ∃M > 0 sao cho p(xM) 6 k. Do đó

+) Với k = 2 thì p(xM ) = 2 suy ra xM có dạng xM = 2α, α ∈ N∗ hayxM+1 = f(xM ) = 2α. Suy ra xm = xM ,∀m > M , vậy dãy {xM}∞0 là dãy bịchặn ∀a > 1.

+) Với k = 3 thì p(xM ) = 2 hoặc p(xM ) = 3. Do vậy, xM sẽ có mộttrong các dạng sau:

∗) xM = 2α.3β với α, β > 1

∗) xM = 3β với β > 1

∗) xM = 2α với α > 1

- Xét xM = 2α.3β. Khi đó xM+1 = f(xM ) = 2α.3β ⇒ xm = xM ,∀m > M- Xét xM = 3β. Khi đó xM+1 = f(xM) = 2.3β ⇒ xm = xM+1,∀m >

M + 1 (theo kết quả của bước trên).- Xét xM = 2α. Khi đó

xM+1 = f(xM ) =

{3 nếu α = 1

3.2α−1 nếu α > 1

Từ đó, theo kết quả của hai bước trên, ta sẽ có xm = xM+2,∀m > M + 2(trường hợp α = 1) hoặc xm = xM+1,∀m > M + 1 (trường hợp α > 1).

Tóm lại, với k = 3 thì dãy {xm}∞0 là dãy bị chặn ∀a > 1.Xét k > 3. Với chú ý rằng: ∀m ∈ N,∀k > 1 nếu p(xm) 6 k thì p(xm+1) 6

k (dễ thấy từ cách xác định dãy {xm}∞0 ), ta thấy p(xm) 6 k,∀m > M . Dođó, ∀m > M ta có:

xm+1 = f(xm) = kxm(1 − 1

p1) · · · (1 − 1

pr)

> kxm(1 − 1

2)(1 − 1

3) · · · (1 − 1

k) (do pr 6 k và k > 3)

= xm. (2)

Mà dãy {xm}∞0 là dãy các số nguyên dương, nên từ (2) suy ra dãy {xm}∞0là dãy không bị chặn ∀a > 1.

Vậy dãy {xm}∞0 là dãy bị chặn ∀a > 1 khi và chỉ khi k = 2 hoặc k = 3.Bài 6. Xét n điểm A1, A2, . . . , An mà có thể tô màu tất cả các đoạn

AiAj thoả mãn đề bài. Xét grafG có tập đỉnh V = {A1, A2, . . . , An} và tậpcạnh là các đoạn được tô màu xanh. Dễ thấy G đơn, vô hướng, n đỉnh vàthoả mãn:

a) d(Ai) 6 4,∀i = 1, n (d(Ai) ký hiệu bậc của đỉnh Ai).

Page 33: Final Version 1989 to 2010 VN TST

32 Chương 2. Đáp án tuyển sinh

b) Với bất cứ hai đỉnh Ai, Aj nào cũng đều tồn tại một xích đơn nốichúng và có độ dài nhỏ hơn hoặc bằng 2.

Vấn đề đặt ra ở bài đã ra tương đương với tìm số đỉnh n lớn nhất củagrafG đơn, vô hướng và thoả mãn a) và b).

Xét G đơn, vô hướng, n đỉnh và thoả mãn a) và b). Xét một đỉnh Ai

bất kỳ của G. Khi đó mỗi đỉnh trong số n− 1 đỉnh còn lại phải hoặc kề vớiAi hoặc kề với ít nhất một đỉnh kề với Ai (theo b)). Kết hợp với a) suy ran 6 1 + 4 + 3 × 4 = 17.

1) Xét n=17: Khi đó dễ thấy, phải có

d(Ai) = 4 ∀i = 1, 17 (*)

và do đó G có tất cả 4×172

= 34 cạnh.

Hình 1Xét đỉnh Ai bất kỳ của G. Từ (*) suy ra Ai kề với đúng 4 đỉnh khác,

giả sử là Ai1, Ai2, Ai3, Ai4. Qui ước gọi tất cả các đỉnh còn lại của G là cácđỉnh rìa, và gọi tất cả các cạnh có cả hai đầu mút là hai đỉnh rìa là các cạnhrìa. Từ b) và (*) suy ra mỗi đỉnh Aij, (j =

−→1, 4) (xem H.1). Từ đó dễ thấy

không có hai đỉnh nào của G cùng với Ai lập thành nhóm ba đỉnh đôi mộtkề nhau, nên trong G không có ba đỉnh nào đôi một kề nhau (vì Ai lấy raxét là đỉnh bất kỳ). Vậy mỗi cạnh rìa đều có hai đầu mút là hai đỉnh rìakhông cùng kề với một đỉnh Aij suy ra mỗi cạnh rìa cho ta một chu trìnhđơn độ dài 5 và đi qua Ai. Mà số cạnh rìa có tất cả 34 − 16 = 18, nên cótất cả 18 chu trình đơn độ dài 5 và đi qua Ai. Vì Ai là đỉnh bất kỳ nên từđó suy ra số chu trình đơn độ dài 5 trong G có tất cả là: 18×17

5/∈ Z, vô lý.

Vậy n 6= 17.2) Xét n=16. Khi đó dễ thấy, phải có

d(Ai) = 4 ∀i = 1, 16 (1)

và do đó G có tất cả 16×42

= 32 cạnh. Xét một đỉnh Ai bất kỳ của G. Theo

(1), Ai kề với đúng 4 đỉnh khác, giả sử là Ai1, Ai2, Ai3, Ai4. Tiếp tục, bằng

Page 34: Final Version 1989 to 2010 VN TST

2.2. Đáp án chọn đội tuyển năm học 1992 - 1993 33

phương pháp lập luận như ở 1), ta sẽ được: mỗi đỉnh Aij, (j = 1, 4) đều kềvới đúng ba đỉnh rìa và có đúng một đỉnh rìa, tạm gọi là Ak, kề với đúnghai đỉnh Aij (xem H.2). Từ đó, do Ai là bất kỳ nên suy ra: Trong G không

có ba đỉnh nào đôi một kề nhau, suy ra mỗi cạnh rìa không liên thuộc Ak

cho ta đúng một chu trình đơn độ dài 5 và đi qua Ai, còn mỗi cạnh rìa liênthuộc Ak cho ta đúng hai chu trình đơn độ dài 5 và đi qua Ai. Mà số cạnhrìa có tất cả là 32 − 16 = 16 và trong số này có đúng hai cạnh liên thuộcAk (do d(Ak) = 4), nên suy ra có tất cả 14 × 1 + 2 × 2 = 18 chu trình đơnđộ dài 5 đi qua Ai. Vì Ai bất kỳ nên suy ra số chu trình đơn độ dài 5 trongG có tất cả là 18×16

5/∈ Z, vô lý. Vậy n 6= 16.

Hình 2

3) Xét n = 15. Ta có G được mô tả ở (H.3) thoả mãn mọi yêu cầu đặtra.

Hình 3

Vậy nmax = 15.

Bình luận:

1) Việc xây dựng G có 15 đỉnh xuất phát từ graf quen thuộc (grafPetersơn) (H.4) và bởi vậy G còn có thể mô tả như sau (H.5).

Page 35: Final Version 1989 to 2010 VN TST

34 Chương 2. Đáp án tuyển sinh

2) Có thể xét trường hợp n = 16 bằng cách khác dễ lập luận chặt chẽhơn. Tuy nhiên việc xét như đã trình bày ở trên đảm bảo cho lời giải nhấtquán về phương pháp.

Hình 4

Hình 5

3) Lời giải trên chỉ dùng cho người chấm thi.

2.3 Đáp án chọn đội tuyển năm học 1993 -

1994

Bài 1. a) Đặt BE = x, AB = CD = a, CF = y, AD = BC = b. TừSABE = SBCF có

1

2.AB.BE sinα =

1

2.BC.CF sin(1800 − α)

⇔ AB.BE = BC.CF

⇔ a.x = b.y (1)

b) Kẻ DB//AE cắt tia BC tại P thì CP = BE = x. Kẻ BQ//AF cắt tia

Page 36: Final Version 1989 to 2010 VN TST

2.3. Đáp án chọn đội tuyển năm học 1993 - 1994 35

DC tại Q thì CQ = FQ− CF = a − y. Theo định lý Talet, ta có:

BM

BD=

BE

BP=

x

b + x(2)

DN

DB=

DF

DQ=

a − y

2a − y(3)

Từ đó tính

MN2 −BM2 − DN2 = (BD − BM −DN)2 − BM2 − DN2

= BD2 − 2BD.BM − 2BD.DN + 2BM.DN

hayMN2 −BM2 − DN2

BD2= 1 − 2

BM

BD− 2

DN

BD+ 2

BM

BD.DN

BD(4)

Thay (2),(3) vào (4) ta được

MN2 − BM2 − DN2

BD2= 1 − 2x

b + x− 2(a − y)

2a − y+ 2

x

b + x

a− y

2a − y=

=1

(b + x)(2a− y)[(b + x)(2a − y)− 2x(2a − y)−

− 2(a − y)(b + x) + 2x(a − y)]

=(2a − y)(b− x) − 2(a − y)b

(b + x)(2a − y)=

yb + yx− 2ax

(b + x)(2a − y)(5)

Thay (1) vào tử số của (5) được

yb + yx − 2ax = ax + yx − 2ax = yx− ax = −x(a− y)

Vậy

MN2 − BM2 − DN2

BD2= −

( x

b + x

)( a − y

2a − y

)= −BM

BD.DN

BD

hayMN2 = BM2 + DN2 − BM.DN (6)

Page 37: Final Version 1989 to 2010 VN TST

36 Chương 2. Đáp án tuyển sinh

Nếu dựng 4M ′N ′S sao cho SM ′ = BM,SN ′ = DN và góc đối diện vớiM ′N ′ bằng 600 thì

M ′N ′2 = SM ′2 + SN ′2 − SM ′.SN ′ (7)

So sánh (6) và (7) ta có MN = M ′N ′, nghĩa là tồn tại 4AM ′N ′ có cáccạnh tương ứng bằng MN,BM,DN . Gọi R là bán kính đường tròn ngoạitiếp 4SM ′N ′. Theo định lý hàm số sin ta có

MN = M ′N ′ = 2R sin 600 =√

3R

nên MN ′ và R cùng giảm.Bài 1. a) Xét biểu thức

ϕ =BM2 + DN2 − MN2

2BM.DN

chỉ cần chứng minh −1 6 ϕ 6 1 (ϕ đó sẽ là cosin của góc α đối diện cạnhcủa tam giác mà ta cần chứng minh sự tồn tại). Kẻ MM1, NN1 song songDA (M1, N1 thuộc AB) thì do Talet

ϕ =BM2

1 + AN22 − M1N

21

2BM1.AN1

Với mỗi điểm P trong hình bình hành ABCD, kẻ PP1 song song DA (P1 ∈AB), kẻ PP2 song song AB (P2 ∈ AD), gọi x(P ) = AP1

AB, y(P ) = AP2

AD.

Giả thiết choBE

AD=

FC

AB= t 0 < t < 1

P thuộc đường chéo BD ⇔ x(P ) + y(P ) = 1 (*)

P thuộc AE ⇔ y(P ) = tx(P ) (**)

P thuộc AF ⇔ y(P ) =1

1 − tx(P ) (***)

Page 38: Final Version 1989 to 2010 VN TST

2.3. Đáp án chọn đội tuyển năm học 1993 - 1994 37

Từ (*), (**) suy ra

x(M1) =AM1

AB=

1

1 + t

Từ (*), (***) suy ra

x(N1) =AN1

AB=

1 − t

2 − t

Từ đó

M1B

AB= 1 − x(M1) =

t

1 + t,

AN1

AB= x(N1) =

1 − t

2 − t

M1N1

AB= x(M1) − x(N1) =

t2 − t + 1

(1 + t)(2 − t)

Vậy

ϕ =

(t

1+t

)2

+(

1−t2−t

)2

−(

t2−t+1(1+t)(2−t)

)2

2 t(1−t)(1+t)(2−t)

dễ tính ϕ = 12

Vậy cosα = 12.

b) Bán kính R đường tròn ngoại tiếp tam giác vừa xây dựng xác địnhbởi

R =MN

2 sin α=

1√3MN

nên khi MN càng nhỏ thì R càng nhỏ.Bài 2. a)

x2 + y2 + z2 + t2 −Nxyzt −N = 0 (1)

⇔ t(t −Nxyzt) = N − (x2 + y2 + z2) (2)

Với ba số nguyên dương bất kỳ a, b, c và N = a2+b2 +c2 thì dễ thấy phươngtrình (2) có nghiệm

x0 = a, y0 = b, z0 = c, t0 = Nabc = (a2 + b2 + c2)abc (*)

Chú ý rằng khi hoán vị bốn số a, b, c,Nabc ta lại được nghiệm (x1, y1, z1, t1)của phương trình (1).

b) Giả sử phương trình (1) có nghiệm nguyên dương, chọn (x0, y0, z0, t0)là nghiệm nguyên dương của (1) sao cho tổng x0 + y0 + z0 + t0 là số nguyên

Page 39: Final Version 1989 to 2010 VN TST

38 Chương 2. Đáp án tuyển sinh

dương nhỏ nhất. Không làm mất tính chất tổng quát, giả định rằng x0 6y0 6 z0 6 t0.

Ta sẽ chứng minh rằng với N > 7 thì nghiệm nguyên dương của phươngtrình (1) với x0 6 y0 6 z0 6 t0 nếu có phải có dạng (*) như trên.

Theo giả thiết t0 là nghiệm của phương trình bậc hai

t2 − Nx0y0z0t + x20 + y2

0 + z20 − N = 0 (3)

Phương trình (3) có nghiệm thứ hai t1 thoả mãn:

t1 + t0 = Nx0y0z0 (4)

t1.t0 = x20 + y2

0 + z20 − N (5)

Từ (4) suy ra t1 ∈ Z. Lại theo giả thiết

N(1 + x0y0z0t1) = t21 + x20 + y2

0 + z20 > 0

nên

t1 > − 1

x0y0z0

Vì t1 ∈ Z nên t1 > 0.Giả sử t1 > 0 khi đó (x0, y0, z0, t1) là nghiệm nguyên dương của (1). Do

cách chọn (x0, y0, z0, t0) thì x0 + y0 + z0 + t1 > x0 + y0 + z0 + t0 nên t1 > t0.Từ đó theo (5) ta có

t20 6 t1t0 = x20 + y2

0 + z20 − N < x2

0 + y20 + z2

0 6 3z20

Ta có

N(1+x0y0z20) 6 N(1+x0y0z0t0) = x2

0+y20+z2

0 +t20 6 z20 +z2

0 +z20 +3z2

0 = 6z20

Từ đó, do N > 7 nên N(1 + x0y0z20) 6 6z2

0 < Nz20 suy ra 1 + x0y0z

20 < z2

0.Điều vô lý này chứng tỏ t1 > 0 là sai, suy ra t1 = 0. Từ (4), (5) suy ra

N = x20 + y2

0 + z20 và t0 = Nx0y0z0

là nghiệm (*) của phương trình (1).Với N = 4k(8m+7) > 7, áp dụng kết quả trên thì N = x2 + y2 + z2. Do

đó nếu chứng minh được phương trình x2 + y2 + z2 = 4k(8m + 7) không cónghiệm nguyên dương thì phương trình (1) cũng không có nghiệm nguyêndương.

+) Khi k = 0 ta có x2 + y2 + z2 = 8m+7 hay x2 + y2 + z2 ≡ 7 (mod 8).Trong ba số x, y, z phải có một số lẻ hoặc cả ba số lẻ. Nếu số a lẻ thì a2 ≡ 1(mod 8), do đó x2 + y2 + z2 6= 7 (mod 8).

Page 40: Final Version 1989 to 2010 VN TST

2.3. Đáp án chọn đội tuyển năm học 1993 - 1994 39

+) Khi k > 0 ta có

x2 + y2 + z2 = 4k(8m + 7) (*)

hay x2+y2+z2 ≡ 0 (mod 4). Trong ba số x, y, z phải có một số chẵn hoặc basố chẵn. Nếu có một số chẵn, còn hai số a, b lẻ thì a2+b2 ≡ 2 (mod 4)), suy rax2+y2+z2 6= 7 (mod 8). Nếu x, y, z đều chẵn, đặt x = 2x1, y = 2y1, z = 2z1

thì (*) tương đương với x2 + y2 + z2 = 4k−1(8m + 7). Sau k lần biến đổinhư thế ta có x2 + y2 + z2 = 8m + 7, nhưng phương trình này vô nghiệmnguyên dương như khi xét k = 0

Bài 3.a)

1 − 4x

x2P (x) +

(1 − 1 − 4x

x2

)P ′(x) − P ′′(x) = 0 (1)

⇔ 1 − 4x

x2(P (x) − P ′(x)) + (P ′(x)− P ′′(x)) = 0 (2)

Đặt Q(x) = P (x) − P ′(x) thì Q′(x) = P ′(x) − P ′′(x) nên

(2) ⇔ 1 − 4x

x2Q(x) + Q′(x) = 0 (3)

b) Ta chứng minh nếu đa thức bậc bốn P (x) có bốn nghiệm dương thìđa thức bậc bốn Q(x) = P (x) − P ′(x) cũng có bốn nghiệm dương. Khôngmất tính chất tổng quát, giả định rằng hệ số bậc cao nhất của P (x) là 1.Đặt

P (x) = x4 − ax3 + bx2 − cx + d = (x − x1)(x − x2)(x − x3)(x − x4)

với x1, x2, x3, x4 là các nghiệm dương. Từ đó theo Định lý Viet thì a, b, c, d >0.

P ′(x) = (x − x2)(x − x3)(x − x4) + (x − x1)(x − x3)(x − x4)+

+(x − x1)(x − x2)(x − x4) + (x − x1)(x − x2)(x − x3)

Q(x) = P (x) − P ′(x) = x4 + a1x3 + b1x

2 + c1x + c + d

Vì Q(x1)Q(x2) < 0, Q(x2)Q(x3) < 0, Q(x3)Q(x4) < 0 nên Q(x) có banghiệm dương là y1, y2, y3. Gọi nghiệm thứ tư là y4 thì y1y2y3y4 = c+ d > 0nên y4 > 0. Vậy Q(x) có bốn nghiệm dương.

c) Đặt R(t) = t4Q(1t). Vì Q(x) có bốn nghiệm dương thì R(t) cũng có

bốn nghiệm dương, do đó lại áp dụng kết quả trên, đa thức R(t) − R′(t)cũng có bốn nghiệm dương.

R(t) − R′(t) = t4Q(1

t) −

[4t3Q(

1

t) − t4

t2Q′(

1

t)]

= (t4 − 4t3)Q(1

t) + t2Q′(

1

t)

Page 41: Final Version 1989 to 2010 VN TST

40 Chương 2. Đáp án tuyển sinh

Hay phương trình sau có bốn nghiệm dương

(t4 − 4t)Q(1

t) + Q′(

1

t) = 0 (4)

Đặt x = 1t

thì phương trình (4) trở thành phương trình (3), nên phương

trình (3)1 − 4x

x2Q(x) + Q′(x) = 0

cũng có bốn nghiệm dương.Bài 3. Đặt P1(x) = e−xP (x). Vì đa thức P (x) có bốn nghiệm dương

nên phương trình P1(x) = 0 có bốn nghiệm dương. Suy ra, phương trình

P ′1(x) = 0 (1)

có ba nghiệm dương. Ta có:

(1) ⇔ e−x[P (x) − P ′(x)] = 0 ⇔ P (x) − P ′(x) = 0

Như vậy, đa thức Q(x) = P (x) − P ′(x) có ba nghiệm dương, giả sử làx1, x2, x3. Tuy nhiên, do degQ(x) = 4 (vì degP (x) = 4) nên Q(x) còn cónghiệm thực thứ tư x4.

Vì đa thức bậc bốn P (x) có bốn nghiệm dương nên không mất tổngquát, có thể coi P (x) có dạng

P (x) = ax4 − bx3 + cx2 − dx + e, với a, b, c, d, e > 0

Từ đó

Q(x) = ax4 + · · · + (−d − 2c)x + (e + d)

Suy ra (theo Định lý Viet)

x1x2x3x4 =e + d

4> 0 ⇒ x4 > 0

Vậy Q(x) có bốn nghiệm dương.Xét đa thức (biến t):

R(t) = t4Q(1

t)

. Dễ thấy degR(t) = 4, R(t) có bốn nghiệm dương. Do đó, theo kết quảphần trên ta có phương trình

R(t) − R′(t) = 0 (2)

Page 42: Final Version 1989 to 2010 VN TST

2.3. Đáp án chọn đội tuyển năm học 1993 - 1994 41

có bốn nghiệm dương. Ta có

(2) ⇔ t4[P (1

t) − P ′(

1

t)] − {t4[P (

1

t) − P ′(

1

t)]}′ = 0

⇔ t4[P (1

t) − P ′(

1

t)] − {4t3[P (

1

t) − P ′(

1

t)] + t4[− 1

t2P ′(

1

t) +

1

t2P ′′(

1

t)]}

⇔ (t2 − 4t)P (1

t) + (1 + 4t − t2)P ′(

1

t) − P ′′(

1

t) = 0 (3)

Đặt 1t

= x. Từ phương trình (3) ta có phương trình (ẩn x):

1 − 4x

x2P (x) +

(1 − 1 − 4x

x2

)P ′(x) − P ′′(x) = 0 (4)

Do (3) có bốn nghiệm dương nên phương trình (4) có bốn nghiệm dương.(Đpcm).

Bài 4. Lấy tâm O của tam giác đều ABC làm gốc của mặt phẳng phứcC, coi A,B,C có toạ vị a, b, c thì b = ae

2iπ3 , c = ae

4iπ3 . Nếu gọi M có toạ vị

z0 thì A′, B′, C ′ có toạ vị a′ = 2z0 − a, b′ = 2z0 − b, c′ = 2z0 − c.

Gọi P là điểm mà B′AP là "tam giác" đều định hướng thuận (có thể

suy biến thành 1 điểm nếu B′ ≡ A) tức P có toạ vị p′ mà p−b′ = eiπ3 (a−b′).

Vậy với chú ý b = ce−2iπ

3 , a + b + c = 0, và 1 − eiπ3 = e

−iπ3 , e

−2iπ3 − e

iπ3 =

eiπ3 (eiπ − 1) = 0 ta có

p = b′ + eiπ3 (a − b′) = 2z0 + b + e

iπ3 (a − 2z0 + b)

= 2z0 + ce−2iπ

3 − eiπ3 (c + 2z0)

= 2z0(1 − eiπ3 ) + c(e

−2iπ3 − e

iπ3 )

= 2z0e−iπ3

Vì kết quả không phụ thuộc a, b, c nên bằng cách hoán vị vòng quanhA,B,C được C ′BP,A′CP cũng là tam giác đều định hướng thuận. Vậy Pcách đều B′ và A, C ′ và B, A′ và C.

Page 43: Final Version 1989 to 2010 VN TST

42 Chương 2. Đáp án tuyển sinh

Nếu còn có Q 6= P cách đều các cặp điểm đó thì−→PQ 6= −→

0 trực giao với−−→AB′,

−−→BC ′,

−−→CA′ khi đó ba véctơ này cùng phương tức là O cùng các điểm có

toạ vị b′ − a = 2z0 + c, c′ − b = 2z0 + a, a′ − c = 2z0 + b phải thẳng hàng,khi đó O và các điểm có toạ vị a− b = a(1− e

2iπ3 ), a− c = a(1− e

4iπ3 ) phải

thẳng hàng là điều vô lý.

b) Xét biểu thức đồng dạng f xác định bởi z0 7→ f(z0) = p = 2z0e−iπ3

(tích quay góc −π3

với vị trí tỷ số 2 cùng tâm O). Trung điểm D của AB có

toạ vị a+b2

nên f(D) có toạ vị (a + b)e−iπ3 = −ce

−iπ3 = −ae

4iπ3 e

−iπ3 = a, vậy

f(D) = A. Vậy có biến đổi đồng dạng f, f(0) = 0, f(D) = A, f(M) = P , từđó f biến đường thẳng DM(M 6= D) thành đường thẳng AP . Hai đườngthẳng này phải cắt nhau tại một điểm N mà có một véctơ chỉ phương đườngthẳng DM tạo với một véctơ chỉ phương đường thẳng AP tạo thành góc−π

3, tức góc định hướng giữa hai đường thẳng đó là −π

3hay 2π

3. Khi M ≡ O

thì P ≡ O, MNP không tạo thành tam giác; Khi M 6= O, góc định hướnggiữa hai đường thẳng OM,OP cũng là −π

3hay 2π

3, nên tứ giác MNPO nội

tiếp: đường tròn ngoại tiếp MNP qua tâm O của ABC. (Điều vừa chứngminh đúng cho mọi đồng dạng thuận)

Bài 4. Qua phép đối xứng tâm SM = R1800

M thì

G → G′

4ABC → A′B′C ′

Qua phép quay

R1200

G′ : 4A′B′C ′ → 4B′C ′A′

trong đó G,G′ là trọng tâm 4ABC,4A′B′C ′

4ABCR1800

M−−−→ 4A′B′C ′ R1200

G′−−−→ 4B′C ′A′

Tích hai phép quay có thể là phép tịnh tiến khi tổng của hai góc quay

là bội của 1800, có thể là phép quay trong trường hợp còn lại. Ở đây tổng

Page 44: Final Version 1989 to 2010 VN TST

2.3. Đáp án chọn đội tuyển năm học 1993 - 1994 43

hai góc quay là 1800 + 1200 = 3000 ≡ −600 (mod 3600) nên tích là phép

quay tâm P xác định R1200

G′ .R1800

M = R−600

P . Qua phép quay R−600

P thì

4ABC → 4B′C ′A′

G → G′

nên tồn tại duy nhất điểm P cách đều A và B′,cách đều B và C ′, cách đềuC và A′.

Từ đó PG = PG′ và (PG,PG′) = −600. Mặt khác SM : G → G′ nên Mlà trung điểm của GG′. Vậy 4PGG′ là tam giác đều và GM⊥MP,GM =MP

2.

b) Xét phép đồng dạng S là phép vị tự quay S = R(GA,GD)G .H

12G thì

S = 4GPA → 4GMD

trong đó (GA,GD) = 600. Vì đường thẳng AP biến thành đường thẳng

DM nên góc MNP = 600 hoặc MNP = 1200 tuỳ theo góc MNP là nhọn

hoặc tù. Vì PGM = 600 nên nếu MNP = 600 là góc nhọn thì G và N đềunhìn đoạn PM dưới một góc 600 suy ra G,N,M,P cùng thuộc một đường

tròn, nếu MNP = 1200 là góc tù thì MNP + MGP = 1200 + 600 = 1800

nên tứ giác GMNP là tứ giác nội tiếp trong một đường tròn. Đường trònnày luôn đi qua điểm G là trọng tâm 4ABC, nên G là điểm cố định.

Cách khác giải câu a: Dựng tam giác đều AB′P ngược hướng với tamgiác đều ABC. Ta chứng minh rằng các 4BC ′P,4CA′P cũng là tam giácđều và cùng hướng với 4AB′P . Thật vậy

−−→BC ′ =

−−→B′C =

−→CA +

−−→AB′ = f(

−→BA) + f(

−→AB) = f(

−−→BP )

trong đó f là phép quay véctơ của mặt phẳng, góc +600. Chứng minh tương

tự ta cũng có−−→CA′ = f(

−→CP ). Bởi vậy 4BC ′P và 4CA′P là các tam giác

đều, cùng hướng với 4AB′P , do đó trung trực của AB′, BC ′, CA′ đồng quyở tâm quay P , góc quay bằng 600, biến 4ABC thành 4B′C ′A′.

Bài 5.

f(√

2x) + f((4 + 3√

2)x) = 2f((2 +√

2)x) ∀x (1)

a) Với x = 0 thì ta có 2f(0) = 2f(0), vậy f(0) = a là hằng số tuỳ ý.b) Với x > 0 ta đặt (2 +

√2)x = t hay x = t

2+√

2. Lúc đó (1) trở thành

f( √

2

2 +√

2t)

+ f(4 + 3

√2

2 +√

2t)

= 2f(t) ∀t > 0 (2)

Page 45: Final Version 1989 to 2010 VN TST

44 Chương 2. Đáp án tuyển sinh

Ta chú ý rằng√

22+

√2

= 1√2+1

=√

2 − 1 và

√2

2 +√

2.4 + 3

√2

2 +√

2=

4√

2 + 6

6 + 4√

2= 1 ⇒ 4 + 3

√2

2 +√

2=

1√2 − 1

.

Ta đặt t = (√

2 − 1)u thì (2) trở thành

f((√

2 − 1)u+1)

+ f((√

2 − 1)u−1)

= 2f((√

2 − 1)u)

∀u (3)

Lại đặt f((√

2 − 1)u)

= g(u) thì (3) trở thành

g(u + 1) + g(u − 1) = 2g(u) ∀u (4)

⇔g(u + 1) − g(u) = g(u) − g(u − 1) ∀u

Đặt g(u+1)− g(u) = h(u) thì h(u+1) = h(u) ∀u. Bằng quy nạp dễ thấy

g(u + n) = nh(u) + g(u)

Vậy

g(u) =

{h(u) + k(u) với 0 6 u < 1

nh(u) + k(u − n) với n 6 u < n + 1∀n ∈ Z (5)

trong đó, k(u), h(u) là các hàm tuỳ ý, h(u) tuần hoàn chu kỳ 1. Thay lạitheo biến số x > 0 ta được

f(x) = g(

log√2−1 x

)với x > 0

trong đó, g(u) được xác định theo (5).c) Với x < 0 ta đặt −(2 +

√2)x = t = (

√2 − 1)u ta có

f(x) = g(

log√2−1 |x|

)với x < 0

Tóm lại

f(x) =

{a tuỳ ý khi x = 0

g(

log√2−1 |x|

)khi x 6= 0

còn g(u) được xác định theo (5).Bài 6. Xét tập A gồm tất cả có bộ thứ tự (a1, a2, . . . , a1994, . . . , a1993+1994)

thoả mãn đồng thời các điều kiện sau:1) ai ∈ {0, 1} ∀i = 1, 1993 + 19942) Số 1 có mặt đúng 1994 lần trong mỗi bộ.

Page 46: Final Version 1989 to 2010 VN TST

2.4. Đáp án chọn đội tuyển năm học 1994 - 1995 45

Xét phân hoạch

A =⋃

A(n1,n2,...,n1994)

ở đây

+) Hợp lấy theo tất cả các bộ có thứ tự các số tự nhiên (n1, n2, . . . , n1994)thoả mãn n1 + 2n2 + · · · + 1994n1994 = 1994.

+) A(n1,n2,...,n1994) là tập gồm tất cả các bộ có thứ tự (a1, a2, . . . , a1993+1994) ∈A và thoả mãn điều kiện là trong mỗi bộ có đúng nk nhóm k ∀k = 1, 1994.(Nhóm k được định nghĩa là nhóm gồm đúng k số 1 đứng liên tiếp trong bộ,nói khác đi là nhóm có 1 trong các dạng sau (1 . . . 1︸ ︷︷ ︸

ksố1

0; 0 1 . . . 1︸ ︷︷ ︸ksố1

0; 0 1 . . . 1︸ ︷︷ ︸ksố1

)).

CardA = C19931993+1994

CardA(n1,n2,...,n1994) =1994!

n1!n2! . . . n1994!(1994 − n1 − · · · − n1994)!

=1994!

n1!n2! . . . n1994!(n2 + 2n3 + · · · + 1993n1994)!

CardA =∑

CardA(n1,n2 ,...,n1994).

Nên suy ra:

T =1

1994!C1993

1993+1994

.

2.4 Đáp án chọn đội tuyển năm học 1994 -

1995

Bài 1. Cho tam giác ABC với mỗi điểm M , gọi khoảng cách đại số từ Mđến đường thẳng BC là ± (khoảng cách thông thường từ M đến BC), lấydấu + hay − tuỳ theo M cùng phía hay khác phía với A đối với BC (tấtnhiên M thuộc BC thì khoảng cách đó bằng không). Tương tự cho khoảngcách đại số từ M đến CA,AB.

1) Xét các đường tròn (AB1C1), (AB2C2) như trong đề bài. Hãy chứngminh trục đẳng phương của cặp đường tròn đó là quỹ tích các điểm M màcác khoảng cách đại số từ M đến AB và đến CA tỉ lệ với γ vàβ.

Page 47: Final Version 1989 to 2010 VN TST

46 Chương 2. Đáp án tuyển sinh

Thực vậy, lấy hệ toạ độ vuông góc Oxy mà O ≡ A, B ∈ Ox+, (−→AB,

−→AC) =

ϕ, 0 < ϕ < 180◦. Khi đó−→ABc

= (1, 0),−→CAb

= (− cos ϕ, sinϕ). Gọi

B1 = (b1 cot ϕ, b1),

B2 = (b2 cot ϕ, b2),

C1 = (c1, 0),

C2(c2, 0)

, c1, c2 6= 0, c1 6= c2, b1, b2 6= 0, b1 6= b2

thì−−−→B1B2 = β

b

−→CA hay ((b2 − b1) cotϕ, b2 − b1) = β(− cosϕ,− sinϕ) suy

ra b2 − b1 = −β sinϕ.

Ta cũng có−−−→C1C2 = γ

c

−→AB tương đương với (c2 − c1, 0) = γ(1, 0) hay

c2 − c1 = γ.Đường tròn (AB1C1) đia qua A,C1 nên x2 + y2 − c1x − λ1y = 0, nó đi

qua B1 nên λ1 = b1−c1 sin ϕ cos ϕsin2 ϕ

, đường tròn (AB2C2): x2+y2−c2x−λ2y = 0,

λ2 = b2−c2 sinϕ cosϕβ

.

Trục đẳng phương hai đường tròn đó là

(c2 − c1)x + (λ2 − λ1)y = 0

⇔ γx − β + γ cosϕ

sinϕy = 0

hayy

γ=

x sinϕ − y cosϕ

β

Để ý rằng y là khoảng cách đại số từ M(x, y) đến AB còn x sinϕ − y cosϕlà khoảng cách đại số từ M(x, y) đến CA, ta suy ra điều phải chứng minh.

2) Với mỗi điểm M , kí hiệu X,Y,Z là khoảng cách đại số từ M đếnBC,CA,AB thì dễ thấy aX + bY + cZ = 2S, (S là diện tích tam giácABC) và ngược lại (X,Y,Z) mà aX + bY + cZ = 2S xác định một điểmM duy nhất có các khoảng cách đại số nói trên là X,Y,Z.

Theo phần 1), phương trình dA là Yβ

= Zγ, của dB là X

α= Z

γ, của dC

là Zα

= Yβ. Điểm chung của dA, dB, dC (nếu có) là điểm M(X,Y,Z) mà

Page 48: Final Version 1989 to 2010 VN TST

2.4. Đáp án chọn đội tuyển năm học 1994 - 1995 47

(X,Y,Z) là nghiệm của hệ

{aX + bY + cZ = 2SXα

= Yβ

= Zγ

hayX

α=

Y

β=

Z

γ=

2S

aα + bβ + cγ

hệ đó có nghiệm (và chỉ có một nghiệm) khi và chỉ khi aα + bβ + cγ 6= 0.Chú ý: Nếu (AB1C1), (AB2C2) cắt nhau tại A′ 6= A, có thể chứng minh

phần 1) (trong trường hợp này) nhờ phép đồng dạng thuận tâm A′, biếnB1 thành C1, biến B2 thành C2 và để hoàn thiện 1) còn cần xét (AB1C1),(AB2C2) tiếp xúc nhau.

Bài 2. Gọi A là tập các giá trị n (n ≥ 3) để đa thức Pn(x) khả quy. Vớin ∈ A ta có

Pn(x) = f(x).g(x) (*)

trong đó

f(x) = amxm + · · · + a1x + a0

g(x) = bsxs + · · · + b1x + b0

Pn(x) = xn+1 + kxn − 870x2 + 1945x + 1995

m ≥ 1, s ≥ 1, m + s = n + 1

Ta sẽ chứng minh m = 1 hoặc s = 1.Giả sử có m ≥ 2 và s ≥ 2 suy ra m < n và s < n. Vì a0b0 = 1995 chia

hết cho 5 và không chia hết cho 25 nên chỉ có một trong hai số a0, b0 là chia

hết cho 5, chẳng hạn a0...5 và b0 không chia hết cho 5. Trong dãy a0, a1, ..., am

gọi r là chỉ số nhỏ nhất để ar không chia hết cho 5, (1 ≤ r ≤ m < n, chú ýtồn tại r vì am bằng ±1). Từ (??) suy ra cr = arb0 + ar−1b1 + · · ·+ a0br. Docách chọn r thì a0, a1, ..., ar−1 đều chia hết cho 5, cr là hệ số của xr trong

Pn(x) cũng chia hết cho 5, suy ra arb0...5, điều này mâu thuẫn với ar, b0 đều

không chia hết cho 5. Vậy giả sử m ≥ 2 và s ≥ 2 là sai, suy ra hoặc m = 1hoặc s = 1, lúc đó Pn(x) có nghiệm nguyên với ∀n ∈ A. Xét các trường hợpsau

a) Nếu |xn| ≥ 2 ∀n ∈ A. Khi đó từ

xn(x + k) = 870x2 − 1945x − 1995

ta có

|xn + k| =|870x2 − 1945x − 1995|

|xn|n

Page 49: Final Version 1989 to 2010 VN TST

48 Chương 2. Đáp án tuyển sinh

Vì A vô hạn nên với n ∈ A đủ lớn thì |xn + k| < 1 suy ra xn + k = 0 suy ra

870x2n − 1945xn − 1945 = 0 hay 174x2

n − 389xn − 399 = 0. Vì 399...3 và 197

...3

nên xn...3. Đặt xn = 3y, ta có 522y2 − 389y − 133 = 0 suy ra k = −3.

b) Nếu |xn| < 2, ∀n ∈ A thì xn chỉ có thể là +1,−1.Với xn = 1 thì Pn(1) = 0 suy ra k = −3071.Với xn = −1 thì Pn(−1) = 0.Từ đó với n chẵn thì k = 821, còn với n lẻ thì k = −819.Thử lại, thấy nếu k = −3, k = −3071 thì Pn(x) khả quy ∀n ≥ 3. Nếu

k = 821 thì Pn(x) khả quy với ∀n chẵn. Nếu k = −819 thì Pn(x) khả quyvới ∀n lẻ.

Bài 3. Có a3 + b3 ≥ 2(ab)3/2 suy ra (a3 + b3)n ≥ 2n(ab)3n/2 ≥ 4(ab)3n/2vì n ≥ 2. Vì vậy, từ

(a3 + b3)n = 4(ab)1995

ta được 3n ≤ 3990. Đặt (a, b) = d ta có a = da1, b = db1 và (a1, b1) = 1.Khi đó, từ (??) có d3n(a3

1 + b31)

n = 4d2990(a1b1)1995 hay

(a31 + b3

1)n = 4d3990−3n(a1b1)

1995.

Suy ra (a31 + b3

1)n...(a1b1)

1995 suy ra (a31 + b3

1)n...(a1b1)

n (do n < 1995 vì 3n ≤3990) suy ra a3

1 + b31

...a1b1. Do vậy

a31

...b1

b31

...a1

a1...b1

b1...a1

(do (a1, b1) = 1) nên a1 = b1 = 1, lại do (a1, b1) = 1) suy ra a = b = d.Khi đó từ (??) có 2n−2 = d3990−3n. Vì d > 1 nên suy ra d có dạng 2k

với k ≥ 1, và do đó n − 2 = k(3990 − 3n) hay n = 3990k+23k+1

, và do đó

n = 1330 − 13283k+1

. Vì n ∈ N∗ nên 1328...3k + 1. Do 1328 = 24.83 và

2i ≡{

1 (mod 3) nếu i chẵn

2 (mod 3) nếu i lẻ, 2i.83 ≡

{1 (mod 3) nếu i chẵn

2 (mod 3) nếu i lẻ

nên suy ra 3k + 1 ∈ {22, 24, 2 × 83, 23 × 83}.Với 3k + 1 = 4 có k = 1 suy ra a = b = 2 và n = 998.Với 3k + 1 = 16 có k = 5 suy ra a = b = 25 và n = 1247.Với 3k + 1 = 166 có k = 55 suy ra a = b = 255 và n = 1322.Với 3k + 1 = 664 có k = 221 suy ra a = b = 2221 và n = 1328.

Page 50: Final Version 1989 to 2010 VN TST

2.4. Đáp án chọn đội tuyển năm học 1994 - 1995 49

Bài 4. Xét graph G có tập đỉnh là tập gồm n điểm đã cho và tập cạnhlà tập gồm 1

2(n2 − 3n + 4) đoạn thẳng đã cho. Từ giả thiết của bài toán ta

thấy trong G tồn tại một cạnh mà sau khi bỏ nó đi thì được G′ không liênthông. Giả sử a và b là hai đỉnh không liên thông với nhau trong G′.

Gọi Va và Vb lần lượt là tập gồm tất cả các đỉnh của G′ mà liên thôngvới a và b. Giả sử |Va| = n1 và |Vb| = n2.

Dễ thấy, G′ có 12(n2 − 3n + 2) cạnh; n1 ≥ 1, n2 ≥ 1, n1 + n2 ≤ n và

1

2(n2−3n+2) ≤ 1

2n1(n1−1)+

1

2n2(n2−1)+

1

2(n−n1−n2)(n−n1−n2−1)

hay (n1 − 1)(1 − n2) + (n − n1 − n2)(1 − n1 − n2) ≥ 0. Do đó

{(n1 − 1)(1 − n2) = 0

(n − n1 − n2)(n1 + n2 − 1) = 0

Vậy n1 = n − 1, n2 = 1 hoặc n2 = n − 1 và n1 = 1.Từ đó suy ra G′ có một đỉnh cô lập và (n−1) đỉnh mà bậc của mỗi đỉnh

bằng n− 2. Do đó G có một đỉnh bậc 1, (n− 2) đỉnh mà bậc của mỗi đỉnhbằng n− 2 và một đỉnh có bậc bằng n− 1. Bởi thế chu trình đơn có độ dàilớn nhất trong G là chu trình đơn độ dài n− 1 nếu n ≥ 4, 0 nếu n = 2hoặcn = 3.

Vậy

kmax =

{n − 1 nếu n ≥ 4

0 nếu n = 2, n = 3

Bài 5. Giả sử n + 1 = 2f(n)(1 + 2α), p + 1 = 2f(p)(1 + 2β) với α, β ≥ 0.Cặp số (n, p) là cặp số đẹp khi và chỉ khi 2f(n) > p hay

2f(n) ≥ p + 1 (1)

Từ đó ta có n + 1 = 2f(n)(1 + 2α) ≥ p + 1 suy ra

n ≥ p (2)

Từ (1) ta có 2f(n) ≥ p + 1 = 2f(p)(1 + 2β) suy ra 2f(n) ≥ 2f(p), thành thửf(n) ≥ f(p). Từ đó

(n + 1)...2f(p) (3)

Ta cần tìm bộ ba số (n, p, q) sao cho ba cặp số (n, p) (p, q) và (n+p+q, n)đều là các cặp số đẹp.

Giả sử n + p + q + 1 = 2f(n+p+q)(1 + 2γ).

Page 51: Final Version 1989 to 2010 VN TST

50 Chương 2. Đáp án tuyển sinh

Theo (2) vì (n, p) và (p, q) là cặp số đẹp nên n + p + q + 1 ≤ 3n + 1. Vì(n+ p+ q, n) là cặp số đẹp nên 2f(n+p+q) ≥ n+1 theo (1). Kết hợp các điềukiện trên có

2f(n+p+q)(1+2γ) = n+p+q+1 ≤ 3n+1 ≤ 3(2f(n+p+q)−1)+1 < 3.2f(n+p+q)

suy ra 1 + 2γ < 3, và do đó 1 + 2γ = 1 hay

n + p + q + 1 = 2f(n+p+q) (4)

Mặt khác, 2f(n+p+q) ≥ n + 1 = 2f(n)(1 + 2α) suy ra

f(n + p + q) ≥ f(n) (5)

Từ 2f(n+p+q) = n + p + q + 1 = (n + 1) + (p + 1) + (q − 1) theo (3) và (5)

ta có (n + 1)...2f(p) và (p + 1)

...2f(p) suy ra (q − 1)...2f(p), nhưng từ cặp số đẹp

(p, q) có 2f(p) > q nên chỉ xảy ra hai trường hợp hoặc q = 0 và f(p) = 0,hoặc q = 1 và f(p) > 0.

Xét q = 0 và f(p) = 0, từ (4) có n + p + q + 1 = n + p + 1 = 2f(n+p+q),đồng thời n + p + 1 = (n + 1) + p = 2f(n)(1 + 2α) + p.

Từ (5) và 2f(n)(1 + 2α) + p suy ra p...2f(n) mà 2f(n) > p nên p = 0. Từ

cặp số đẹp (n + p + q, n) = (n, n) suy ra n + 1 ≥ 2f(n) ≥ n + 1 suy ran − 1 = 2f(n) = 2m. Tử lại, ta thấy bộ ba số (n, p, q) = (2m − 1, 0, 0) thỏamãn với m ∈ Z và m ≥ 0.

Xét q = 1 và f(p) > 0. Từ (4) ta có 2f(n+p+q) = n+p+ q+1 = (n+1)+(p+1) = 2f(n)(1+2α)+2f(p)(1+2β). Chú ý rằng f(n+p+q) ≥ f(n) ≥ f(p)

suy ra 2f(p)...2f(n) nên f(p) = f(m).Từ 2f(p) = 2f(n) ≥ p + 1 = 2f(p)(1 + 2β) suy ra 1 + 2β = 1 suy ra

p+1 = 2f(p). Ta có 2f(n+p+q) = n+p+q+1 = (n+1)(+(p+a) = n+1+2f(n),suy ra n + 1 = 2f(n+p+q) − 2f(n) = 2k − 2m. Thử lại ta thấy, bộ ba số(n, p, q) = (2k − 2m − 1, 2m − 1, 1) thỏa mãn với k,m ∈ Z và k > m ≥ 1.

Bài 6. 1) Ta tính đạo hàm

f ′(x) =2x(x3 − 3x + 3)

3(x2 − 1)2.

Dễ dàng chứng minh được x3 − 3x + 3 > 0 ∀x > 1. Từ đó suy ra f ′(x) >0 ∀x > 1 và do đó hàm f(x) đồng biến trên (1,+∞). Hơn nữa lại cólim

x→1+f(x) = −∞ và lim

x→+∞f(x) = +∞. Nếu suy ra f(x) với tập xác định

(1,+∞) sẽ có tập giá trị là (−∞,+∞).Từ các kết quả ở trên, theo định lí về hàm ngược, ta suy ra tồn tại hàm

g(x) liên tục trên R, có tập giá trị là (1,+∞), và f(g(x)) = x, ∀x ∈ R.

Page 52: Final Version 1989 to 2010 VN TST

2.5. Đáp án chọn đội tuyển năm học 1995 - 1996 51

tiếp theo, ta sẽ chứng minh g(x) > x, ∀x ∈ R. Thật vậy, với x ≤ 1 thìg(x) > 1 ≥ x. Với x > 1 thì do x − f(x) = x3−3x+3

3(x2−1)nên x > f(x) ∀x > 1,

hay g(x) > x, (do tính đồng biến của f(x) trên (1,+∞).2) Kí hiệu gn(x) = g(g(...g(x))...). Ta sẽ tìm a dưới dạng a = gn(x0) với

x0 ∈ R. Khi đó

a0 = gn(x0)

a1 = f(x0) = gn−1(x0) > 1

a2 = f(a1) = gn−2(x0) > 1

· · · · · · · · ·an−1 = g(x0) > 1

an = x0

Với x0 6= ±1 thì an+1f(x0). Do g(x) > x, ∀x nên a0 > a1 > a2 > · · · an. Suyra nếu chọn x0 sao cho x0 = ±1 và f(x0) = gn(x0) thì dãy {an} sẽ là dãytuần hoàn với chu kỳ dương nhỏ nhất bằng n + 1.

Bây giờ ta sẽ chứng minh tồn tại x0 thỏa mãn các điều kiện nói trên. Thậtvậy, xét hàm h(x) = f(x)− gn(x) trên (−1, 0]. Ta có h(0) = f(0)− gn(0) =1 − gn(0) < 0, và limx→−1+ h(x) = +∞ (do gn(−1) là số xác định). Hơnnữa, do h(x) liên tục trên (−1, 0) của phương trình h(x) = 0, ta có a > 1và dãy {an} là dãy tuần hoàn với chu kỳ dương bé nhất bằng 1995.

2.5 Đáp án chọn đội tuyển năm học 1995 -

1996

Bài 1. Gọi 3n điểm đã cho là A1, A2, ..., A3n. Hiển nhiên trong mặt phẳngchứa 3n điểm đó, ta có thể dựng được đường thẳng ∆ sao cho Ai /∈ ∆,i = 1, 3n, A1, A2, ..., A3n nằm về cùng một phía của ∆; và ∆ không songsong với AiAj (∀inot = j ∈ {1, 2, ..., 3n}).

Kí hiệu dAi là khoảng cách từ điểm Ai đến ∆. Khi đó dAi 6= dAj (∀i 6=j ∈ {1, 2, ..., 3n}). Không mất tính tổng quát, giả sử

dA1 < dA2 < · · · < dA3n (1)

Qua mỗi điểm A3i+1, i = 0, ..., n− 1, kẻ đường thẳng ∆i ‖ ∆ dễ dàng suy ran tam giác A3j+1A3j+2A3j+3, i = 0, ..., n− 1 đôi một rời nhau và mỗi điểmAi, i = 1, 3n) là đỉnh có đúng một tam giác trong số n tam giác đó.

Bây giờ ta sẽ chứng minh tổng S diện tích của n tam giác nói trên thoảmãn S < 1

2. Thật vậy, xét 4A3i+1A3i+2A3i+3, i ∈ {0, 1, ..., n− 1}) và gọi Si

là diện tích của nó. Dễ thấy có thể dựng được hai đường thẳng a, b cùngvuông góc với ∆ và sao cho

Page 53: Final Version 1989 to 2010 VN TST

52 Chương 2. Đáp án tuyển sinh

1) a đi qua đúng một trong ba điểm A3i+1, A3i+2, A3i+3 còn b đi qua ítnhất một trong hai điểm còn lại.

2) cả ba điểm A3i+1, A3i+2, A3i+3 cùng nằm trong dải phẳng (kể cả haibiên) bị giới hạn bởi a và b.

Thế thì nếu gọi {A} = a∩∆i, {B} = a∩∆i+1, {C} = b ∩∆i+2, {D} =b ∩ ∆i ta sẽ có hình chữ nhật ABCD

chứa toàn bộ ∆A3i+1A3i+2A3i+3. Từ đó Si < 12SABCD = 1

2AD.CD < 1

2di

với di là khoảng cách giữa hai đường thẳng ∆i và ∆i+1. Từ đó suy ra

S =

n−1∑

i=0

Si < 12

n−1∑

i=0

di ≤ 12A1A3n ≤ 1

2.

(vì A1A3n ≤ 1). Bài toán được chứng minh.

Bài 2. Từ an =∑[n

2 ]i=0

(2i+1

n

)3i suy ra công thức tổng quát

an =(1 +

√3)n − (1 −

√3)n

2√

3.

Xét n chẵn và n lẻ.1) Với n = 2k, ta có an = 2kuk với

uk =(2 +

√3)k − (2 −

√3)k

2√

3(1)

Dãy {uk} thoả mãnuk+2 = 4uk+1 (2)

với u1 = 1, u2 = 4

Gọi g(k) là số l ∈ N lớn nhất để uk...2l.

Từ an = 2kun suy raf(2k) = k + g(h) (3)

Từ (2) thấy k lẻ, suy ra uk lẻ. Vậy k lẻ thì

g(k) = 0 (4)

nghĩa làf(2k) = k nếu k lẻ (4′)

Xét k = 2m chẵn thì

u2m = um.dm với dm = (2 +√

3)m + (2 −√

3)m (5)

Page 54: Final Version 1989 to 2010 VN TST

2.5. Đáp án chọn đội tuyển năm học 1995 - 1996 53

Dãy {dm} thoả mãn

dm+2 = 4dm+1 − dm với d1 = 4, d2 = 14 (6)

Từ (6) và vì dm chẵn với mọi m nên suy ra dm+2 ≡ −dm (mod 8).

Nếu m lẻ, thì dm ≡ 4 (mod 8)

Nếu m chẵn, thì dm ≡ 6 (mod 8)(7)

Từ (7) và u2m = um.dm suy ra

g(2m) =

{g(m) + 2 nếu m lẻ

g(m) + 1 nếu m chẵn(8)

Với k = 2sh, h lẻ thì từ (8) và g(k) = 0 suy ra

g(2sh) = g(2s−1h) + 1 = · · · = g(2h) + s− 1 = g(h) + 2 + s− 1 = s+ 1 (9)

Từ (3), (9), và do (4′ ta có

f(2k) =

{k với k lẻ

k + s + 1 với k = 2sh, s ≥ 1, k lẻ(10)

2) Với n = 2k + 1. Ta thấy dãy {an} thoả mãn

an+2 = 2an+1 + 2an, với a1 = 1, a2 = 2. (11)

Ta chứng minhf(2k + 1) = k (12)

bằng quy nạp với k = 0, 1 dễ thấy đúng. Giả sử đúng với k. Từ (10) cóf(2k) ≥ k và theo giả thiết quy nạp f(2k + 1) = k nên

a2(k+1)+1 = 2a2(k+1) + 2a2k+1 = 2.2k+1.n + 2.2kN = 2k+1(2M + N)

với N lẻ theo quy nạp. Vậy f(2(k + 1) + 1) = k + 1 đúng với k + 1.3) Tìm mọi n để f(n) = 1996Nếu 2k + 1, theo f(2k + 1) = k thì f(n) = f(2k + 1) = k = 1996 suy ra

n = 3993.Nếu n = 2s+1h với h lẻ. Với s = 0 suy ra f(2k) = k lẻ không thoả mãn

với s ≥ 1 suy ra f(2s+1h) = 2sh + s + 1 = 1996 suy ra 2sh + s = 1995 suyra h lẻ và 1 ≤ s ≤ 9. Thử thấy s = 1, n = 3998 và s = 3, n = 3984 vớis = 5, 7, 9 không có nghiệm. Đáp số n = 3984, n = 3998, n = 3993.

Page 55: Final Version 1989 to 2010 VN TST

54 Chương 2. Đáp án tuyển sinh

Bài 3. Đặt

f(a, b, c) = (a + b + c)4 + a4 + b4 + c4 − 12abc(a + b + c) − 47(a4 + b4 + c4)

= (a + b + c)4 + 37(a4 + b4 + c4) − 12abc(a + b + c)

Vì f(a, b, c) = f(−a,−b,−c) nên chỉ cần xét f(a, b, c) tại a, b, c mà a+b+c ≥0. Khi đó chỉ có thể xảy ra một trong ba trường hợp sau

Nếu a, b, c ≥ 0. Lúc này ta có a+ b + c ≥ 3(abc)1/3 suy ra (a+ b + c)4 ≥27abc(a + b + c) suy ra f(a, b, c) ≥ 15abc(a + b + c) ≥ 0 và f(a, b, c) = 0 khivà chỉ khi a = b = c = 0.

Nếu có đúng một trong ba số a, b, c là số âm. Khi đó có f(a, b, c) > 0.

Nếu có đúng hai trong ba số a, b, c là số âm. Không giảm tính tổng quát,coi a, b,< 0 từ a + b + c ≥ 0 suy ra c ≥ −a(+b) > 0. Đặt a = −α, b = −βvới α, β > 0, thế thì c ≥ α + β và

f(a, b, c) = f(−α,−β, c)

= {c − (α + β)}4 + 37(c4 + α4 + β4) − 12αβc{c − (α + β)}.

Vì α, β > 0 nên α4 + β4 ≥ 2(α+β2

)4 và 0 < α < β ≤ (α+β2

)2. Do đó ta có

đánh giá

f(a, b, c) ≥ {c − (α + β)}4 + 37{c4 + 2(

α + β

2)4} − 12c(

α + β

2)2{c − (α + β)}

= (c − 2x)4 + 37(c4 + 2x4) − 12cx2(c − 2x)

trong đó x = α+β2

, x > 0.

Đặt c = tx, từ c ≥ 2x suy ra t ≥ 2. Vậy ta tiếp tục đánh giá f(a, b, c)

f(a, b, c) ≥ x4{(t − 2)4 + 37(t4 + 2) − 12t(t − 2)}

= x4{(t − 2)4 + (t4 + 2) − 12t(t − 2) − 47(t4 + 2)}

= x4{2(t − 1)4 + 16 − 47(t4 + 2)} = 2x4{(t− 1)4 − 2

7t4 + 52

7}

Xét g(t) = (t−1)4− 27t4+ 52

7trên [2,+∞). Ta có g′(t) = 4{(t−1)3− 2

7t3}. Dễ

thấy rằng với t ≥ 2 thì g′(t) > 0 hay t > 11−t0

với t0 =√

27, g′(t) = 0 khi và

chỉ khi t = 11−t0

, g′(t) < 0 khi và chỉ khi 2 ≤ t ≤ 11−t0

. Do đó g(t) ≥ g( 11−t0

)

với t ≥ 2.

Ta có

g(1

1 − t0) = 2

7

3

√2

7

( 1

1 − t0

)4

−27

( 1

1 − t0

)4

+527.

Page 56: Final Version 1989 to 2010 VN TST

2.5. Đáp án chọn đội tuyển năm học 1995 - 1996 55

Do t0 = 3

√27

nên

g( 1

1 − t0

)= 2

7

{26 − (1 − t0(

1

1 − t0)4

}= 2

7

{26 − 1

(1 − t0)3

}

= 27

26 − 1

(1 − 3

√27)3

.

Ta chứng minh rằng 26 > 1

(1− 3√

27)4

. Thật vậy, bất đẳng thức này tương

đương với mỗi bất đẳng thức sau

1 − 3

√2

7− 3

√1

26> 0

1 − 2

7− 1

26− 3

3

√1

91> 0

41

182>

13√

91

68921 > 66428.

Vậy nên g(t) ≥ g( 11−t0

) > 0 ∀t ≥ 2. Tóm lại f(a, b, c) ≥ 0 ∀a, b, c ∈ R và

f(0, 0, 0) = 0 nên mina,b,c∈R f(a, b, c) = 0

Bài 4. Chú ý rằng phép đối xứng qua đường phân giác AB của góc định

hương (−−→AH,

−−→AH ′) biến

−−→AH thành

−−→AH ′, còn phép đối xứng qua đường phân

giác ngoài AC của góc (−−→AH,

−−→AH ′) thì biến

−−→AH thành

−−→AH ′′ = −

−−→AH ′.

Hình 11) Giả sử SAB(M) = M1, SBC(M1) = M2, SCA(M2) = M ′. Đặt f =

SCA.SBC.SAB. Gọi H,K, I lần lượt là hình chiếu của A,B,C xuống cạnhđối diện. Gọi đường thẳng IK là ∆. Ta sẽ chỉ ra f : ∆ → ∆ và bảo tồnhướng của ∆.

Page 57: Final Version 1989 to 2010 VN TST

56 Chương 2. Đáp án tuyển sinh

a) Khi 4ABC không vuông thì H,K, I phân biệt, do đó AB,BC,CAlà phân giác ngoài của 4HKI. Khi có một góc tù, chẳng hạn ∠A > 90◦ thìBC là phân giác ngoài, còn CA,AB là phân giác trong 4HKI. The chú ýtrên, dễ thấy f : 4 → 4 và bảo tồn hướng của ∆.

Hình 2b) Khi C = H = K = 90◦ (C ≡ K) thì ∆ là đường thẳng CI.

c) Khi B = H = I = 90◦ (B ≡ I) thì ∆ là đường thẳng BK.

d) Khi A = K = I = 90◦ (A ≡ I ≡ K) thì ∆ là đường thẳng đối xứngcủa đường thẳng AH qua AB (hay qua AC) (Hình 1)

2) f | ∆ : ∆ → ∆ bảo tồn khoảng cách và hướng nên f | ∆ là phéptịnh tiến theo véctơ −→v nào đó (−→v ‖ ∆) (xem hình 2: M,N ∈ ∆, M ′ =

f(M), N ′ = f(N) thì−−−→MM ′ =

−−→NN ′ = −→v ) và hình 1: H ′ ∈ ∆, H ′′ = f(H ′)

thì−−−→H ′H ′′ = −→v ).Với M tuỳ ý, gọi J là hình chiếu vuông góc của M xuống ∆ thì f(J) là

hình chiếu vuông góc của f(M) xuống ∆ và−→JJ ′ = −→v . Do f đảo hướng nên

M và f(M) = M ′ nằm khác phía đối với ∆ nên MM ′ ≥ JJ ′ = |−→v |, và nếuM /∈ ∆ thì MM ′ > JJ ′. Vậy tập hợp các điểm M để MM ′ đạt giác trị bénhất là đường thẳng ∆.

3) a) Khi lấy liên tiếp phép đối xứng qua các trục là hoán vị vòng quanhba cạnh 4ABC được

SAB ◦ SCA ◦ SBC = SAB ◦ f ◦ SAB

SBC ◦ SAB ◦ SCA = SCA ◦ f ◦ SCA

đều có dạng Sx ◦ f ◦ Sx = g (Sx là phép đối xứng qua trục x).Đặt ∆′ = Sx(∆) thì với M0 ∈ ∆′ khoảng cách

ρ(M0, g(M0)) = ρ(M0, Sx ◦ f ◦ S−1x (M0))

= ρ(S−1x (M0), f(S−1

x (M0)) = |−→v | ≤ ρ(M,f(M))

= ρ(M,SxgSx(M)) = ρ(Sx(M), g(Sx(M)))

Vậy khoảng cách ngắn nhất đối với g = Sx ◦ f ◦ Sx cũng là |−→v |.

Page 58: Final Version 1989 to 2010 VN TST

2.5. Đáp án chọn đội tuyển năm học 1995 - 1996 57

b) Khi lấy liên tiếp phép đối xứng qua các trục theo thứ tự sau:

SAB.SBC, SCA = f−1

thì khoảng cách bé nhất của MM ′ cũng như đối với f đã xét trên. Lấy hoánvị vòng quanh các trục đối xứng ở trên ta được

SCA ◦ SAB ◦ SBC, SBC ◦ SCA ◦ SAB

đều có dạng Sx ◦ f−1 ◦ Sx = g′ nên theo a) khoảng cách ngắn nhất đối vớig′ cũng là −→v .

Chú ý: Có thể chứng minh 1), 2): f là phép đối xứng trượt, trục ∆ điqua trung điểm Mf(M) (M tuỳ ý) và xác định được ∆ (một cách hìnhhọc) đến kết quả như trên, véctơ trượt là −→v .

Có thể xét cụ thể các trường hợp tam giác ABC tương tự trên đây, tínhđược khoảng cách bé nhất nói trên là |−→v | = a cos A+b cos B+c cos C, chẳnghạn khi 4ABC có ba góc nhọn thì |−→v | kà chu vi tam giác HIK nên |−→v | cótính chất đối xứng đối với a, b, c và ∠A,∠B,∠C : |−→v | = IH + HK + KI.

Bài 5. Giả sử số học sinh được mời là 65 em. Ta đặt tương ứng mỗiem với một điểm trên mặt phẳng và hai em được đặt tương ứng với haiđiểm khác nhau. Với mỗi cặp, hai em chưa quen nhau ta nối hai điểm tươngứng với hai em đó bởi một đoạn thẳng. Khi đó ta được một Graph đơn, vôhương, có 65 đỉnh, bậc mỗi đỉnh không nhỏ hơn 56 và với hai đỉnh kề nhaubất kỳ luôn tồn tại ít nhất một điểm không kề với cả hai đỉnh ấy, có 65đỉnh và thoả mãn

1) Bậc của mỗi đỉnh không lớn hơn 82) Với hai đỉnh không kề nhau, tồn tại ít nhất một đỉnh kề với cả hai

đỉnh ấy.Xét G: xét đỉnh A bất kỳ của G và gọi A1, A2, ..., Ak (k ≤ 8) là tất cả

các đỉnh kề với A. Nếu k ≤ 7 thì sẽ có tối đa 72 = 49 đỉnh mà mỗi đỉnhkề với ít nhất một trong các đỉnh A1, A2, ..., Ak và không kề với A. Suy rasố đỉnh của G không vượt quá 49 + 7 + 9 = 57 < 65 trái với giả thiết. Vậyphải có k = 8, suy ra mỗi đỉnh của G có bậc bằng 8. Từ đây, kết hợp với2), ta được

Page 59: Final Version 1989 to 2010 VN TST

58 Chương 2. Đáp án tuyển sinh

i) Ai, Aj không kề nhau ∀i 6= j ∈ {1, 2, ..., 8}ii) Mỗi đỉnh Ai, (i = 1, 8), ngoài A ra sẽ kề với đúng bảy đỉnh khác và

nếu kí hiệu Ait t = 1, 7) là bảy đỉnh ấy thì {Ai1, ..., Ai7}∩ {Aj1, ..., Aj7} = ∅(∀i /∈ j ∈ {1, 2, ..., 8}).

Từ đó suy ra trong G không có chu trình đơn độ dài 3 cũng như chutrình đơn đội dài 4. Do vậy Ait và Ais không kề nhau (∀i = 1, 8), ∀t 6= s ∈{1, 2, ..., 7}, và do đó nếu Ait kề Ajs (i 6= j) thì Ait không kề Ajm m 6= s.Từ đó suy ra có tất cả 14

(38

)= 14.7.8 chu trình đơn độ dài 6 đi qua A.

Vì A là đỉnh bất kỳ của G nên số chu trình đơn độ dài 6 trong G là14.7.8.65

6= 49.8.65

3/∈ Z. Điều vô lý. Vây không tồn tại G và do đó không tồn

tại G thoả mãn đề bài.

Bài 6. Ta xét bài toán tổng quát với x0 > 0 cho trước bất kỳ.1) Với a = 0 hiển nhiên xi = 0, ∀i > 0 dãy hội tụ.2) Với a > 0 kí hiệu f(x) = a

1+x2 và g(x) = f(f(x)). Từ đó suy ra f(x)

là hàm giảm trên (0,+∞) nên g(x) là hàm tăng trên (0,∞).Xét dãy {x2n} (chỉ số chẵn). Đặt un = x2n thì un+1 = g(un). Do g(x)

tăng nên {un} là dãy đơn điệu (tăng hoặc giảm). Mặt khác, 0 < un < a,nên luôn luôn tồn tại limun = limn→∞ x2n = l. Lấy giới hạn khi n → ∞ có

g(l) = l (1)

a) Ta chứng minh 0 < a ≤ 2 thì dãy {xn} có giới hạn. Thật vậy, giả sửf(l) = v hay

a = v + vl2 (2)

Ta có f(f(l)) = f(v) tương đương với f(v) = g(l) = l hay

a = v + lv2 (3)

Trừ (2) cho (3) được (v − l)(vl − 1) = 0. Nếu v 6= l thì vl = 1, thay vào(2) được v + l = a suy ra v.l là nghiệm của phương trình x2 − ax + 1 = 0.Vì a ≤ 2 nên a = 2 suy ra v = l = 1 trái điều giả sử. Vậy, v = l, tức làf(l) = l. Từ đó limx→∞ x2n+1 = limx→∞ x2n = f(l) = l = limx2n.

Vậy limn→∞ xn tồn tại.b) Ta chứng minh với a > 2 thì dãy {xn} hội tụ khi và chỉ khi a = x3

2+x1.Thật vậy, giả sử a = x3

0 + x0 suy ra x0 = a1+x2

0= x1 suy ra a = x3

1 + x1 suy

ra x2 = x1,.v..v... Vậy {xn} là dãy hằng, suy ra tồn tại limn→∞ xn = x0.Ngược lại, giả sử tồn tại limn→∞ xn = k. Qua giới hạn ta được f(k) = k

hay a = k3 + k, suy ra k là nghiệm duy nhất của x3 + x − a. Ta có

g(x) − x =(1 + x2)2(a− x) − a2x

a2 + (1 + x2)2=

−(x2 + x − a)(x2 − ax + 1)

a2 + (1 + x2)2.

Page 60: Final Version 1989 to 2010 VN TST

2.6. Đáp án chọn đội tuyển năm học 1996 - 1997 59

Vì a > 2 nên phương trình x2 − ax + 1 = 0 có hai nghiệm phân biệt, α, β.Giả sử α < β, ta thấy α < k < β. Thật, vậy vì a > 2 suy ra k > 1 do đóa− ak2 < 0. Thay a = k3 + k vào ta được k3 + k − ak2 < 0 hay α < k < β.

Dấu của g(x) − x là dấu của −(x − k)(x − α)(x − β). Ta có bảng xétdấu của g(x) − x như sau

x α k βg(x) − x + 0 − 0 + 0 −

Xét dãy un = x2n như trên thì dãy {un} là đơn điệu. Vì limn→∞ un = k nên∃n0 để α < un0 < β. Nếu α < un0 < k thì un0+1 = g(un0) < un0 suy ra{un} là dãy giảm và un0 < k, do đó nó không hội tụ về k. Tương tự nếuk < un0 < β thì {un} là dãy tăng với un0 > k do đó nó không hội tụ về k.Vậy chỉ có thể un0 = k. Chú ý rằng nếu ∃i: xi = k thì suy ra xi−1 = k. Thậtvậy x0 = k = a

1+x2i−1

suy ra k(1 + x2i−1) = a = k2 + k suy ra x2

i−1 = k2 suy

ra xi−1 = k. Thành thử un0 = k, do đó un = k hay x0 = k. Vì a = k3 + knên a = x3

0 + x0. Điều phải chứng minh.

3) Với a < 0. Đặt yn = −xn. Khi đó

yn+1 − xn+1 =−a

1 + x2n

=|a|

1 + y2n

.

Vậy {xn} hội tụ khi và chỉ khi {yn} hội tụ. Vì yn > 0, ∀n ≥ 1 nên theotrên với |a| ≤ 2 thì {yn} hội tụ. Nếu |a| > 2 thì {yn} hội tụ khi và chỉ khi|a| = y3

1 + y1. Mà từ (2.5) ta có |a| = y1(1+ y20) = y1(1+x2

0) suy ra x0 = y1,thành ra |a| = x3

0 + x0.

Kết luận: Dãy {xn} hội tụ khi và chỉ khi |a| ≤ 2 hoặc |a| = x30 + x0 =

1997√

1996 khi x0 =√

1996.

2.6 Đáp án chọn đội tuyển năm học 1996 -

1997

Bài 1. Gọi O là tâm mặt cầu ngoại tiếp ABCD. Điều kiện xác định điểmP là

PA2+a21+b2+c2 = PB2+b2

1+c2+a2 = PC2+c21+a2+b2 = PD2+a2+b2

1+c21.

Page 61: Final Version 1989 to 2010 VN TST

60 Chương 2. Đáp án tuyển sinh

Tính

PA2 + a21 + b2 + c2 = PA2 + DA2 + BA2 + cA2

=−→PA2 +

−−→DA2 +

−→BA2 +

−→CA2

= (−→PO +

−→CA)2 + (

−−→DO +

−→OA)2 + (

−−→BO +

−→OA)2 + (

−→CO +

−→OA)2

=−→PO2 + 4

−→OA2 + 2

−→PO

−→OA − 2

−→OA2(

−−→OD +

−−→OB +

−→OC +

−→OA)+

2−→OA2 +

−−→DO2 +

−−→BO2 +

−→OC2

= PO2 + 9R2 + 2−→PO.

−→OA.

−−→OE

= PO2 + 9R2 − 2−→OA(

−−→OE +

−→OP ).

trong đó đặt−−→OE =

−→OA +

−→OA +

−→OC +

−−→OD (điểm E xác định). Tính tương

tự đối với PB2 + b21 + c2 + a2, PC2 + c2

1 + a2 + b2, PD2 + a21 + b2

2 + c22. thì

ta được

−→OA(

−−→OE +

−→OP ) =

−−→OB(

−−→OE +

−−→OD) =

−→OC(

−−→OE +

−→OP ) =

−−→OD(

−−→OE +

−→OP )

Ta có−→OA(

−−→OE +

−→OP ) =

−−→OB(

−−→OE +

−→OP hay (

−→OA−−−→

OB)(−−→OE +

−→OP ) = 0

hay−→BA(

−−→OE2 +

−→OP ) = 0. Từ đó ta có

(−−→OE +

−→OP ).

−→BA = 0

(−−→OE +

−→OP ).

−→CA = 0

(−−→OE +

−→OP ).

−−→DA = 0

hay là−−→OE2 +

−→OP =

−→0 hay

−→PO =

−−→OE. Vậy điểm P xác định duy nhất.

Từ PA2 = (−→PO2+

−→OA)2 =

−→PO2+

−→OA2+2

−→PO.

−→OA = PO2+R2+2

−→PO.

−→OA

Page 62: Final Version 1989 to 2010 VN TST

2.6. Đáp án chọn đội tuyển năm học 1996 - 1997 61

thì

PA2 + PB2 + PC2 + PD2 = 4PO2 + 4R2 + 2−→PO(

−→OA +

−−→OB +

−→OC +

−−→OD)

= 4PO2 + 4R2 + 2−→PO.

−−→OE

= 4PO2 + 4R2 + 2PO2

= 6PO2 + 4R2 ≥ 4R2.

Dấu đẳng thức xảy ra khi và chỉ khi PO2 = 0 hay P ≡ O. Khi đó ta có

a21 + b2 + c2 = b2

1 + c2 + a2 = c21 + a2 + b2 = a2

1 + b21 + c2

1

hay a1 = a, b1 = b, c1 = c, tức là tứ diện ABCD gần đều.

Bài 2. 1) Giả sử k là số sao cho có thể thiết lập được hệ thông đườngbay thoả mãn các điều kiện của đề bài. Khi đó, tổng số đường bay trực tiếpgiữa hai thành phố sẽ là 25×k

2. Suy ra k ≡ 0 (mod 2).

Xét một thành phố A bất kỳ. Theo giả thiết, từ A có đường bay trựctiếp đên k thành phố khác, gọi là A1, A2, ..., Ak. Mỗi thành phố Ai, i = 1, k,lại có đường bay trực tiếp đến k − 1 thành phố khác, (không kể A). Hơnnữa, ta lại có: Nếu từ B đến A không có đường bay trực tiếp thì từ B phảicó đường bay trực tiếp đến ít nhất một thành phố Ai. Từ những lập luậntrên suy ra, số thành phố chỉ có thể tối đa là 1+ k + k(k− 1) = k2 +1. Nhưvậy 25 ≤ k2 + 1. Kết hợp với k ≡ 0 (mod 2), suy ra k ≥ 6.

2) Với k = 6 ta sẽ chỉ ra cách thiết lập hệ thống đường bay thoả mãncác điều kiện của đề bài. Chia 25 thành phố thành năm nhóm, mỗi nhómgồm năm thành phố. Các thành phố của nhóm thứ i, i = 1, 5, ta kí hiệu bởiA

(i)1 , A

(i)2 , A

(i)3 , A

(i)4 , A

(i)5 . Với các thành phố trong cùng nhóm i, ta thiết lập

các đường bay A(i)1 A

(i)2 , A

(i)2 A

(i)3 , A

(i)3 A

(i)4 , A

(i)4 A

(i)5 , A

(i)5 A

(i)1 . Giữa các thành

phố thuộc hai nhóm i, j bất kỳ, i 6= j ∈ {1, 2, 3, 4, 5}, xây dựng các đường

bay sau A(i)1 A

(j)1 , A

(i)2 A

(j)4 , A

(i)3 A

(j)2 , A

(i)4 A

(j)5 , A

(i)5 A

(j)3 .

Page 63: Final Version 1989 to 2010 VN TST

62 Chương 2. Đáp án tuyển sinh

Bằng cách xây dựng các đường bay như trên, ta có: Từ thành phố Abất kỳ sẽ có đường bay trực tiếp đến đúng hai thành phố, trong cùng nhómvới A và có đường bay trực tiếp đến đúng bốn thành phố khác nhóm vớiA. Do vậy từ mỗi thành phố sẽ có đường bay trực tiếp đến đúng sáu thànhphố khác.

Hơn nữa, với A,B là hai thành phố bất kỳ mà giữa chúng không cóđường bay trực tiếp ta thấy:

- Nếu A,B thuộc cùng nhóm thì dễ thấy luôn tồn tại một thành phốtrong nhóm đó mà từ C có đường bay trực tiếp đến cả A và B.

- Nếu A,B không cùng nhóm thì qua hình vẽ trên dễ dàng kiểm trađược sự tồn tại của thành phố C mà từ C có đường bay trực tiếp đến cả Avà B.

3) Vậy kmin = 6.

Bài 3. Giả sử α là số thực sao cho tồn tại dãy vô hạn các số tự nhiên{an} thoả mãn các điều kiện của đề bài. Dễ thấy α = 0 là một số thực nhưvậy. Do đó, để tìm số α lớn nhất, dưới đây ta chỉ xét α > 0.

Từ giả thiết về dãy {an} suy ra ai + aj ≥ aαi+j, ∀i, j ∈ N∗. Dẫn tới

2a2i ≥ aα2i+1 ∀i ∈ N (1)

Nếu α 6= 1 thì (1) tương đương với mỗi bất đẳng thức sau

a1

1−α a2i ≥ (21

1−α .a2i+1)α ∀i ∈ N

a1

1−α a2i ≤ (21

1−α .a2i)1/α ∀i ∈ N

hay

a1

1−α a2n ≤ (21

1−α .a2n−1)1/α ∀i ∈ N∗

Suy ra 21

1−α .a2n ≤ (21

1−α a1)1/αn, ∀n ∈ N∗. Mà a2n > 19972n

nên 21

1−α .19972n<

(21

1−α .a1)1/αn, ∀n ∈ N∗, hay

ln 21

1−α + 2n ln 1997 <1

αnln(2

11−α .a1) ∀n ∈ N∗.

hay

ln 1997 <1

2n.αnln(2

11−α .a1) −

1

2nln 2

11−α ∀n ∈ N∗ (2)

Từ (2) suy ra α ≤ 12, vì nếu α > 1

2thì, cho n → ∞, từ (2) ta được

ln 1997 ≤ 0, vô lý.Nếu α = 1 thì (1) cho ta a2i+1 ≤ 2a2i, ∀i ∈ N, hay a2n ≤ 2a2n−1

. Suy raa2n ≤ 2na1, ∀n ∈ N∗. Mà a2n > 19972n

nên 19972n< 2n.a1, ∀n ∈ N∗, hay

2n ln 1997 < n ln 2 + ln a1, ∀n ∈ N∗. Bất đẳng thức này tương đương với

ln 1997 <n

2n. ln 2 +

ln a1

2n∀n ∈ N∗ (3)

Page 64: Final Version 1989 to 2010 VN TST

2.6. Đáp án chọn đội tuyển năm học 1996 - 1997 63

Cho n → ∞, từ (3) ta được ln 1997 ≤ 0 (vô lý). Tóm lại, ta phải có α ≤ 12.

2) Với α = 12

ta sẽ xây dựng dãy {an} thoả mãn các điều kiện của đề

bài. Xét dãy {an} được xác định bởi a1 = 2 × 1997, a2 = 4 × 19972 − 2 vàan+1 = 2 × 1997an − an−1 ∀n ≥ 2. Dựa vào phương trình đặc trưng và cácsố hạng ban đầu a1, a2 của {an} dễ dàng tìm được an = pn + qn và n ∈ N∗,trong đó p = 1997 +

√19972 − 1 và q = 1997 −

√19972 − 1. Suy ra an ∈ N,

n ∈ N∗ và

an = pn + qn ≥ 2(p + q

2

)n

= 2 × 1997n > 1997n ∀n ∈ N∗ (4)

Tiếp theo, ta sẽ chứng minh

un = a[n2 ]

+ a[n+12 ], ∀n ≥ 2 (5)

Vì a[n2 ] + a[n+1

2 ] = n nên (5) tương đương với

(a[n

2 ]−t + a[n+12 ]+t

) ...(a[n

2 ]+ a[n+1

2 ]

)∀n ≥ 2, t ∈ {0, 1, ...,

[n2

]− 1} (6)

Do vậy, để chứng minh (5), ta sẽ chứng minh (6).Với t = 0 thì (6) hiển nhiên đúng.Với t = 1, ta có: nếu n = 2k thì

a[n2 ]−1 + a[n+1

2 ]+1 = ak−1 + ak+1 = 2 × 1997ak...(ak + ak)

hay (a[n

2 ]−1 + a[n+12 ]+1

) ...(a[n

2 ]+ a[n+1

2 ]

)

Nếu n = 2k + 1 thì

a[n2 ]−1 + a[n+1

2 ]+1 = ak−1 + ak+2 = 2 × 1997ak − ak+1 + 2 × 1997ak+1 − ak

= (2 × 1997 − 1)(ak+1 + ak)...(ak+1 + ak)

Từ đây suy ra (6) đúng.Giả sử ta đã có (6) đến t mà 1 ≤ t ≤

[n2

]− 1. Ta sẽ chứng minh (6)

cũng đúng với t := t + 1. Thật vậy, ta có

a[n2 ]−(t+1) + a[n+1

2 ]+(t+1) = 2 × 1997a[ n2 ]−t − a[n

2 ]−t+1+

+ 2 × 1997a[ n+12 ]+t − a[n+1

2 ]+t−1

= 2 × 1997(a[n

2 ]−t + a[n+12 ]+t

)−(a[n

2 ]−(t−1) + a[n+12 ]+(t−1)

)

Page 65: Final Version 1989 to 2010 VN TST

64 Chương 2. Đáp án tuyển sinh

Theo giả thiết quy nạp thì

(a[n

2 ]−t + a[n+12 ]+t

)−(a[n

2 ]−(t−1) + a[n+12 ]+(t−1)

) ...(a[n

2 ] + a[n+12 ]

).

Theo nguyên lý quy nạp, (6) cũng đồng thời là (5), được chứng minh. Từđó, ta có

un = p[n2 ] + q[

n2 ] + p[

n+12 ] + q[

n+12 ]

≥ 2(pn/2 + qn/2) > 2(pn + qn)1/2 = 2a1/2n > a1/2

n .

Từ đây và (4) cho thấy dãy {an} thoả mãn các điều kiện của đề bài ứng vớiα = 1

2.

3) Vậy αmax = 12.

Bài 4. Tìm công thức tổng quát của f(n).Từ

f(n + 2) = 503f(n + 1) − 1996f(n) (*)

có phương trình đặc trưng x2 − 503x + 1996 = 0. Phương trình này có hainghiệm nguyên là x=4 và x2 = 499, từ đó f(n) = 4n + 499n.

Ta sẽ chứng minh rằng ước nguyên tố p = p(s) của f(2s) có dạngp(s) = 21+su + 1 với u ∈ N.

Vì (4, 499) = 1 nên nếu

p(s) | f(2s) = 42s

+ 4992s

(**)

với s ≥ 1 thì p(s) lẻ, suy ra (4, p(s)) = 1 và (499, p(s)) = 1.Từ (**) có thể viết 42s

+ 4992s= tp(s).

Giả sử p(s)−1 = 2mv với v lẻ, m ≤ s thì có thể áp dụng định lý Fermat

p(s) = p | (4002s−m

)p−1 − 1 = (4992s

)p−12m − 1 = (pt − 42s

)p−12m − 1

= p.k + (−42s

)p−12m − 1 = p.k + (−1)

p−12m

{(42s−m

)p−1 − 1}

+ (−1)p−12m − 1

mà p | (42s−m)p−1 − 1 nên (−1)

p−12m = 1 suy ra v = p−1

2m

...2, trái với điều giả

sử. Vậy m > s suy ra p(s) = 21+s.u + 1.Đặt

Sk =k∑

i=1

p(si) =k∑

i=1

(21+siui + 1) =k∑

i=1

21+siui + k.

Với s = min{si | i = 1, 2, ..., k} thì Sk = 21+sm + k, (s ≥ k). Từ đó

Sk...2t tương đương với (21+s,+k)

...2t hay k...2t (vì t ≤ k ≤ s < 1 + s, suy ra

2r | 21+s).

Page 66: Final Version 1989 to 2010 VN TST

2.6. Đáp án chọn đội tuyển năm học 1996 - 1997 65

Bài 5. 1) Điều kiện để phương trình

4n2x = log2(2n2x + 1) (1)

có nghĩa là x > − 12n2 . Với điều kiện đó ta có (1) tương đương với

24n2x = 2n2x + 1 (2)

Đặt 4n2x + 1 = t, từ phương trình (2) ta có phương trình (ẩn t)

2t = t + 1 (3)

Với 0 < t < 1 ta có 2t < 2t + (1 − t) = t + 1.Với t < 0, t > 1 ta có 2t > 2t + (1 − t) = t + 1.Với t = 0, t = 1 ta có 2t = t + 1.Như vậy, phương trình (3) có tất cả hai nghiệm là t = 0 và t = 1. Dẫn tới

phương trình (2), và cũng là (1) cũng có hai nghiệm xn = 0 và xn = − 14n2 .

2) Với xn = 0 ta có axn + bxn = a0 + b0 = 2 = 2 + 3xn ∀a, b > 0. Xétxn = − 1

4n2 . Khi đó các bất đẳng thức sau là tương đương

axn + bxn ≥ 2 + 3xn ∀n ∈ N∗.

1

2(a− 1

4n2 + b−1

4n2 ) ≥ 1 +3

2(− 1

4n2) ∀n ∈ N∗

(a− 1

4n2 + b−1

4n2

2

)−4n2

≤{

1 +3

2

(− 1

4n2

)}−4n2

.

Cho n → +∞, từ bất đẳng thức trên ta được√

ab ≤ e3/2. Ngược lại, với√ab ≤ e3/2, ta có

1

2(a− 1

4n2 + b−1

4n2 ) ≥ (√

ab)−1

4n2 ≥ e32(− 1

4n2 ≥ 1 +3

2(− 1

4n2)

hay axn + bxn ≥ 2 + 3xn.3) Vậy, tất cả các cặp số thực dương a, b cần tìm là a, b > 0 sao cho

ab ≤ e3.

Bài 6. Trước hết, ta chứng minh khẳng định sau:Khẳng định K. Cho n điểm phân biệt cùng nằm trên một đường thẳng.

Tô n điểm đói bởi hai màu xanh, đỏ sao cho có đúng k điểm được tô bởimàu xanh, và giữa hai điểm màu xanh liên tiếp (tính từ trái qua phải) có ítnhất p điểm được tô bởi màu đỏ (tính từ trái qua phải) có ít nhất p điểmđược tô bởi màu đỏ và ở bên phải điểm màu xanh cuối cùng có ít nhất pđiểm được tô bởi màu đỏ và ở bên phải điểm màu xanh cuối cùng có ít nhấtp điểm được tô bởi màu đó. Khi đó số cách tô màu khác nhau là

(k

n−kp

)

Page 67: Final Version 1989 to 2010 VN TST

66 Chương 2. Đáp án tuyển sinh

Chứng minh. Lần lượt từ trái trái qua phải, gọi các điểm là 1, 2, ..., n. Đặttương ứng mỗi cách tô màu với bộ k số nguyên dương (i1 < i2 < · · · < ik),trong đó i1, i2, ..., ik là các điểm được tô màu xanh. Dễ thấy, tương ứng nóitrên xác lập một song ánh từ tập gồm tất cả các cách tô màu tới tập

T = {(i1 < i2 < · · · < ik) | ij ∈ {1, 2, ..., n− p} ∀i = 1, k;

ij+1 − ij > p ∀i = 1, k − 1

Xét ánh xạ

f : T → T ′ = {(j1 < j2 < · · · < jk) | {1, 2, ..., n− kp} ∀t = 1, k}(i1 < i2 < · · · < ik) ∈ T 7→ (i1, i2 − p, ..., ik − (k − 1)p) ∈ T ′

Dễ chứng minh được f là song ánh từ T đến T ′. Từ đó, ta có điều phảichứng minh.

2) Trở lại bài toán. Lần lượt, theo chiều kim đồng hồ, gọi các điểm làA1, A2, ..., An. Gọi X là tập gồm tất cả các cách tô màu khác nhau. Xétphân hoạch

X = X ′ ∪ X ′′

trong đó X = {x ∈ X | trong x có điểm màu xanh thuộc{Ai, ..., Ap}},X ′′ = X\X ′. Hiển nhiên, với x ∈ X ′′ thì trong x không có điểm màu xanhnào thuộc tập {A1, A2, ..., Ap}. Do đó, theo khẳng định K ta có cardX ′′ =(

kn−kp

).

Xét X ′. Kí hiệu X ′i = {x ∈ X ′ | trong x có điểm Ai được tô màu xanh,

i = 1, p. Thế thì X ′i ∩ X ′

j = ∅ ∀i 6= j ∈ {1, 2, ..., p} và X =p⋃

i=1

X ′i.

Với mỗi i = 1, p, theo khẳng định K, ta có cardX ′i =

(k−1

n−1−p−(k−1)p

)=(

k−1n−kp−1

). Do đó card X ′ = p

(k−1

n−kp−1

).

Vậy cardX =(

kn−kp

)+ p(

k−1n−kp−1

)

2.7 Đáp án chọn đội tuyển năm học 1997 -

1998

Bài 1.

Bổ đề 1. Nếu P (x) ∈ R[x] thoả mãn

|P (x)| ≤ cx1998 với x ∈ R c là hằng số (1)

thì P (x) = ax1998, với a là hằng số.

Page 68: Final Version 1989 to 2010 VN TST

2.7. Đáp án chọn đội tuyển năm học 1997 - 1998 67

Nếu P (x) = a hằng số thì P (x) = a = 0. Giả sử deg p = n ≥ 1

P (x) = anxn + an−1x

n−1 + · · · + a1x + a0, an 6= 0.

Khi đó (1) tương đương với

|anxn + an−1x

n−1 + · · · + a1x + a0| ≤ cx1998 (2)

Ta sẽ chứng minh bằng phản chứng rằng n ≤ 1998. Thật vậy, nếu n > 1998thì từ (2) ta có

∣∣∣∣an +an−1

x+ · · · + a0

xn

∣∣∣∣ ≤c

xn−1998, (∀x > 0)

Cho n → ∞, suy ra an = 0, trái với giả thiết. Vậy n ≤ 1998.Ta chứng minh ak = 0 với k = 0, 1, ..., n − 1 bằng quy nạp. Với k = 1

thì từ (2) cho x = 0 suy ra a0 = 0 đúng với k = 1. Giả sử a0 = a1 = · · · =ak−1 = 0 và k < n. Thay vào (2) được

|anxn + · · · + akx

k| ≤ cx1998

hay|anx

n−k + · · · + ak+1x + ak| ≤ cx1998−k ∀x > 0

Cho x → ∞0, vì 1998−k ≥ n−k > 0 nên vế phải tiến tới 0, suy ra |ak| ≤ 0hay ak = 0, đúng. Vậy P (x) = anx

n, do đó |anxn| ≤ cx1998 với mọi n ≤ 1998

và an 6= 0. Dễ thấy n = 1998 vì nếu trái lại thì an = 0.Bây giờ ta chứng minh bài toán. Lấy c = 1, đặt q(x) = p1(x) ∈ R[x], ta

có với mọi ε > 0 thì

|q(x)− pε(x)| = |p1(x) − pε(x)|≤ |f(x) − p1(x)| + |(f(x) − pε(x)|≤ x1998 + εx1998

= (1 + ε)x1998.

Do đó theo bổ đềpε(x) − q(x) = aεx

1998 (3)

Lại cóεx1998 ≥ |f(x) − pε(x)| = |f(x) − q(x)− aεx

1998| (4)

Từ (4) ta có|h(x)− aεx

1998| ≤ εx1998, ∀x ∈ R (5)

Cho x = 0, suy ra h(0) = 0. Từ (5) suy ra ∀x1 6= 0, x2 6= 0

Page 69: Final Version 1989 to 2010 VN TST

68 Chương 2. Đáp án tuyển sinh

∣∣∣∣h(x1)

x19981

− aε

∣∣∣∣ ≤ ε,

∣∣∣∣h(x2)

x19982

− aε

∣∣∣∣ ≤ ε.

Thành thử,∣∣∣∣h(x1)

x19981

− h(x2)

x19982

∣∣∣∣ ≤ 2ε ∀x1, x2 6= 0, ∀ε > 0.

Cho ε → 0 thì h(x1)

x19981

= h(x2)

x19982

, tức là h(x)x1998 = c, c là hằng số, tức là

h(x) = cx1998 ∀x 6= 0 (6)

Vì h(0) = 0, suy ra h(x) = cx1998, ∀x ∈ R. Từ đó f(x) = q(x) + h(x) =q(x) + cx1998 là một đa thức vì q(x) là đa thức.

Bài 2. Hai đường tròn C và C′ tiếp xúc nhau tại A. Phép nghịch đảof tâm A, phương tích AI2 = (2R)2 biến đường tròn C thành đường thẳngd tiếp xúc với C tại I, biến đường tròn C′ thành đường thẳng d′ vuông gócvới AI và cắt đường thẳng AI tại J , với AO.AJ = 4R2, suy ra AJ = 4R,và do đó IJ = 2R. f biến mỗi đường tròn C〉 ∈ H thành đường tròn Γi,i = 1, 2, ..., n tiếp xúc với d và d′, do đó Γi có bán kính R = IJ

2. C∞ tiếp xúc

với C tại M1, suy ra f(M1) = P1 là tiếp điểm của Γ1 và d, P1 = (AM1) ∩ d.

C\ tiếp xúc với C tại Mn, suy ra f(Mn) = Pn là tiếp điểm của Γn

và d. Vì có n đường tròn C∞, ..., C\ nên có n đường tròn Γ1,Γ2, ...,Γn vớiΓi = f(C〉). Suy ra P1Pn = (n − 1)2R. Chú ý rằng P1Pn = P1I + IPn hoặcP1Pn = IP1 − IPn.

Bây giờ ta tính IP1. Đặt IAM1 = α (0 ≤ α < π2) thì IP1 = 2R tanα.

Sử dụng định lý hàm số Cosine trong tam giác OO′O1 cho ta

O′O21 = OO′2 + OO2

1 − 2OO′.OO1 cos(π − 2α).

Page 70: Final Version 1989 to 2010 VN TST

2.7. Đáp án chọn đội tuyển năm học 1997 - 1998 69

Từ đó ta có các đẳng thức tương đương sau

(R

2+

R

p1

)2

=R2

4+ R2

(1 − 1

p1

)2

+2.R

2.R(1 − 1

p1) cos 2α

(p1 + 2

2p1

)2

=1

4+(p1 − 1

p1

)2

+p1 − 1

p1cos 2α.

(p1 + 2)2 = p21 + 4(p1 − 1)2 + 4p1(p1 − 1) cos 2α

cos 2α =3 − p1

p1 − 1. (1)

Đặt tan α = t thì

cos 2α =1 − t2

1 + t2(2)

Từ (1) và (2) ta có

(3 − p1)(1 + t2) = (p1 − 1)(1 − t2)

suy ra tan2 α = p1−2. Vậy IP1 = 2R√

p1 − 2. Tương tự IPn = 2R√

pn − 2.Do IP1 ≥ 0 và IPn ≥ 0 nên p1 ≥ 2 và pn ≥ 2.

Điều kiện cần. Chú ý rằng P1Pn = IP1 − IPn. và IP1 = 2R√

p1 − 2,IPn = 2R

√pn − 2 suy ra điều kiện cần và đủ để có họ C1, C2, ..., Cn thuộc

H là

2R√

p1 − 2 + 2R√

pn − 2 = (n − 1)2R

hay √p1 − 2 +

√pn − 2 = n − 1 (2a)

khi I nằm trong [p1, pn].

|2R√

p1 − 2 − 2R√

pn − 2| = (n − 1)2R.

hay

|√

p1 − 2 −√

pn − 2| = n − 1 (2b)

khi I nằm ngoài (p1, pn).Bình phương hai vế của (2a) và (2b) ta được

±√

(p1 − 2)(pn − 2) = (n − 1)2 − (p1 − 2) − (pn − 2)

Lại bình phương hai vế và rút gọn ta được

(p1 − pn)2 = (n − 1)2(2(p1 + pn) − (n − 1)2 − 8) (3)

Page 71: Final Version 1989 to 2010 VN TST

70 Chương 2. Đáp án tuyển sinh

Điều kiện đủ. Giả sử ta có (3). Coi p1 = p là ẩn và pn là tham số thì (3)tương đương với

p2 − 2p((n − 1)2 + pn) + p2n − 2pn(n − 1)2 + (n − 1)2((n − 1)2 + 8) = 0.

Để phương trình này có nghiệm thì ∆ ≥ 0 hay

((n − 1)2 + pn)2 − (p2n − 2pn(n − 1)2 + (n − 1)4 + 8(n − 1)2) ≥ 0.

Giải bất phương trình này cho ta pn ≥ 2, vì n ≥ 3.Tương tự, nếu ta coi pn là ẩn thì suy ra p1 ≥ 2.Nếu có số pi ≥ 2, (i = 1, n) thì trên tia gốc I của đường thẳng d có điểm

Pi để IPi = 2√

pi − 2. Xét dãy đường tròn ξ1, ξ2, ..., ξn lần lượt tiếp xúcngoài với nhau và tiếp xúc với cả hai đường thẳng d, d′. Phép nghịch đảotâm A, phương tích 4R2 biến dãy đường tròn ξ1, ξ2, ...ξn thành dãy đườngtròn C∞, C∈, ..., C\ lần lượt tiếp xúc nhau và tiếp xúc với hai đường trònC, C′.

Bài 3. Bất đẳng thức cần chứng minh tương đương với mỗi bất đẳngthức sau

exp{ k∑

i=1

( 1

pi+

1

p2i

)}≥ lnm.

k∏

i=1

e

( 1

pi+

1

p2i

)

> lnm.

Để chứng minh bất đẳng thức trên ta sử dụng các bổ đề

Bổ đề 1. Chứng minh rằng nếu 0 < x ≤ 12

thì

ex+x2

>1

1 − x

Ta viết bất đẳng thức cần chứng minh dưới dạng (1 − x)ex+x2> 1. Xét

hàm số f(x) = (1 − x)ex+x2trên [0, 1

2]. Ta có f(0) = 1, và tính đạo hàm

f ′(x) = xex+x2(1 − 2x). Dễ thấy rằng f ′(x) > − với mọi 0 < x ≤ 1

2. Từ đó

ta cóf(x) > f(0) = 1.

Từ bổ đề 1 suy ra

e

1

pi+

1

p2i >

(1 − 1

pi

)−1

.

Page 72: Final Version 1989 to 2010 VN TST

2.7. Đáp án chọn đội tuyển năm học 1997 - 1998 71

hay là

k∏

i=1

e

1

pi

+1

p2i >

k∏

i=1

(1 − 1

pi

)−1

.

Bổ đề 2. Chứng minh bất đẳng thức, m,n, k ∈ N

k∏

i=1

(1 − 1

pi

)−1

>n∑

m=1

1

m.

Dễ thấy (1 − 1

pi

)−1

> 1 +1

pi+ · · · + 1

psi

, ∀s ∈ N.

Giả sử s ∈ N thoả mãn 2s ≤ n < 2s+1. Khi đó

A =k∏

i=1

(1 − 1

pi

)−1

>k∏

i=1

(1 +

1

pi+ · · · + 1

psi

)

=∑ 1

pα11 · · · pαk

k

,

trong đó 0 ≤ αi ≤ s. Tổng chạy trên tất cả các bộ (α1, ..., αs) với 0 ≤ αi ≤ s.Mặt khác với mỗi m ≤ n, ta có

m = pα11 pα2

2 · · · pαkk với αi ≥ 0

mà 2s+1 > n ≥ m > pαii ≥ 2αi . Do đó αi < s + 1 suy ra αi ≤ s ∀i.

Vậy 1m

là một số hạng của tổng A. Vậy

∑ 1

pα11 ...pαk

k

>n∑

m=1

1

m.

Bài toán được chứng minh nếu ta chứng minh được rằng

Bổ đề 3.n∑

m=1

1

m> lnn.

Thật vậy,

∫ m+1

m

dx

x<

∫ m+1

m

dx

i=

1

m∀m = 1, 2, ...

Page 73: Final Version 1989 to 2010 VN TST

72 Chương 2. Đáp án tuyển sinh

Suy ra

n∑

m=1

1

m>

n∑

m=1

∫ m+1

m

dx

x=

∫ n+1

1

dx

x= ln(1 + n) > lnn.

Bây giờ chứng minh

n∑

m=1

1

m> ln(n + 1).

limn→∞

(1 +

1

n

)n

= e và dãy(1 +

1

n

)n

tăng nên(1 + 1

n

)n

< e ∀n ≥ 1.

Suy ra n ln(1 +

1

n

)< 1 hay

1

n> ln

(1 +

1

n

), ∀n ≥ 1.

n∑

m=1

1

m= 1+

1

2+

1

3+ · · ·+ 1

n> ln 2+ln(1+

1

2)+ln(1+

1

3)+ · · ·+ln(1+

1

n)

= ln 2 + ln3

2+ ln

4

3+ · · · + ln

n + 1

n= ln

2.3.4.n(n + 1)

1.2.3...n= ln(n + 1).

Bài 4. Rõ ràng đa thức bậc 0, P (x) = 1 và đa thức bậc nhất không cótính chất trên. Xét đa thức

P (x) = xn + · · · + a1x + a0

với ai ∈ Z, n > 1. Ta chứng minh P (x) luôn có tính chất đã nêu.Ký hiệu

A = {p ∈ P : P > P (1) − P (0), P > −P (0)}

Khi đó |A| = ∞ với mọi p ∈ A, xét đa thức

Qo(x) = P (x) − P (0) − p

Vì limx→∞ Qp(x) = +∞, Qp(1) = P (1)−P (0)− p < 0 nên ∃xp ∈ R, xp > 1để Q(xp) = 0 hay

P (xp) = P (0) + p ∈ N∗, ∀p ∈ A.

Có hai trường hợp sau

Page 74: Final Version 1989 to 2010 VN TST

2.7. Đáp án chọn đội tuyển năm học 1997 - 1998 73

1) Nếu tồn tại vô số p ∈ A để xp /∈ Q thì bài toán được chứng minh. (Vìnếu p1 6= p2 ∈ A thì P (xp1) 6= P (xp2) do đó xp1 6= xp2).

2) Nếu chỉ có hữu hạn p ∈ A để xp /∈ Q thì tồn tại p0 để xp ∈ Q, ∀p ≥ p0,p ∈ A. Khi đó xp là nghiệm hữu tỉ của Qp(x) → xp ∈ Z và xp | Qp(0) = −p.Mà xp > 1 nên xp = p. Vậy ∀p ≥ p0, p ∈ A : Qp(p) = 0 hay P (p) = P (o)+phay P (x) = P (0) + x, mâu thuẫn vì deg P > 2.

Bài 5. Điều kiện cần. Giả sử d = 2x2 +2xy +3y2. Đặt N = 5+19981998

Vì N lẻ, N không chia hết cho 5 nên d lẻ và d không chia hết cho 5. Ta có

d = 2x(x + y) + 3y2. d lẻ, suy ra y lẻ. Nếu x chẵn thì 2x(x + y)...4. Nếu x lẻ

thì 2x(x + y)...4 .

Vậy luôn có d ≡ 3y2 ≡ 3 (mod 4). Lại có d = 2x2 + 2xy + 3y2 suy ra2d = (2x + y)2 + 5y2 do đó 2d ≡ (2x + y)2 (mod 5). Dẫn đến 2d ≡ {0,±1}(mod 5) suy ra 4d ≡ ±2 (mod 5) do đó d ≡ ±2 (mod 5).

Nếu {d ≡ 2 (mod 5)

d ≡ 3 (mod 4)hay

{d ≡ 7 (mod 5)

d ≡ 7 (mod 4)

thì d ≡ 7 (mod 20).Nếu {

d ≡ 3 (mod 5)

d ≡ 3 (mod 4)

thì d ≡ 3 (mod 20).Điều kiện đủ. Kí hiệu 1998999. Khi đó N = a2 + 5. Xét tập

A = {ax + y}, x = 0, 1, 2, ..., [√

d], y = 0, 1, 2, ..., [√

d]

Đặt q = [√

d], ta có

q ≤√

d < q + 1.

Vì |A| = (q + 1)2 > d nên ∃(x1, y1), (x2, y2) với (x1, y1) 6= (x2, y2) để

ax1 + y ≡ ax2 + y2 (mod d)

suy ra a(x1 −x2)+ y1 − y2 ≡ 0 (mod d). Đặt x0 = x1 − x2, y0 = y1 − y2 thì

ax0 + y0 ≡ 0 (mod d).

Do đó a2x20 − y2

0 = (ax0 + y0)(ax0 − y0) ≡ 0 (mod d). Mà a2 ≡ −5(mod d), suy ra, −5x2

0 − y20 ≡ 0 (mod d), hay

5x20 + y2

0

...d (1)

Page 75: Final Version 1989 to 2010 VN TST

74 Chương 2. Đáp án tuyển sinh

Lại có |x0| = |x1 − x2| ≤ q ≤√

d, |y0| = |y1 − y2| ≤ q ≤√

d. Do đóx2

0 ≤ d, y20 ≤ d. Vậy

5x20 + y2

0 ≤ 6d (2)

Từ (1) và (2) suy ra

5x20 + y2

0 = kd với k = 0, 1, 2, 3, 4, 5, 6.

Ta chứng minh rằng không xảy ra k = 0, 1, 4, 5, 6.Thậy vậy,Nếu 5x2

0 + y20 = 0 thì x0 = y0 = 0 suy ra x1 = x2 và y1 = y2, loại.

Nếu 5x20 + y2

0 = d thì y2o ≡ d (mod 4) dẫn đến y2

0 ≡ ±2 (mod 5), loại.Nếu 5x2

0 + y20 = 4d thì y2

0 ≡ 4d (mod 5), dẫn đến y20 ≡ −d (mod 5) và

y20 ≡ ±2 (mod 5), loại.

Nếu 5x20+y2

0 = 5d thì y0 = 5y1, do vậy 5x20+25y2

1 = 5d suy ra x20+5y2

1 = dhay x2

0 ≡ d ≡ ±2 (mod 5) loại.Nếu 5x2

0 + y20 = 6d thì y2

0 ≡ 6d (mod 5) suy ra y20 ≡ d (mod 5) hoặc

y20 ≡ ±2 (loại)

Vậy chỉ xảy ra [5x2

0 + y20 = 2d

5x20 + y2

0 = 3d

Nếu 5x20 + y2

0 = 2d suy ra x20 + y2

0 ≡ 2d (mod 4). Vì d ≡ 3 (mod 4) suyra x2

0 + y20 ≡ 2 (mod 4) nên x0, y0 phải cùng lẻ. Đặt y0 − x0 = 2y ta có

y0 = x0 + 2y suy ra

5x20 + y2

0 = 5x20 + (x0 + 2y)2 = 2d

→ 6x20 + 4x0y + 4y2 = 2d

→ 3x20 + 2x0y + 2y2 = d

→ d = 2y2 + 2yx0 + 3x20 biểu diễn được

Nếu 5x20 + y2

0 = 3d suy ra y20 − x2

0 ≡ 0 (mod 3) hay (y0 − x0)(y0 + x0)...3

Nếu y0 + x0...3 suy ra x0 + y0 = 3y hay

y0 = 3y − x0 → 5x20 + y2

0 = 5x20 + (3y − x0)

2 = 6x20 − 6yx0 + 9y2

0 = 3d

→ 2x20 − 2yx0 + 3y2

0 = d

→ 2x20 + 2(−y)x0 + 3(−y)2 = d biểu diễn được

Nếu y0 − x0 = 3y suy ra y0 = x0 + 3y khi đó ta có

5x20 + y2

0 = 5x20 + (x0 + 3y)2 = 6x2

0 + 6x0y + 9y2 = 3d

→ d = 2x20 + 2x0y + 3y2

Page 76: Final Version 1989 to 2010 VN TST

2.7. Đáp án chọn đội tuyển năm học 1997 - 1998 75

Điều phải chứng minh.

Bài 6. Chuyển bài toán sang ngôn ngữ Graph, trong đó mỗi người coilà một điểm trên mặt phẳng, còn quan hệ quen nhau coi là một cạnh (1đoạn thẳng với giả thiết rằng các đoạn thẳng này không cắt nhau trừ haiđiểm đâu mút), ta có graph G đơn, vô hướng với tập đỉnh gồm n điểm

P = {A1, A2, . . . , An} và bậc của đỉnh A bất kỳ là d(A) ≥[

n+23

].

Điều kiện "hai người bất kỳ quen nhau hoặc quen nhau gián tiếp" chứngtỏ Graph G là liên thông.

Trong G (hữu hạn) xét đường gấp khúc nhiều cạnh nhất P0, giả sử P0

có k đỉnh là P0 = {A1, A2, . . . , Ak} với AiAi+1 (i = 1, 2, . . . , k − 1) là cáccạnh (Ai kề với Ai+1).

Do điều kiện (3) thì k ≤ n − 1.Gọi N(A) là tập các đỉnh kề với đỉnh A. Ta có

N(A1) ⊂ {A2, . . . , Ak} và N(Ak) ⊂ {A1, . . . , Ak−1}

vì trái lại thì tồn tại đường gấp khúc khác có nhiều cạnh hơn P0.Giả sử N(Ai) = {Ai1, Ai2, . . . , Ais}, i ∈ {1, 2, . . . , n} ký hiệu

N(Ai)+ = {Ai1+1, Ai2+1, . . . , Ais+1}

N(Ai)− = {Ai1−1, Ai2−1, . . . , Ais−1}

Do k ≤ n − 1 nên tồn tại đỉnh B /∈ P0. Ta có N(B) ∩ N(Ak)+ = ∅

Thật vậy nếu ∃Aj ∈ N(B) ∩ N(Ak)+ thì tồn tại đường gấp khúc

(A1, . . . , Aj−1, Ak, Ak−1, . . . , Aj+1, Aj, B) có k + 1 cạnh, trái giả thiết đốivới P0. Lập luận tương tự có N(B) ∩ N(A1)

− = ∅. Ta cũng có N(A1)− ∩

N(Ak)+ 6= ∅ vì nếu trái lại thì

|N(B) ∪ N(A1)− ∪ N(Ak)

+| = |N(B)|+ |B(A1)−|+ |N(Ak)

+| ≥

≥ 3[n + 2

3

]> 3(n + 2

3− 1)= n − 1

Page 77: Final Version 1989 to 2010 VN TST

76 Chương 2. Đáp án tuyển sinh

suy ra số đỉnh của tập hợp này lớn hơn hoặc bằng n mà tập hợp đó khôngchứa đỉnh B. Mâu thuẫn.

Vậy ∃Ai ∈ N(A1)− ∩ N(Ak)

+

Khi đó tồn tại đường gấp khúc khép kín có k−1 đỉnh thuộc tập Pc\{Ai}là (A1, A2, . . . , Ai−1, Ak, Ak−1, . . . , Ai+1)

Tập còn lại chứa các đỉnh đôi một không kề nhau (không có đoạn thẳngnối chúng) vì nếu trái lại, chẳng hạn có B1, B2 /∈ P0\{Ai} mà B1 kề với B2

do tính liên thông tồn tại đường gấp khúc chứa B1, B2 và P0\{Ai} có nhiềucạnh hơn P0, mâu thuẫn.

2.8 Đáp án chọn đội tuyển năm học 2001 -

2002

Bài 1. Trước hết ta chứng minh H thuộc tia MB. Thật vậy, giả sử ngượclại H thuộc tia MC (xem hình 1). Khi đó gọi K là trung điểm của AB ta có

BK = KH = AB/2 = HM và KM//AC. Suy ra: M1 = K1 = (1800−B)/2

và M2 = C. Do đó:

1800 = M1 + M2 = (1800 − B)/2 + C

⇔ C = 900 + (B)/2

Điều này mâu thuẫn với giả thiết C là góc nhọn. Vì vậy ta có điều phảichứng minh.

Gọi A1 và H1 tương ứng là điểm đối xứng với A và H qua trung trựccủa BC. (Xem hình 2). Ta có:

AA1 = HH1 = 2HM = AB = A1C ⇒ A1CA = A1AC = ACB

⇒ ABC = BCA1 = 2ACB

⇒ A1AN = (BAC)/3 + A1AC = (1800 − 3ACB)/3 + ACB = 600

Page 78: Final Version 1989 to 2010 VN TST

2.8. Đáp án chọn đội tuyển năm học 2001 - 2002 77

Suy ra 4ANA1 là tam giác đều ⇒ NA = NA1 = AA1 = AB =

NP ⇒ N là tâm đường tròn ngoại tiếp 4APA1 ⇒ APA1 = (ANA1)/2 =

300 ⇒ (BAC)/3 = PAN = NPA = (APA1)/2 = 150 ⇒ BAC = 450 ⇒ACB = 450 và ABC = 900. Như vậy, 4ABC là tam giác vuông cân tại B.

Ngược lại, dễ thấy 4ABC vuông cân tại B thoả mãn tất cả các yêu cầucủa đề bài.

Tóm lại, tất cả các tam giác ABC phải tìm là các tam giác vuông cântại B.

Bài 2. Trước hết ta nhận xét rằng, với mỗi giá trị N0 cho trước thì hoặcngười A hoặc người B sẽ có chiến lược thắng cuộc.

Xét hàm số f : N∗ → {0, 1} xác định như sau:f(n) = 1 nếu người A có chiến lược thắng cuộc khi số ghi ở thời điểm

ban đầu là n.f(n) = 0 nếu người B có chiến lược thắng cuộc khi số ghi ở thời điểm

ban đầu là n.Dễ thấy: f(1) = f(2) = 1, f(3) = 0 và với mỗi n ≥ 3 ta có:+) Nếu f(n − 1) = 0 hoặc f([n/3]) = 0 thì f(n) = 1+) Nếu f(n − 1) = 1 và f([n/3]) = 1 thì f(n) = 0Suy ra: f(1) = f(2) = 1, f(3) = 0 và

f(n) = 1 − f(n − 1)f([n/3]) ∀n ≥ 3 (1)

Từ đó với mọi k ≥ 1 ta có:

f(3k) = 1 − f(3k − 1)f(k)

f(3k + 1) = 1 − f(3k)f(k) = 1 − (1 − f(3k − 1)f(k))f(k)

= 1 − f(k) + f(3k − 1)f(k)

Cộng vế theo vế hai đẳng thức trên ta được:

f(3k) + f(3k + 1) + f(k) = 2 ∀k ≥ 1 (2)

Hoàn toàn tương tự ta chứng minh được:

f(3k + 1) + f(3k + 2) + f(k) = 2 ∀k ≥ 1 (3)

Từ đó suy ra: f(3k + 2) = f(3k) ∀k ≥ 1 Hơn nữa, từ (1),(2) và (3) tacòn có: ∀k ≥ 1 nếu f(k) = 0 thì f(k + 1) = 1 và f(3k) = f(3k + 1) =f(3k + 2) = 1. Suy ra, nếu f(k) = 0 thì f(3k + 3) = 0. Từ đây, vì f(3) = 0nên f((3k − 3)/2 = 0 ∀k ≥ 2. Do đó:

f(120) = f((35 − 3)/2) = 0

f((32002 − 1)/2 = 1 − f((32002 − 3)/2)f([(32002 − 1)/6]) = 1

f((32002 + 1)/2) = f(2 + (32002 − 3)/2 = f((32002 − 3)/2) = 0

Page 79: Final Version 1989 to 2010 VN TST

78 Chương 2. Đáp án tuyển sinh

Vậy:Nếu N0 = 120 thì người B có chiến lược thắng cuộc.Nếu N0 = (32002 − 1)/2 thì người A có chiến lược thắng cuộc.Nếu N0 = (32002 + 1)/2 thì người B có chiến lược thắng cuộc.Bài 3. Dễ thấy, nếu gọi p là ước nguyên tố lớn hơn

√2m + 1 của m thì

p là duy nhất vàp > 3 (*)

Xét số nguyên dương M mà đối với nó tồn tại tập T = {x1 = m,x2; . . . ;xk =M} thoả mãn các điều kiện của đề bài. Ta sẽ chứng minh: M ≥ m + p.

Thật vậy, vì p >√

2m + 1 nên p có số mũ 1 trong phân tích chuẩn củam. Do đó từ giả thiết tích x1x2 . . . x + k là số chính phương suy ra trongcác số x2, . . . , xk phải có ít nhất 1 số là bội của p. Vì vậy M = xk ≥ m + p.

Với M = m + p, xét tập T = {m; (m/p)(p + 1); ((2m + p)/2p)(p −1); ((2m + p)/2p)(p + 1); ((m + p)/p)(p1);m + p}. Từ (*) dễ dàng chứngminh được:

+) Tất cả các số thuộc T đều là số nguyên dương.+) m < (m/p)(p + 1), ((2m + p/2p)(p − 1), ((2m + p)/2p)(p + 1), ((m +

p)/p)(p − 1) < m + p+) Tích tất cả các số thuộc T là số chính phương.Hơn nữa, nếu trong bốn số (m/p)(p + 1), ((2m + p)/2p)(p − 1), ((2m +

p)/2p)(p + 1), ((m + p)/p)(p − 1) có hai số trùng nhau thì khi bỏ hai số đóra khỏi tập T ta sẽ nhận được tập T1 có đầy đủ các tính chất nêu trên.

Các lập luận nêu trên chứng tỏ với M = m + p tồn tại tập T thoả mãntất cả các điều kiện của đề bài.

Vậy, số nguyên dương M nhỏ nhất cần tìm là m + p, trong đó p là ướcnguyên tố lớn hơn

√2m + 1 của m.

Bài 4. Xét số k bất kỳ thoả mãn điều kiện của đề bài. Với mỗi bộ(i1, i2, . . . , ik) mà

1 ≤ i1 < i2 < · · · < ik ≤ n (*)

Ký hiệu M(i1, . . . , ik) là số cột chỉ gồm các ô được đánh dấu của bảng k×2ntạo nên từ k hàng i1, i2, . . . , ik. Đặt:

M =∑

M(i1, . . . , ik) (1)

ở đây tổng lấy theo tất cả các bộ (i1, i2, . . . , ik) thoả mãn (*).Với mỗi i ∈ {1, 2, . . . , 2n, gọi mi là số hàng (của bảng n× 2n) có ô thứ i

được đánh dấu và gọi si là số bảng k × 2n có cột thứ i chỉ gồm các ô đượcđánh dấu. Ta có:

si = Cm ∀i ∈ {1, 2, . . . , 2n},2n∑

i=1

mi = n2

Page 80: Final Version 1989 to 2010 VN TST

2.8. Đáp án chọn đội tuyển năm học 2001 - 2002 79

M =

2n∑

i=1

si =

2n∑

i=1

Cm ≥2n∑

i=1

(mi−k+1) = n2−2kn+2n = n(n−2k+2) (2)

Từ (1) và (2) suy ra tồn tại (i1, i2, . . . , ik) sao cho:

M(i1, i2, . . . , ik) ≥ (n(n − 2k + 2))/Cn =k!(n − 2k + 2)

(n − k + 1)(n − k + 2) . . . (n − 1)

(Đpcm)Bài 5. Yêu cầu của bài ra tương đương với việc tìm tất cả các cặp đa

thức P (x),H(x) ∈ Z[x] sao cho:

(H(x))2 − (x2 + 6x + 10)(P (x))2 = −1 (1)

Nhận thấy, nếu cặp đa thức (P,H) thoả mãn (1) thì các cặp đa thức(−P,H), (P,−H), (−P,−H) cũng thoả mãn (1). Vì vậy ta chỉ cần xétcác cặp đa thức (P,H) với P,H thuộc tập hợp Z+[x] các đa thức có hệ sốcao nhất là số nguyên dương.

Đặt√

x2 + 6x + 10 = α. Từ tính bất khả quy của tam thức f(x) =x2 +6x+10 ta có thể dễ dàng chứng minh, bằng phương pháp phản chứng,rằng α không thể biểu diễn dưới dạng A(x)/B(x), trong đó A(x) và B(x)là các đa thức với hệ số thực. Từ đó suy ra với mỗi n ∈ N, tồn tại duy nhấtcặp đa thức M(x), T (x) ∈ Z[x] sao cho:

(x + 3 + α)2n+1 = M(x) + αT (x)

Tiếp theo ta có các nhận xét sau:Nhận xét 1: Nếu M(x), T (x) ∈ Z[x] và

(x + 3 + α)2n+1 = M(x) + αT (x), n ∈ N (2)

thì (M(x))2 − (x2 + 6x + 10)(T (x))2 = −1.Chứng minh. Với n ∈ N, từ tính duy nhất của biểu diễn (2) ta có

(x + 2 − α)2n+1 = M(x) − αT (x) (3)

Nhân (2) với (3) vế theo vế, ta được điều cần chứng minh.Nhận xét 2: Nếu đa thức hằng S(x) và đa thức G(x) ∈ Z+[x] thoả

mãn hệ thức

(G(x))2 − (x2 + 6x + 10)(S(x))2 = 1 ∀x ∈ R

thì S(x) = 0 và G(x) = 1.

Page 81: Final Version 1989 to 2010 VN TST

80 Chương 2. Đáp án tuyển sinh

Chứng minh. Dễ thấy phải có degG ≤ 1. Từ đây dễ dàng suy ra điềucần chứng minh.

Nhận xét 3: Nếu P (x),H(x) ∈ Z+[x] thoả mãn (1) thì tồn tại số tựnhiên n sao cho P (x) = T (x) và H(x) = M(x), trong đó M(x) và T (x)được xác định theo (2).

Chứng minh. Dễ thấy tồn tại số tự nhiên n sao cho

degT ≤ degP < degT ∗ (*)

trong đó T ∗(x) là đa thức được xác định theo (2) ứng với n + 1. Xét các đathức H∗(x) và P ∗(x) xác định bởi:

(H(x) + αP (x))(M(x) − αT (x)) = H∗(x) + αP ∗(x)

Ta có: (H(x))2 − (x2 + 6x + 10)(P ∗(x))2 = 1.Mặt khác, lại có:

P ∗(x) = P (x)M(x)−H(x)T (x) = [(T (x))2−(P (x))2]/[P (x)M(x)+H(x)T (x)](4)

Vì degT = 2n và degT ∗ = 2n+2 nên từ (*) ta thấy có thể xảy ra hai trườnghợp sau:

Trường hợp 1: degP = degT = 2n. Khi đó deg[(T (x))2 − (P (x))2] ≤ 4nvà deg[P (x)M(x) + H(x)T (x)] = 4n + 1. Vì vậy (4) tương đương với

(T (x))2 − (P (x))2 = 0 và P ∗(x) = 0.

Trường hợp 2: degP = degT + 1 = 2n + 1.deg[(T (x))2 − (P (x))2] =4n + 2 và deg[P (x)M(x) + H(x)T (x)] = 4n + 2. Vì thế từ (5.4) suy raP ∗(x) là đa thức hằng. Do đó, theo nhận xét 2, ta phải có P ∗(x) = 0 và(T (x))2 − (P (x))2 = 0. Suy ra degP = degT , trái với giả thiết ban đầu.Mâu thuẫn nhận được cho thấy trường hợp này không thể xẩy ra.

Nhận xét 3 được chứng minh.Từ ba nhận xét nêu trên suy ra tất cả các đa thức P (x) cần tìm theo

yêu cầu của đề ra:

P (x) = ±((x + 3 + α)2n+1 − (x + 3 − α)2n+1)/2α

Bài 6. Với mỗi tập hữu hạn T các số nguyên dương đặt:

D(T ) =∏

t −∑

t2

trong đó tích và tổng lấy theo tất cả các số t thuộc T .

Page 82: Final Version 1989 to 2010 VN TST

2.9. Đáp án chọn đội tuyển năm học 2003 - 2004 81

Xét tập A0 = {1, 2, 4, . . . , 22002}. Dễ thấy maxA0 <∏

a − 1 (tích lấytheo tất cả a ∈ A0) và D(A0) > 0. Xây dựng tập A1 = A0 ∪ {

∏a − 1}. Ta

có |A1| = |A0| + 1 và

D(A1) =∏

a−∑

a2 =∏

a(∏

a−1)−(∑

a2+(∏

a−1)2) = D(A0)−1 > 0

Trên cơ sở tập A1 ta xây dựng tập A2 theo các xây dựng A1 trên cơ sởA0. Quá trình xây dựng trên được tiếp tục một khi ta còn nhận được tậpAi có D(Ai) > 0. Vì sau mỗi lần xây dựng số phần tử của Ai tăng 1 vàD(Ai) giảm 1 nên tồn tại k sao cho |Ak| > 2002 và D(Ak) = 0. Giả sửAk = {a1 < a2 < · · · < am}. Khi đó am sẽ là nghiệm nguyên dương của tamthức với hệ số nguyên:

f(X) = X2 −(m−1∏

i=1

ai

)X +

m−1∑

i=1

a2

Từ đó suy ra biệt thức 4 của f phải là số chính phương và ta có điều phảichứng minh.

2.9 Đáp án chọn đội tuyển năm học 2003 -

2004

Bài 1. Xét tập hợp

S0 = {p1, p2, ..., p1001, p1002, p1.p1003, p2.p1004, ..., p1001.p2003, p1002.p2004}

trong đó p1, p2, ..., p2003, p2004 là 2004 số nguyên tố phân biệt lớn hơn 1.Dễ thấy tập S0 nêu trên có các tính chất như đề bài yêu cầu. Hơn nữa,

rõ ràng trong 1002-tập con {p1, p2, ..., p1001, p1002} của S0 không có hai sốnào mà ước chung lớn nhất của cúng khác 1. Suy ra k ≥ 1003.

Xét một tập S tuỳ ý thoả mãn các yêu cầu của đề bài. Với mỗi số s ∈ S,ký hiệu g(s) là số các số thuộc S không nguyên tố cùng nhau với s. Từ cácgiả thiết của bài toán suy ra g(s) ≥ 1 và g(s) = g(t) với mọi s, t ∈ S. Nghĩalà g(s) = m ∀s ∈ S, trong đó m là một hằng số nguyên dương.

Xét một 1003-tập con T tuỳ ý của S. Ta sẽ chứng minh rằng trong Ttồn tại hai số phân biệt mà ước số chung lớn nhất của chúng khác 1.

Thật vậy, giả sử ngược lại rằng trong T không có hai số nào mà ước sốchung lớn nhất của chúng khác 1. Khi đó, kí hiệu

A = {a ∈ S | ∃t ∈ T, (a, t) 6= 1}

Page 83: Final Version 1989 to 2010 VN TST

82 Chương 2. Đáp án tuyển sinh

ta có A ∩ T = ∅. Và vì thế

|A| ≤ 2004 − 1003 = 1001 (1)

Mặt khác, do số số thuộc S (kể cả lặp) mà mỗi số đều không nguyên tốcùng nhau với ít nhất một số thuộc T là 1003m và mỗi số bị tính lặp tối đam lần, nên

|A| ≥ 1003.m

m= 1003 (2)

Mâu thuẫn giữa (1) và (2) chứng tỏ điều giả sử ở trên là sai. Vì thế, ta cóđiều muốn chứng minh.

Tóm lại, từ các chứng minh trên ta được kmin = 1003.

Bài 2. i) Dễ thấy, nếu α = 0 thì các hàm số f(x) ≡ 0 và f(x) ≡ 1 làcác hàm số thoả mãn các điều kiện của đề bài. Vì thế α = 0 không phải làgiá trị cần tìm.

ii) Xét α 6= 0.Thay x = 0 vào hệ thức của đề bài

f(x2 + y + f(y)) = (f(x))2 + αy (1)

ta đượcf(x + f(y)) = (f(0))2 + αy ∀y ∈ R (2)

Từ đó suy ra f là toàn ánh từ R lên R. Do đó, tồn tại x0 sao cho f(x0) = 0.Thay y = 0 vào (1), ta được

f(x2 + f(0)) = (f(x))2 ∀y ∈ R (3)

Lần lượt thay x = x0 và x = −x0 vào (3), ta được

0 = f(x20 + f(0)) = (f(−x0))

2.

Từ đây suy ra f(−x0) = 0. Từ đó, lần lượt thay y = x0 và y = −x0 vào (2),ta được

(f(0))2 + αx0 = 0 và (f(0))2 + α(−x0) = 0

hay f(0) = 0 và x0 = 0. Như vậy,

f(x) = 0 ⇔ x = 0 (4)

Do đó, từ (2) ta cóf(y + f(y)) = αy ∀y ∈ R (5)

và từ (3) ta cóf(x2) = (f(x))2 ∀x ∈ R (6)

Page 84: Final Version 1989 to 2010 VN TST

2.9. Đáp án chọn đội tuyển năm học 2003 - 2004 83

Từ (1) và (6) suy ra

f(x + y + f(y)) = f(x) + αy ∀x ≥ 0, ∀y ∈ R (7)

Xét X ≥ 0 tuỳ ý. Trong (7), thay x = X, y = −f(X)α

, ta được

f(X − f(X)

α+ f(−f(X)

α

))= 0

Từ đó, theo (4) ta có

X − f(X)

α+ f(−f(X)

α

)= 0

hay

−X = −f(X)

α+ f(−f(X)

α

)(8)

Suy ra

f(−X) = f(−f(X)

α

)+f(−f(X)

α

))= α

(−f(X)

α

)= −f(X)

Từ đó, do X tuỳ ý và do (4), suy ra f(x) là hàm số lẻ trên R. Từ (8), doX ≥ 0 tuỳ ý, suy ra hàm số g(x) = x+ f(x), ∀x ∈ R, nhận mọi giá trị thựckhông dương. Mà, dễ thấy, g(x) là hàm số lẻ trên R nên suy ra g(x) nhậnmọi giá trị thực. Vì thế, hàm số g là một toàn ánh từ R lên R.

Xét x ≥ 0, y ∈ R tuỳ ý. vì g là một toàn ánh từ R lên R nên tồn tạiy0 ∈ R sao cho y0 + f(y0) = y. Do đó

f(x + y) = f(x + y0 + f(y0)) = f(x) + αy0

= f(x) + f(y0 + f(y0) = f(x) + f(y)

Suy raf(x + y) = f(x) + f(y), ∀x, y ≥ 0 (9)

Hơn nữa từ (6) và (4) suy ra f(x) > 0, ∀x > 0. Từ đó với x > y ≥ 0, tacó 0 < f(x − y) = f(x) − f(y), hay f(x) > f(y). Như vậy

f(x) là hàm số đồng biến trên R+ (10)

Từ (9) và (10) suy ra f(x) = ax ∀x ∈ R+ trong đó a là một hằng số thực.Từ đây, vì f là hàm số lẻ trên R nên f(x) = ax, ∀x ∈ R.

Ngược lại, thay f(x) = ax vào (1) ta được

a(x2 + y + ay) = a2x2 + αy, ∀x, y ∈ R.

Page 85: Final Version 1989 to 2010 VN TST

84 Chương 2. Đáp án tuyển sinh

Suy ra a = a2 và a + a2 = α, hay a = 1, và α = 2, do α 6= 0. Dễ thấy, vớiα = 2 thì hàm số f(x) = x, x ∈ R, thoả mãn tất cả các điều kiện của đềbài.

Tóm lại, các kết quả đã thu được ở trên cho thấy: Có một và chỉ mộthàm số f : R → R thoả mãn hệ thức của đề bài khi và chỉ khi α = 2.

iii) Vậy, tất cả các số thực α thoả mãn yêu cầu của đề bài là α = 2.

Bài 3. 1) Gọi H là giao điểm của CM và PQ. Ta cần chứng minh PH =

HQ. Xét hai tam giác BCK và MBK có MBK = CKB, CBK = KMBvì cùng chắn cung BC của dường tròn tâm O1. Do đó 4BCK v 4MBK,suy ra

BC

MB=

CK

BK(1)

Tương tự ta có 4ACK v MAK, suy ra

AC

MA=

CK

AK(2)

Mà AK = BK nên từ (1) và (2) ta được

BC

MB=

AC

MA(3)

Bây giờ ta chứng minh hai tam giác MPB và CQB đồng dạng. Thật vậy,

BMP = BCQ, vì cùng bù với ACB, MPB = CQB vì cùng chắn cung ABcủa đường tròn tâm O2. Do đó, 4MPB v 4CQB. Suy ra,

BC

BM=

CQ

MP(4)

Từ (3) và (4) ta được

AC

MA=

CQ

MPhay

CQ

CA.MA

MP= 1 (5)

Áp dụng định lý Menelauss cho tam giác APQ với cát tuyết MCH, ta được

HP

HQ.CQ

CA.MA

MP= 1 (6)

Từ (5) và (6) suy ra PH = HQ.

2) Ta có CBM = PAQ = QBP, BCM = BAM = BQP . Do đó4BCM v 4BQP . Vì thế, tồn tại phép đồng dạng f tâm B biến 4BCMthành 4BQP .

Page 86: Final Version 1989 to 2010 VN TST

2.9. Đáp án chọn đội tuyển năm học 2003 - 2004 85

Vì các điểm B,C,M nằm trên (O1) và các điểm B,Q,P nằm trên (O2)nên f biến (O1) thành (O2). Gọi N là giao điểm thứ hai của AK và đườngtròn (O2), ta có

CBA = QAN = QBN

Mà f biến C thành Q nên từ đó suy ra f biến A thành N . Vì thế, phépđồng dạng f biến giao điểm K của các tiếp tuyến tại A và B của đườngtròn (O1) thành giao điểm L của các tiếp tuyến tại N và B của đường tròn(O2). Vì A,K là các điểm cố định nên N cũng là điểm cố định. Do đó L làđiểm cố định.

Vì f biến các điểm M,C thành các điểm P,Q và K nằm trên MC nênL nằm trên PQ. Vì thế, đường thẳng PQ luôn đi qua một điểm cố định khiđiểm M di động trên đường tròn (O1).

Bài 4. 1) Bằng cách tính toán trực tiếp, ta có x3 = 1201 và x4 = 2003.Viết lại hệ thức xác định dãy (xn) dưới dạng

xn+2 − xn+1 − xn = 2√

xn+1xn − 2 ∀n ≥ 1.

Từ đó suy ra

x2n+2 + x2

n+1 + x2n − 2xn+2xn+1 − 2xn+2xn + 2xn+1xn = 4(xn+1xn − 2)

Do đó

x2n+2 + x2

n+1 − 2xn+2xn+1 − 2xn+2xn − 2xn+1xn + 8 = 0 ∀n ≥ 1 (1)

x2n+3 + x2

n+2 − 2xn+3xn+2 − 2xn+3xn+1 − 2xn+2xn+1 + 8 = 0 ∀n ≥ 0 (2)

Lấy (2) trừ (1), ta được

(xn+3 − xn)(xn+3 + xn − 2xn+2 − 2xn+1) = 0 ∀n ≥ 1 (3)

Dễ thấy dãy (xn) là một dãy tăng. Vì thế, từ (3) ta được

xn+3 + xn − 2xn+2 − 2xn+1 = 0 ∀n ≥ 1.

hayxn+3 = 2xn+2 + 2xn+1 − xn, ∀n ≥ 1 (4)

Mà x1, x2, x3 là các số nguyên dương nên từ (4) suy ra xn là số nguyêndương với mọi n ≥ 1.

2) Với mỗi số nguyên dương n, ký hiệu rn là số dư trong phép chi xn

cho 104. Khi đó, từ (4) ta có

rn+3 ≡ 2rn+2 − rn (mod 104), ∀n ≥ 1 (5)

Page 87: Final Version 1989 to 2010 VN TST

86 Chương 2. Đáp án tuyển sinh

hay

rn ≡ 2rn+2 + 2rn+1 − rn+3 (mod 104), ∀n ≥ 1 (6)

Xét dãy các bộ ba số

(r1, r2, r3), (r2, r3, r4), ..., (rn, rn+1, rn+2), (rn+1, rn+2, rn+3), ...

Dãy trên có vô số bộ số. Tuy nhiên, do rn ∈ {0, 1, 2, ..., 104 − 1} ∀n ≥ 1nên chỉ có hữu hạn bộ đôi một khác nhau (tối đa là 1012 bộ). Vì thế, theonguyên lý Dirichlet, tồn tại hai bộ trùng nhau. Nghĩa là, tồn tại các sốnguyên dương m, k sao sao cho

(rm, rm+1, rm+2) = (rm+k, rm+k+1, rm+k+2).

Từ đó, nhờ (6) dễ chứng minh được

(r1, r2, r3) = (rk+1, rk+2, rk+3) (7)

Từ (7), nhờ (5), bằng phương pháp quy nạp theo n dễ dàng chứng minhđược dãy (rn) là dãy tuần hoàn, kể từ số hạng thứ nhất. Mà r4 = 2003 nênsuy ra có vô số số nguyên dương n sao cho rn = 2003. Nói cách khác, tồntại vô số số nguyên dương n sao cho biểu diễn thập phân của xn có bốn chữsố tận cùng là 2003.

3) Với mỗi số nguyên dương, ký hiệu sn là số dư trong phép chia xn cho4. Khi đó, từ (4) ta được

sn+3 ≡ 2sn+2 + 2sn+1 − sn (mod 4) ∀n ≥ 1 (8)

Bằng cách tính toán trực tiếp, ta có s1 = 3, s2 = 2, s3 = 1, s4 = 3, s5 =2, s6 = 1. Từ đó suy ra s1 = s4, s2 = s5 và s3 = s6. Từ đây, nhờ (8), bằngphương pháp quy nạp theo n, dễ dàng chứng minh được dãy sn) là dãy tuầnhoàn theo chu kỳ 3. Vì thế sn 6= 0 với mọi n ∈ N∗. Suy ra, với mọi n ∈ N∗,xn − 2004 không chia hết cho 4, và do đó không chia hết cho 104. Nói cáchkhác, không tồn tại số nguyên dương n sao cho xn có bốn chữ số tận cùnglà 2004.

Bài 5. Đặt AA1 = a,BB1 = b, CC1 = c,DD1 = d,EE1 = e, FF1 =f,A1B1 = x,B1C1 = y,C1D1 = z,D1E1 = t, E1F1 = u, F1A1 = v.

Gọi B2 là điểm đối xứng với B qua A1B1, C2 là điểm đối xứng với C

qua B1C1. Khi đó, do A1B1C1 = 120◦ nên B2B1C2 = 60◦. Suy ra tam giácB2B1C2 là tam giác đều. Vì thế B2C2 = b. Do đó

a + b + c = A1B2 + B2C2 + C2C1 ≥ A1C1 (1)

Page 88: Final Version 1989 to 2010 VN TST

2.9. Đáp án chọn đội tuyển năm học 2003 - 2004 87

Mặt khác, theo định lý hàm số cosin ta có

A1C1 =√

x2 + y2 + xyy ≥√

3

2(x + y) (2)

Từ (1) và(2) suy ra

a + b + c ≥√

3

2(x + y) (3)

Vì các cạnh của lục giác BACDEF có vai trò như nhau nên từ (3) suyra ta có các bất đẳng thức

b + c + d ≥√

3

2(y + z); c + d + e ≥

√3

2(z + t); d + e + f ≥

√3

2(t + u);

e + f + a ≥√

3

2(u + v); f + a + b ≥

√3

2(u + x).

Cộng vế theo vế bất đẳng thức (3) và năm bất đẳng thức trên, ta được

a + b + c + d + e + f ≥√

3

3(x + y + z + t + u + v)

hay p ≥ 2√

33

p1.

Dấu đẳng thức xảy ra khi và chỉ khi lục giác ABCDEF là lục giác đều.

Bài 6. Đặt s = |S| và f(n) = |Sn| ∀n ∈ N. Với mỗi n ∈ N, kí hiệuRn = {a+x | x ∈ Sn−1}. Rõ ràng, Rn là một tập con của Sn và |Rn| = |Sn−1|nên

f(n) = |Rn| + |Sn\Rn| = f(n − 1) + |Sn\Rn|, ∀n ∈ N.

Đặt m = s(a− 1). Dễ thấy tổng của n > m số thuộc S phải chứa ít nhất asố hạng trùng nhau. Do đó, với lưu ý rằng

d + d + · · · + d︸ ︷︷ ︸a số hạng

= a + a + · · · + a︸ ︷︷ ︸d số hạng

∀ d ∈ S.

suy ra tổng của n > m số thuộc S phải thuộc Rn. Vì vậy, Sn\Rn là một tậpcon của Sm. Do đó

Sn\Rn = Sm\Rn ∀n > m.

Đặt k = (m − 1)a. Rõ ràng, với n > k thì mỗi số thuộc Rn\Rn−1 đều lớnhơn hoặc bằng

a + n − 1 ≥ a + k = a + (m − 1)a = ma.

Page 89: Final Version 1989 to 2010 VN TST

88 Chương 2. Đáp án tuyển sinh

Do mọi số của Sm\Rk phải nhỏ hơn ma nên Sm\Rn = Sm\Rn−1 với mọin > k. Từ đó suy ra Sn\Rn = Sm\Rk, và do đó

|Sn\Rn| = |Sm\Rk| = |Sk\Rk| ∀n > k.

Vì thế, đặt b = f(k) − k|Sk\Rk|, ta có

f(n) = f(n−1)+ |Sk\Rk| = (n−k)|Sk\Rk|+f(k) = n|Sk\Rk|+b ∀n > k.

Tiếp theo, ta chứng minh |Sk\Rk| = a. Thật vậy, xét các trường hợp sau:Trường hợp 1. s = 2. Khi đó k = 2a2 − 3a ≥ a. Giả sử S = {c, a}. Do

(c, a) = 1 theo giả thiết của đề bài, nên Sk\Rk = {0, c, 2c, ..., (a − 1)c}. Vìvậy |Sk\Rk| = a.

Trường hợp 2. s > 2. Gọi c là số nhỏ nhất trong S, ta có (c, a) = 1.Vì thế, theo chứng minh ở trường hợp 1, tồn tại số nguyên tố b′ sao chof(n) ≥ an + b′ với mọi n > 2a2. Do đó, nếu |Sk\Rk| ≤ a− 1 thì

an + b′ ≤ f(n) = n|Sk\Rk| ≤ (a− 1)n + b.

Suy ra n ≤ b − b′, là điều vô lí vì n có thể lớn tuỳ ý. Như vậy, phải có|Sk\Rk| ≥ a. Vậy, tóm lại, tồn tại số nguyên dương k và số nguyên b saocho

|Sn| = an + b với mọi n > k.

Windows XP
Typewriter
Chapter II:2005 to 2010
Page 90: Final Version 1989 to 2010 VN TST

1

ĐỀ THI VÀ LỜI GIẢI

ĐỀ CHỌN ĐỘI TUYỂN QUỐC GIA

DỰ THI OLYMPIC TOÁN QUỐC TẾ

CỦA VIỆT NAM

TỪ NĂM 2005 ĐẾN NĂM 2010

Page 91: Final Version 1989 to 2010 VN TST

2

PHẦN I

*****

ĐỀ BÀI

Page 92: Final Version 1989 to 2010 VN TST

3

ĐỀ THI CHỌN ĐỘI TUYỂN QUỐC GIA DỰ THI IMO 2005

*Ngày thi thứ nhất. Bài 1. Cho tam giác ABC có (I) và (O) lần lượt là các đường tròn nội tiếp, ngoại tiếp.

Gọi D, E, F lần lượt là tiếp điểm của (I) trên các cạnh BC, CA, AB. Gọi , , A B Cω ω ω lần lượt là

các đường tròn tiếp xúc với hai đường tròn (I) và (O) lần lượt tại các điểm D, K (với đường

tròn Aω ); tại E, M (với đường tròn Bω ) và tại F, N (với đường tròn Cω ). Chứng minh rằng:

1. Các đường thẳng , , DK EM FN đồng quy tại P.

2. Trực tâm của tam giác DEF nằm trên đoạn OP.

Bài 2. Trên một vòng tròn có n chiếc ghế được đánh số từ 1 đến n. Người ta chọn ra

k chiếc ghế. Hai chiếc ghế được chọn gọi là kề nhau nếu đó là hai chiếc ghế được chọn liên

tiếp. Hãy tính số cách chọn ra k chiếc ghế sao cho giữa hai chiếc ghế kề nhau, không có ít

hơn 3 chiếc ghế khác. Bài 3. Tìm tất cả các hàm số :f →ℤ ℤ thỏa mãn điều kiện:

3 3 3 3 3 3( ) ( ( )) ( ( )) ( ( ))f x y z f x f y f z+ + = + +

*Ngày thi thứ hai.

Bài 4. Chứng minh rằng: 3 3 3

3 3 3

3

( ) ( ) ( ) 8

a b c

a b b c c a+ + ≥

+ + +

trong đó , ,a b c là các số thực dương.

Bài 5. Cho số nguyên tố ( 3)p p > . Tính:

a)

12 22

1

22

p

k

k kS

p p

=

= −

∑ nếu 1 (mod 4)p ≡ .

b)

122

1

p

k

kS

p

=

=

∑ nếu 1 (mod8)p ≡ .

Bài 6. Một số nguyên dương được gọi là “số kim cương 2005” nếu trong biểu diễn

thập phân của nó có 2005 số 9 đứng cạnh nhau liên tiếp. Dãy ( ) , 1, 2,3,...na n = là dãy tăng

ngặt các số nguyên dương thỏa mãn na nC< (C là hằng số thực dương nào đó).

Chứng minh rằng dãy số ( ) , 1, 2,3,...na n = chứa vô hạn “số kim cương 2005”.

Page 93: Final Version 1989 to 2010 VN TST

4

ĐỀ THI CHỌN ĐỘI TUYỂN QUỐC GIA

DỰ THI IMO 2006

* Ngày thi thứ nhất.

Bài 1. Cho tam giác ABC có H là trực tâm. Đường phân giác ngoài của góc BHC cắt các

cạnh AB, AC lần lượt tại D và E. Đường phân giác trong của góc BAC cắt đường tròn ngoại

tiếp tam giác ADE tại điểm K. Chứng minh rằng đường thẳng HK đi qua trung điểm của BC.

Bài 2. Hãy tìm tất cả các cặp số tự nhiên ( ) ; n k với n là số nguyên không âm và k là

số nguyên lớn hơn 1 sao cho số : 2006 2 517 4.17 7.19n n nA = + + có thể phân tích được thành

tích của k số nguyên dương liên tiếp.

Bài 3. Trong không gian cho 2006 điểm mà trong đó không có 4 điểm nào đồng

phẳng. Người ta nối tất cả các điểm đó lại bởi các đoạn thẳng. Số tự nhiên m gọi là số tốt nếu

ta có thể gán cho mỗi đoạn thẳng trong các đoạn thẳng đã nối bởi một số tự nhiên không

vượt quá m sao cho mỗi tam giác tạo bởi ba điểm bất kì trong số các điểm đó đều có hai

cạnh được gán bởi hai số bằng nhau và cạnh còn lại gán bởi số lớn hơn hai số đó.

Tìm số tốt có giá trị nhỏ nhất.

* Ngày thi thứ hai .

Bài 4. Chứng minh rằng với mọi số thực , , [1;2]x y z∈ , ta luôn có bất đẳng thức sau :

1 1 1( )( ) 6( )

x y zx y z

x y z y z z x x y+ + + + ≥ + +

+ + + .

Hỏi đẳng thức xảy ra khi và chỉ khi nào ?

Bài 5. Cho tam giác ABC là tam giác nhọn, không cân, nội tiếp trong đường tròn tâm

O bán kính R. Một đường thẳng d thay đổi sao cho d luôn vuông góc với OA và luôn cắt các

tia AB, AC. Gọi M, N lần lượt là giao điểm của đường thẳng d và các tia AB, AC. Giả sử các

đường thẳng BN và CN cắt nhau tại K; giả sử đường thẳng AK cắt đường thẳng BC.

1. Gọi P là giao của đường thẳng AK và đường thẳng BC. Chứng minh rằng đường tròn

ngoại tiếp của tam giác MNP luôn đi qua một điểm cố định khi d thay đổi.

2. Gọi H là trực tâm của tam giác AMN. Đặt BC = a và l là khoảng cách từ điểm A đến HK.

Chứng minh rằng đường thẳng HK luôn đi qua trực tâm của tam giác ABC.

Từ đó suy ra: 2 24l R a≤ − . Đẳng thức xảy ra khi và chỉ khi nào?

Bài 6. Cho dãy số thực ( )na được xác định bởi:

0 1

1 11, ( )

2 3n nn

a a aa+= = + với mọi n = 1, 2, 3, …

Chứng minh rằng với mọi số nguyên n, số2

3

3 1nn

Aa

=−

là một số chính phương và nó có ít

nhất n ước nguyên tố phân biệt.

Page 94: Final Version 1989 to 2010 VN TST

5

ĐỀ THI CHỌN ĐỘI TUYỂN QUỐC GIA DỰ THI IMO 2007

*Ngày thi thứ nhất.

Bài 1. Cho hai tập hợp A,B là tập hợp các số nguyên dương thỏa mãn A B n= =

(với n là số nguyên dương) và có tổng các phần tử bằng nhau. Xét bảng ô vuông n n× .

Chứng minh rằng ta có thể điền vào mỗi ô vuông của bảng một số nguyên không âm

thỏa mãn đồng thời các điều kiện:

i/ Tổng của các phần tử ở mỗi hàng là các phần tử của tập A.

ii/ Tổng của các phần tử ở mỗi cột là các phần tử của tập B.

iii/ Có ít nhất 2( 1)n k− + số 0 trong bảng với k là số các phần tử chung của A và B.

Bài 2. Cho tam giác nhọn ABC với đường tròn nội tiếp I. Gọi ( )ak là đường tròn có

tâm nằm trên đường cao của góc A, đi qua điểm A và tiếp xúc trong với đường tròn (I) tại 1A .

Các điểm 1 1, B C xác định tương tự .

1/ Chứng minh 1 1 1, , AA BB CC đồng qui tại P.

2/ Gọi ( ), ( ), ( )a b cJ J J lần lượt là các đường tròn đối xứng với đường tròn bàng tiếp

các góc A, B, C của tam giác ABC qua trung điểm BC, CA, AB.

Chứng minh P là tâm đẳng phương của 3 đường tròn nói trên.

Bài 3. Cho tam giác ABC. Tìm giá trị nhỏ nhất của biểu thức sau:

2 2 2 2 2 2

2 2 2

cos cos cos cos cos cos2 2 2 2 2 2

cos cos cos2 2 2

A B B C C A

SC A B

= + + .

*Ngày thi thứ hai.

Bài 4. Tìm tất cả các hàm số liên tục :f →ℝ ℝ thỏa mãn:

2 1( ) ( )

3 9

xf x f x= + + với mọi x∈ℝ .

Bài 5. Cho A là tập con chứa 2007 phần tử của tập:{1, 2, 3,..., 4013, 4014} thỏa mãn

với mọi ,a b A∈ thì a không chia hết cho b. Gọi mA là phần tử nhỏ nhất của A.

Tìm giá trị nhỏ nhất của mA với A thỏa mãn các điều kiện trên.

Bài 6. Cho đa giác 9 cạnh đều (H). Xét ba tam giác với các đỉnh là các đỉnh của đa giác

(H) đã cho sao cho không có hai tam giác nào có chung đỉnh.

Chứng minh rằng có thể chọn được từ mỗi tam giác 1 cạnh sao cho 3 cạnh này bằng nhau.

Page 95: Final Version 1989 to 2010 VN TST

6

ĐỀ THI CHỌN ĐỘI TUYỂN QUỐC GIA DỰ THI IMO 2008

*Ngày thi thứ nhất.

Bài 1. Trong mặt phẳng cho góc xOy. Gọi M, N lần lượt là hai điểm lần lượt nằm trên

các tia Ox, Oy. Gọi d là đường phân giác góc ngoài của góc xOy và I là giao điểm của trung

trực MN với đường thẳng d. Gọi P, Q là hai điểm phân biệt nằm trên đường thẳng d sao

cho IM IN IP IQ= = = , giả sử K là giao điểm của MQ và NP.

1. Chứng minh rằng K nằm trên một đường thẳng cố định.

2. Gọi d1 là đường thẳng vuông góc với IM tại M và d2 là đường thẳng vuông góc với IN

tại N. Giả sử các đường thẳng d1, d2 cắt đường thẳng d tại E, F. Chứng minh rằng các đường

thẳng EN, FM và OK đồng quy.

Bài 2. Hãy xác định tất cả các số nguyên dương m sao cho tồn tại các đa thức với hệ

số thực ( ), ( ), ( , )P x Q x R x y thỏa mãn điều kiện:

Với mọi số thực a, b mà 2 0ma b− = , ta luôn có ( ( , ))P R a b a= và ( ( , ))Q R a b b= .

Bài 3. Cho số nguyên n > 3. Kí hiệu T là tập hợp gồm n số nguyên dương đầu tiên.

Một tập con S của T được gọi là tập khuyết trong T nếu S có tính chất: Tồn tại số nguyên

dương c không vượt quá 2

n sao cho với 1 2,s s là hai số bất kì thuộc S ta luôn có 1 2s s c− ≠ .

Hỏi tập khuyết trong T có thể có tối đa bao nhiêu phần tử ?

*Ngày thi thứ hai.

Bài 4. Cho m, n là các số nguyên dương. Chứng minh rằng (2 3) 1nm + + chia hết cho

6m khi và chỉ khi 3 1n + chia hết cho 4m.

Bài 5. Cho tam giác ABC nhọn, không cân có O là tâm đường tròn ngoại tiếp.

Gọi AD, BE, CF là các đường phân giác trong của tam giác. Trên các đường thẳng AD, BE, CF

lần lượt lấy các điểm L, M, N sao cho AL BM CN

kAD BE CF

= = = (k là một hằng số dương).

Gọi (O1), (O2), (O3) lần lượt là các đường tròn đi qua L, tiếp xúc với OA tại A ; đi qua M, tiếp

xúc với OB tại B và đi qua N, tiếp xúc với OC tại C.

1. Chứng minh rằng với 1

2k = , ba đường tròn (O1), (O2), (O3) có đúng hai điểm chung

và đường thẳng nối hai điểm chung đó đi qua trọng tâm tam giác ABC. 2. Tìm tất cả các giá trị k sao cho 3 đường tròn (O1), (O2), (O3) có đúng hai điểm chung.

Bài 6. Kí hiệu M là tập hợp gồm 2008 số nguyên dương đầu tiên. Tô tất cả các số thuộc M bởi ba màu xanh, vàng, đỏ sao cho mỗi số được tô bởi một màu và mỗi màu đều được dùng để tô ít nhất một số. Xét các tập hợp sau:

31 {( , , ) ,S x y z M= ∈ trong đó x, y, z có cùng màu và ( ) 0 (mod 2008)}x y z+ + ≡ ;

32 {( , , ) ,S x y z M= ∈ trong đó x, y, z đôi một khác màu và ( ) 0 (mod 2008)}x y z+ + ≡ .

Chứng minh rằng 1 22 S S> . (Kí hiệu 3M là tích Đề - các M M M× × ) .

Page 96: Final Version 1989 to 2010 VN TST

7

ĐỀ THI CHỌN ĐỘI TUYỂN QUỐC GIA DỰ THI IMO 2009

*Ngày thi thứ nhất. Bài 1. Cho tam giác nhọn ABC nội tiếp đường tròn (O). Gọi 1 1 1, , A B C và 2 2 2, , A B C

lần lượt là các chân đường cao của tam giác ABC hạ từ các đỉnh A, B, C và các điểm đối xứng với 1 1 1, , A B C qua trung điểm của các cạnh , , BC CA AB . Gọi 3 3 3, , A B C lần lượt là các giao

điểm của đường tròn ngoại tiếp các tam giác 2 2 2 2 2 2, , AB C BC A CA B với (O).

Chứng minh rằng: 1 3 1 3 1 3, , A A B B C C đồng quy.

Bài 2. Cho đa thức 3 2( ) 1P x rx qx px= + + + trong đó , , p q r là các số thực và 0r > .

Xét dãy số ( )na xác định như sau:

21 2 3

3 2 1

1, ,

. . . , 0 n n n n

a a p a p q

a p a q a r a n+ + +

= = − = −

= − − − ≥

Chứng minh rằng: nếu đa thức ( )P x có một nghiệm thực duy nhất và không có

nghiệm bội thì dãy số ( )na có vô số số âm.

Bài 3. Cho các số nguyên dương , a b sao cho , a b và ab đều không phải là số chính

phương. Chứng minh rằng trong hai phương trình sau:

2 2

2 2

1

1

ax by

ax by

− =

− = −

có ít nhất một phương trình không có nghiệm nguyên dương.

*Ngày thi thứ hai.

Bài 4. Tìm tất cả các số thực r sao cho bất đẳng thức sau đúng với mọi a, b, c dương: 3

1

2

a b cr r r r

b c c a a b + + + ≥ + + + +

Bài 5. Cho đường tròn (O) có đường kính AB và M là một điểm bất kì nằm trong (O), M không nằm trên AB. Gọi N là giao điểm của phân giác trong góc M của tam giác AMB với đường tròn (O). Đường phân giác ngoài góc AMB cắt các đường thẳng NA, NB lần lượt tại P, Q. Đường thẳng MA cắt đường tròn đường kính NQ tại R, đường thẳng MB cắt đường tròn đường kính NP tại S và R, S khác M.

Chứng minh rằng: đường trung tuyến ứng với đỉnh N của tam giác NRS luôn đi qua một điểm cố định khi M di động phía trong đường tròn.

Bài 6. Một hội nghị toán học có tất cả 6 4n + nhà toán học phải họp với nhau đúng

2 1n + lần ( )1n ≥ . Mỗi lần họp, họ ngồi quanh một cái bàn 4 chỗ và n cái bàn 6 chỗ, các vị

trí ngồi chia đều khắp mỗi bàn. Biết rằng hai nhà toán học đã ngồi cạnh hoặc đối diện nhau ở một cuộc họp này thì sẽ không được ngồi cạnh hoặc đối diện nhau ở một cuộc họp khác.

a/ Chứng minh rằng Ban tổ chức có thể xếp được chỗ ngồi nếu 1n = . b/ Hỏi rằng Ban tổ chức có thể sắp xếp được chỗ ngồi được hay không với mọi 1n > ?

Page 97: Final Version 1989 to 2010 VN TST

8

ĐỀ THI CHỌN ĐỘI TUYỂN QUỐC GIA DỰ THI IMO 2010

* Ngày thi thứ nhất.

Bài 1. Cho tam giác ABC không vuông tại A có đường trung tuyến AM. Gọi D là một

điểm di động trên đường thẳng AM. Gọi 1 2( ), ( )O O là các đường tròn đi qua D, tiếp xúc với

BC lần lượt tại B và C. Gọi P, Q lần lượt là giao điểm của đường thẳng AB với đường tròn

1( )O , đường thẳng AC với đường tròn 2( )O . Chứng minh rằng:

1. Tiếp tuyến tại P của 1( )O và tiếp tuyến tại Q của 2( )O phải cắt nhau tại một điểm.

Gọi giao điểm đó là S.

2. Điểm S luôn di chuyển trên một đường thẳng cố định khi D di động trên AM.

Bài 2. Với mỗi số n nguyên dương, xét tập hợp sau :

{ }11( ) 10( ) |1 , 10k hnT k h n n k h= + + + ≤ ≤ .

Tìm tất cả giá trị của n sao cho không tồn tại , ; na b T a b∈ ≠ sao cho ( )a b− chia hết cho 110.

Bài 3. Gọi một hình chữ nhật có kích thước 1 2× là hình chữ nhật đơn và một hình

chữ nhật có kích thước 2 3× , bỏ đi 2 ô ở góc chéo nhau (tức là có 4 ô vuông nhỏ) là hình chữ

nhật kép. Người ta ghép khít các hình chữ nhật đơn và hình chữ nhật kép này lại với nhau

được một bảng hình chữ nhật có kích thước là 2008 2010× .

Tìm số bé nhất các hình chữ nhật đơn có thể dùng để ghép.

* Ngày thi thứ hai.

Bài 4. Cho , ,a b c là các số thực dương thỏa mãn điều kiện: 1 1 1

16( )a b ca b c

+ + ≥ + + .

Chứng minh rằng:

3 3 3

1 1 1 8

9( 2( )) ( 2( )) ( 2( ))a b a c b c b a c a c b+ + ≤

+ + + + + + + + +.

Hỏi đẳng thức xảy ra khi nào?

Bài 5: Trong một hội nghị có n nước tham gia, mỗi nước có k đại diện ( )1n k> > .

Người ta chia .n k người này thành n nhóm, mỗi nhóm có k người sao cho không có hai

người nào cùng nhóm đến từ cùng một nước.

Chứng minh rằng có thể chọn ra một nhóm gồm n người sao cho họ thuộc các nhóm khác

nhau và đến từ các nước khác nhau.

Bài 6: Gọi nS là tổng bình phương các hệ số trong khai triển của nhị thức (1 )nx+ ,

trong đó n là số nguyên dương; x là số thực bất kì.

Chứng minh rằng: 2 1nS + không chia hết cho 3 với mọi n.

Page 98: Final Version 1989 to 2010 VN TST

9

PHẦN II *****

LỜI GIẢI

Page 99: Final Version 1989 to 2010 VN TST

10

LỜI GIẢI ĐỀ THI

CHỌN ĐỘI TUYỂN QUỐC GIA DỰ THI IMO 2005

Bài 1. Cho tam giác ABC có (I) và (O) lần lượt là các đường tròn nội tiếp, ngoại tiếp.

Gọi D, E, F lần lượt là tiếp điểm của (I) trên các cạnh BC, CA, AB. Gọi , , A B Cω ω ω lần lượt

là các đường tròn tiếp xúc với hai đường tròn (I) và (O) lần lượt tại các điểm D, K (với đường

tròn Aω ); tại E, M (với đường tròn Bω ) và tại F, N (với đường tròn Cω ). Chứng minh rằng:

1. Các đường thẳng DK, EM, FN đồng quy tại P. 2. Trực tâm của tam giác DEF nằm trên đoạn OP.

1. Trước hết, ta sẽ chứng minh bổ đề sau: Cho ba đường tròn (O1), (O2), (O3) có bán kính đôi một khác nhau; A, B, C lần lượt là tâm vị tự của các cặp đường tròn (O1) và (O2), (O2) và (O3), (O3) và (O1). Chứng minh rằng nếu trong các tâm vị tự đó, có ba tâm vị tự ngoài hoặc hai tâm vị tự trong, một tâm vị tự ngoài thì A, B, C thẳng hàng. *Chứng minh:

Gọi 1 2 3, ,R R R lần lượt là bán kính của các đường

tròn 1 2 3( ), ( ), ( )O O O , các giá trị 1 2 3, ,R R R này đôi

một khác nhau.

Theo tính chất về tâm vị tự, ta có: 1 1

22

( 1)aAO R

RAO= − .

Tương tự: 2 2

33

( 1)bBO R

RBO= − , 3 3

11

( 1)cCO R

RCO= − , trong

đó, mỗi số , ,a b c nhận giá trị là 0 (khi nó là tâm vị tự ngoài) hoặc 1 (khi nó là tâm vị tự trong).

Theo giả thiết trong a, b, c có ba giá trị là 0 hoặc hai giá trị 0, một giá trị 1. Từ đó:

31 2

2 3 1

. . 1COAO BO

AO BO CO= , theo định lí Menelaus đảo cho tam giác 1 2 3O O O , ta có: A, B, C thẳng hàng.

Bổ đề được chứng minh. *Trở lại bài toán: Gọi P’ là tâm vị tự trong của hai đường tròn (O) và (I). Dễ thấy: D là điểm tiếp xúc ngoài

của Aω và (I) nên cũng chính là tâm vị tự trong của hai đường tròn này; K là điểm tiếp xúc trong

của hai đường tròn Aω và (O) nên là tâm vị tự ngoài của hai đường tròn này. Theo bổ đề trên thì

', ,P D K thẳng hàng hay đường thẳng DK đi qua P’. Tương tự, các đường thẳng EM và FN cũng

đi qua P’; tức là ba đường thẳng DK, EM, FN đồng quy và điểm P’ chính là điểm P của đề bài.

B

C

AO1O2

O3

Page 100: Final Version 1989 to 2010 VN TST

11

2. Ta chứng minh bổ đề sau: Cho tam giác ABC có (O), (I) lần lượt là tâm đường tròn ngoại tiếp, nội tiếp của tam giác

ABC. Đường tròn (I) tiếp xúc với các cạnh BC, CA, AB lần lượt tại D, E, F. Chứng minh rằng trực tâm H của tam giác DEF nằm trên đường thẳng OI.

* Chứng minh: Gọi M, N, P lần lượt là trung điểm của các

đoạn EF, FD, DE. Dễ thấy AI là trung trực của đoạn EF nên M thuộc đường thẳng AI hay A, M, I thẳng hàng. Tương tự: B, N, I và C, P, I cũng thẳng hàng. Xét phép nghịch đảoΦ tâm I, phương

tích 2r với r là bán kính đường tròn (I). Dễ thấy: tam giác IEA vuông tại E có EM là

đường cao nên: 2 2.IM IA IE r= = , suy ra: : M AΦ → . Tương tự: : ,N B P CΦ → → .

Do đó: : MNP ABCΦ ∆ → ∆ . Gọi E là tâm đường tròn ngoại tiếp của tam giác MNP thì

: E OΦ → , suy ra: E, I, O thẳng hàng.

Hơn nữa, I là tâm đường tròn ngoại tiếp tam giác DEF, E là tâm đường tròn ngoại tiếp tam giác MNP cũng chính là tâm đường tròn Euler của tam giác DEF này nên E, I, H thẳng hàng.

Từ đó suy ra H, I, O thẳng hàng. Bổ đề được chứng minh.

* Trở lại bài toán: Gọi H là trực tâm tam giác DEF thì theo bổ đề trên: H, I, O thẳng hàng. Theo câu 1/, điểm P nằm trên đoạn OI. Suy ra: 4 điểm H, I, P, O thẳng hàng. Từ đó suy ra trực tâm H của tam giác DEF nằm trên đường thẳng OI. Ta có đpcm.

N

M

H

K

PF

E

D

IO

B C

A

HP

N

MF

E

D

I O

A

B C

Page 101: Final Version 1989 to 2010 VN TST

12

Bài 2. Trên một vòng tròn có n chiếc ghế được đánh số từ 1 đến n. Người ta chọn ra k chiếc ghế. Hai chiếc ghế được chọn gọi là kề nhau nếu đó là hai chiếc ghế được chọn liên tiếp. Hãy tính số cách chọn ra k chiếc ghế sao cho giữa hai chiếc ghế kề nhau, không có ít hơn 3 chiếc ghế khác. *Trước hết, ta chứng minh bổ đề sau: Cho n điểm phân biệt nằm trên đường thẳng được tô một trong hai màu, xanh hoặc đỏ thỏa mãn các điều kiện sau:

- Có đúng k điểm được tô màu xanh. - Giữa hai điểm màu xanh liên tiếp có ít nhất p điểm được tô màu đỏ (tính từ trái sang). - Ở bên phải điểm màu xanh cuối cùng có ít nhất p điểm được tô màu đỏ.

Khi đó, số cách tô màu là: kn kpC − .

*Chứng minh: Đánh số các điểm đã cho là 1, 2,3,..., n . Đặt tương ứng mỗi cách tô màu

với một bộ k các số nguyên dương 1 2( , ,..., )ki i i trong đó 1 2, ,..., ki i i là các điểm được tô màu xanh.

Dễ thấy tương ứng nói trên chính là một song ánh từ tập các cách tô màu đến tập hợp T sau:

1 2 1{( , ,..., ) | {1,2,..., }, 1, ; , 1, 1}k s s sT i i i i n p s k i i p i k+= ∈ − ∀ = − > ∀ = − .

Xét ánh xạ sau:

1 2 1' {( , ,..., ) | {1,2,..., }, , 1, }k t t tT T j j j j n kp j j t k+→ = ∈ − > ∀ = .

Ta sẽ chứng minh ánh xạ này là một song ánh. Thật vậy:

*Xét một bộ 1 2( , ,..., ) 'kj j j T∈ . Khi đó, ta xét tiếp bộ: 1 2 3( , , 2 ,..., ( 1) )kj j p j p j k p+ + + − .

Do 1 tj n kp≤ ≤ − nên phần tử lớn nhất của bộ này là ( 1)kj k p+ − có giá trị không vượt quá

( 1)n kp k p n p− + − = − . Từ đó suy ra: ( 1) {1,2,..., }, 1tj t p n kp t+ − ∈ − ∀ ≥ .

Hơn nữa: [ ] [ ]1 1( 1) ( )t t t tj tp j t p j j p p+ ++ − + − = − + > .

Từ đó suy ra bộ 1 2 3( , , 2 ,..., ( 1) )kj j p j p j k p T+ + + − ∈ .

Do đó, tương ứng này là một toàn ánh.

*Xét bộ 1 2( , ,..., )ki i i T∈ . Khi đó, hoàn toàn tương tự trên, ta cũng chứng minh được bộ

1 2 2( , , 2 ,..., ( 1) ) 'ki i p i p i k p T− − − − ∈ .

Ta sẽ chứng minh rằng nếu có hai bộ khác nhau 1 2 1 2( , ,..., ), ( ' , ' ,..., ' )k ki i i i i i T∈ thì các bộ tương

ứng thuộc T= 'T của chúng cũng phải khác nhau. Nhưng điều này là hiển nhiên do hai bộ này là

khác nhau nên tồn tại chỉ số s sao cho 's si i≠ , khi đó ( 1) ' ( 1)s si s p i s p− − ≠ − − .

Suy ra, tương ứng này cũng là một đơn ánh. Vậy tương ứng 'T T→ là một song ánh. Nhận xét trên được chứng minh.

Do đó: | | | ' | kn kpT T C −= = . Bổ đề được chứng minh.

Page 102: Final Version 1989 to 2010 VN TST

13

*Trở lại bài toán: Ta xét tổng quát giá trị 3 trong đề bài bởi giá trị p tương ứng với bổ đề trên.

Đánh số các ghế trong đề bài theo chiều kim đồng hồ là 1 2, ,..., nA A A (xem như là các điểm nằm

trên một vòng tròn) ; mỗi ghế được chọn xem như được tô màu xanh và không được chọn xem như được tô màu đỏ; gọi X là tập hợp tất cả các cách tô màu k điểm trong n điểm đã cho thỏa mãn đề bài.

Xét phân hoạch: ' '' ' "X X X X X X= ∪ ⇒ = + .

trong đó 'X là cách tô màu thỏa mãn có một điểm được tô màu xanh thuộc 1 2 3{ , , ,..., }pA A A A và

'' \ 'X X X= , khi đó rõ ràng, với mọi phần tử thuộc "X thì không có điểm nào được tô màu

xanh thuộc 1 2 3{ , , ,..., }pA A A A , tức là mọi điểm trong tập này đều được tô màu đỏ. Ta cắt đường

tròn ngay tại điểm 1,p pA A + thì rõ ràng sẽ tạo được một đường thẳng thỏa mãn tất cả điều kiện

của bổ đề đã nêu ở trên, suy ra: '' kn kpX C −= . Ta chỉ còn cần tính số phần tử của 'X .

Xét tập hợp 'iX trong đó mỗi phần tử của 'iX có đúng một điểm iA được tô màu xanh,

1,i p= ; khi đó rõ ràng ' ' ,i jX X i j∩ = ∅ ∀ ≠ và 1

' 'p

ii

X X=

=∪ .

Với mỗi 1,i p= , theo bổ đề trên, ta thấy: 1 11 ( 1) 1

k kn p k p n kpC C− −− − − − − −= , tức là các tập 'iX này có cùng số phần tử. Suy ra: 1

1' kn kpX pC −− −= .

Do đó: 11' '' k k

n kp n kpX X X C pC −− − −= + = + .

Thay 3p = , ta được số cách chọn ghế tương ứng trong đề bài là 13 3 13k k

n k n kC C −− − −+ .

Vậy số cách chọn ghế thỏa mãn tất cả các điều kiện của đề bài là: 13 3 13k k

n k n kC C −− − −+ .

Page 103: Final Version 1989 to 2010 VN TST

14

Bài 3. Tìm tất cả các hàm số :f →ℤ ℤ thỏa mãn điều kiện: 3 3 3 3 3 3( ) ( ( )) ( ( )) ( ( ))f x y z f x f y f z+ + = + +

* Trước hết, ta sẽ chứng minh bổ đề sau: Với mọi số nguyên dương lớn hơn 10, lập phương của nó đều có thể biểu diễn được dưới dạng tổng của 5 lập phương của các số nguyên khác có giá trị tuyệt đối nhỏ hơn nó. * Thật vậy: Ta cần tìm mối liên hệ đó với số 10n > trong từng trường hợp n chẵn và n lẻ.

- Với n là số lẻ, đặt 2 1n k= + . Ta cần tìm một đẳng thức đúng với mọi k mà trong đó 2 1k + là biểu thức có giá trị tuyệt đối lớn nhất, các biểu thức còn lại phải là nhị thức bậc nhất có hệ số của k lớn nhất là 2. Khi đó

để khử 38k xuất hiện ở trong ( )32 1k + , ta chọn ( )3

2 1k− − ; ta thấy vẫn còn số hạng chứa k bậc

hai trong đó, ta chọn tiếp hai biểu thức khác có chứa k cùng hai hằng số bằng cách dùng tham số như sau: Giả sử hai biểu thức cần tìm có dạng ( ), ( ); ,ak b ak b a b+ − ∈ℤ và hai số cần tìm là

,c d ∈ℤ , tức là:

( ) ( ) ( )

3 3 3 3 3 3

2 2 2 3 3 3

2 2 3 3 3

(2 1) (2 1) ( ) ( )

24 2 6 2

(24 6 ) (2 2 ) 0

k k ak b ak b c d

k a bk b c d

k a b b c d

+ − − = + − − + +

⇔ + = + + +

⇔ − + − − − =

Ta cần chọn , , ,a b c d sao cho 2 3 3 34, 2 2a b b c d= + + = trong đó 2a ≤ .

Dễ thấy 2a ≠ vì nếu 2a = thì từ 2 4 1a b b= ⇒ = , trùng với biểu thức cần đánh giá; do đó,

1, 4a b= = , suy ra: 3 3 126c d+ = − , ta chọn được 5, 1c d= − = − .

Do đó:

3 3 3 3 3 3(2 1) (2 1) ( 4) (4 ) ( 5) ( 1)k k k k+ = − + + + − + − + − (1)

Thử lại, ta thấy biểu thức này đúng với mọi k. - Với n là số chẵn, đặt 2 2n k= + .

Lập luận hoàn toàn tương tự, ta có được đẳng thức sau: 3 3 3 3 3 3(2 2) (2 2) ( 8) (8 ) ( 10) ( 2)k k k k+ = − + + + − + − + − (2)

Bổ đề được chứng minh. *Trở lại bài toán:

Trong đẳng thức đã, thay 0x y z= = = , ta được: 3 2(0) 3 (0) (0) 0 3 (0) 1f f f f= ⇔ = ∨ = .

Do hàm này chỉ lấy giá trị trên :f →ℤ ℤ nên không thể có 23 (0) 1f = , tức là (0) 0f = .

Page 104: Final Version 1989 to 2010 VN TST

15

- Thay 0y z= = , ta có: 3 3 3 3 3( ) ( ( )) ( (0)) ( (0)) ( ( ))f x f x f f f x= + + = .

- Lại thay y z= − , ta có: 3 3 3 3 3 3( ) ( ( )) ( ( )) ( ( )) ( ( )) ( ( )) 0 ( ) ( ),f x f x f y f y f y f y f y f y y= + + − ⇔ + − = ⇔ = − − ∀ ∈ℤ .

Ta sẽ chứng minh rằng: 3: ( ) . (1)k f k k f∀ ∈ =ℤ bằng quy nạp. (*)

*Thật vậy:

- Với 1k = , trong giả thiết, thay 1, 0x y z= = = , ta có 3 3(1) (1) ( 1) (1)f f f f= ⇒ − = −

- Với 2k = , trong giả thiết, thay 1, 0x y z= = = , ta có 3 3(2) 2 (1) ( 2) 2 (1)f f f f= ⇒ − = −

- Với 3k = , trong giả thiết, thay 1x y z= = = , ta có 3 3(3) 3 (1) ( 3) 3 (1)f f f f= ⇒ − = −

- Thay 2, 0x y z= = = , ta có 3 3 3 3(8) (2) (2 (1)) 8 (1) ( 8) 8 (1)f f f f f f= = = ⇒ − = − .

- Thay 2, 1, 0x y z= = = , ta có 3 3 3 3(9) (2) (1) 9 (1) ( 9) 9 (1)f f f f f f= + = ⇒ − = − .

- Thay 2, 1x y z= = = , ta có: 3 3 3 3(10) (2) 2 (1) 10 (1) ( 10) 10 (1)f f f f f f= + = ⇒ − = − .

- Thay 2, 1, 0x y z= = − = , ta có: 3 3 3 3(7) (2) (1) 7 (1) ( 7) 7 (1)f f f f f f= − = ⇒ − = − .

- Thay 2, 1x y z= = = − , ta có: 3 3 3 3(6) (2) 2 (1) 6 (1) ( 6) 6 (1)f f f f f f= − = ⇒ − = − .

- Trong đẳng thức (1) của bổ đề trên, ta thay 2k = , suy ra: 3 3 3 3 3 35 3 6 2 ( 5) ( 1)= + + + − + − hay 3 3 3 3 3 3(5 5 1 ) (3 6 2 )f f+ + = + + .

3 3 3 3 3 3 32 (5) (1 ) (3) (6) (2) (5) 5 (1) ( 5) 5 (1)f f f f f f f f f⇒ + = + + ⇒ = ⇒ − = − .

- Trong đẳng thức (2) của bổ đề trên, ta thay 1k = , suy ra: 3 3 3 3 3 34 0 9 7 ( 10) ( 2)= + + + − + − hay 3 3 3 3 3 3(4 10 2 ) (9 7 0 )f f+ + = + + .

3 3 3 3 3 3 3 3(4) (10) (2) (9) (7) (0) (4) 4 (1) ( 4) 4 (1)f f f f f f f f f f⇒ + + = + + ⇒ = ⇒ − = − .

Như thế, ta đã chứng minh được (*) đúng với mọi 10k ≤ .

Với 10k > , xét 0k > thì theo bổ đề ở trên, lập phương của k đều có thể biểu diễn được

dưới dạng tổng của 5 lập phương khác có giá trị tuyệt đối nhỏ hơn nó.

Hơn nữa, dễ thấy rằng với , , , , ,a b c d e f ∈ℤ thỏa 3 3 3 3 3 3a b c m n p+ + = + + và ta đã có: 3 3 3 3 3( ) (1), ( ) (1), ( ) (1), ( ) (1), ( ) (1)f b bf f c cf f m mf f n nf f p pf= = = = = thì 3( ) (1)f a af= .

Từ đó, suy ra 3( ) (1), 10f k kf k= ∀ > .

Với 10k < − thì 3 3( ) ( ) ( (1)) (1)f k f k kf kf= − − = − − = .

Do đó, theo nguyên lí quy nạp (*) được chứng minh.

Mặt khác, trong giả thiết đã cho, thay 1, 0x y z= = = , ta có: 3(1) (1) (1) 1 (1) 0f f f f= ⇔ = ± ∨ = .

- Nếu (1) 1f = − thì ( ) ,f k k k= − ∀ ∈ℤ , thử lại thấy thỏa.

- Nếu (1) 0f = thì ( ) 0,f k k= ∀ ∈ℤ , thử lại thấy thỏa.

- Nếu (1) 1f = thì ( ) ,f k k k= ∀ ∈ℤ , thử lại thấy thỏa.

Vậy tất cả hàm số cần tìm là ( ) ,f k k k= ∀ ∈ℤ ; ( ) ,f k k k= − ∀ ∈ℤ và ( ) 0,f k k= ∀ ∈ℤ .

Page 105: Final Version 1989 to 2010 VN TST

16

Bài 4. Chứng minh rằng: 3 3 3

3 3 3

3

( ) ( ) ( ) 8

a b c

a b b c c a+ + ≥

+ + +

trong đó , ,a b c là các số thực dương.

*Trước hết, ta sẽ chứng minh bổ đề sau: “Nếu , , ,a b c d là các số thực dương có tích bằng 1 thì:

2 2 2 2

1 1 1 11

(1 ) (1 ) (1 ) (1 )a b c d+ + + ≥

+ + + +.”

Thật vậy:

Ta thấy với hai số thực dương tùy ý thì: 2 2

1 1 2

(1 ) (1 ) 1x y xy+ ≥

+ + + (*)

2 22 2 2

2

2 2 2

2 2 3 3 2 2

2 2 2 2 2 2

( 1) ( 1) 1(*) ( 1) ( 1) (1 ) ( 1)

( 1) 1

( 2 2 2)(1 ) ( ) 2( ) 1

( 2 2 2) ( 2 2 2 )

( 2 2 2 ) (2 2 2 ) 1

1

x yx y xy xy x y

xy x y xy

x y x y xy xy x y xy x y

x y x y x y xy x y xy xy

x y x y x y xy xy xy x y

x

+ + + ⇔ ≥ ⇔ + + + + ≥ + + + + + + +

⇔ + + + + + ≥ + + + + + +

⇔ + + + + + + + + + ≥

≥ + + + + + + + + +

⇔ + 2 2 2 2 2 2( ) 2 ( ) (1 ) 0y x y xy x y xy x y xy+ ≥ + ⇔ − + − ≥

Do đó:

2 2 2 2

1 1 1 1 1 1 2 21

(1 ) (1 ) (1 ) (1 ) 1 1 1 2

ab cd ab cd

a b c d ab cd ab cd abcd ab cd

+ + + ++ + + ≥ + = = =

+ + + + + + + + + + +.

Do đó bổ đề được chứng minh. Đẳng thức xảy ra khi và chỉ khi 1a b c d= = = = . Trong bổ đề trên, thay , , , 1a x b y c z d= = = = , ta có kết quả sau:

Với x, y, z là các số thực dương và 1xyz = thì:

2 2 2

1 1 1 3

(1 ) (1 ) (1 ) 4x y z+ + ≥

+ + +. Đẳng thức xảy ra khi và chỉ khi 1x y z= = = .

*Trở lại bài toán đã cho:

Đặt , , , , 0; 1b c a

x y z x y z xyza b c

= = = ⇒ > = .

BĐT đã cho ban đầu tương đương với:

3 3 33 3 3

1 1 1 3 1 1 1 3

8 (1 ) (1 ) (1 ) 8(1 ) (1 ) (1 )b c a x y za b c

+ + ≥ ⇔ + + ≥+ + ++ + +

.

Theo BĐT Cauchy cho các số dương, ta có:

Page 106: Final Version 1989 to 2010 VN TST

17

33 3 6 2 3 2

1 1 1 1 3 1 1 3 1 13 . .

(1 ) (1 ) 8 8.(1 ) 2 (1 ) (1 ) 4 (1 ) 16x x x x x x+ + ≥ = ⇔ ≥ −

+ + + + + +.

Hoàn toàn tương tự:

3 2

1 3 1 1.

(1 ) 4 (1 ) 16y y≥ −

+ +,

3 2

1 3 1 1.

(1 ) 4 (1 ) 16z z≥ −

+ +.

Cộng từng vế các BĐT này lại, ta được:

3 3 3 2 2 2

1 1 1 3 1 1 1 3.

(1 ) (1 ) (1 ) 4 (1 ) (1 ) (1 ) 16x y z x x x

+ + ≥ + + − + + + + + +

.

Ta cần chứng minh:

2 2 2

3 1 1 1 3 3.

4 (1 ) (1 ) (1 ) 16 8x x x

+ + − ≥ + + +

2 2 2

1 1 1 3

(1 ) (1 ) (1 ) 4x x x⇔ + + ≥

+ + +

với x, y, z thỏa mãn các điều kiện đã nêu. (**) Theo bổ đề trên thì (**) đúng. Vậy ta có đpcm. Dấu đẳng thức xảy ra khi và chỉ khi 1x y z a b c= = = ⇔ = = .

Page 107: Final Version 1989 to 2010 VN TST

18

Bài 5. Cho số nguyên tố ( 3)p p > . Tính:

a.

12 22

1

22

p

k

k kS

p p

=

= −

∑ nếu 1 (mod 4)p ≡ .

b.

122

1

p

k

kP

p

=

=

∑ nếu 1 (mod8)p ≡ .

*Trước hết, ta sẽ chứng minh hai bổ đề sau:

(1) Bổ đề 1: Với p là số nguyên tố thỏa 1 (mod 4)p ≡ thì mỗi số tự nhiên a với: 1

12

pa

−≤ ≤

sẽ tồn tại duy nhất số tự nhiên b thỏa 1

12

pb p

+≤ ≤ − và: 2 2 0(mod )a b p+ ≡ .

*Chứng minh: Theo định lí Wilson: ( 1)! 1(mod )p p− ≡ − .

Với mỗi 1

1,2,3,...,2

pk

−= , ta thấy: 2(mod ) ( ) (mod )p k k p k p k k p− ≡ − ⇒ − ≡ − .

Kết hợp với giả thiết 1

1(mod 4) 22

pp

−≡ ⇒ ⋮ , ta được:

2 21

2 1 11 ( 1)! ( 1) . ! ! (mod )

2 2

p p pp p

− − − − ≡ − ≡ − ≡

. Đặt 21! 1(mod )

2

ppϕ ϕ

− = ⇒ ≡ −

.

Với mỗi 1

12

pa

−≤ ≤ , ta chọn

11

2

pb p

+≤ ≤ − thỏa 2 2 2. (mod )b a pϕ≡ , dễ thấy b tồn tại và duy

nhất. Khi đó: 2 2 2 2(1 ) 0(mod )a b a pϕ+ ≡ + ≡ . Bổ đề được chứng minh.

(2) Bổ đề 2:

Với x là số thực bất kì thì [ ] [ ]2 2x x− bằng 1 nếu 1

{ } 12

x≤ < và bằng 0 nếu 1

0 { }2

x≤ < .

*Chứng minh: Ta có: [ ] { }x x x= + . Suy ra:

[ ] [ ] [ ] [ ] [ ] [ ] [ ]2 2 2[ ] 2{ } 2 [ ] { } 2{ } 2 { } 2{ }x x x x x x x x x− = + − + = − = . Do đó:

-Nếu 1

{ } 12

x≤ < thì [ ] [ ] [ ]1 2{ } 2 2{ } 1 2 2 1x x x x≤ < ⇒ = ⇒ − = .

- Nếu 1

0 { }2

x≤ < thì [ ] [ ] [ ]0 2{ } 1 2{ } 0 2 2 0x x x x≤ < ⇒ = ⇒ − = .

Bổ đề được chứng minh. *Trở lại bài toán:

Page 108: Final Version 1989 to 2010 VN TST

19

1. Ta thấy tổng đã cho là:

12 22

1

22

p

k

k kS

p p

=

= −

∑ có đúng

1

2

p −số hạng.

Theo bổ đề 2 thì mỗi số hạng trong tổng đó nhận hai giá trị là 0 hoặc 1. (1)

Theo bổ đề 1 thì với mỗi số tự nhiên a thỏa 1

12

pa

−≤ ≤ thì tồn tại duy nhất số tự nhiên b thỏa

11

2

pb p

+≤ ≤ − sao cho 2 2 2 20(mod ) ( ) 0(mod )a b p a p b p+ ≡ ⇒ + − ≡ ; do đó, tồn tại duy nhất

số tự nhiên 'a thỏa 1

1 '2

pa

−≤ ≤ sao cho 2 2' 0(mod )a a p+ ≡ .

Gọi ,x y lần lượt là số các số dư của phép chia 2k cho p (1

12

pk

−≤ ≤ ) có giá trị lớn hơn

1

2

p −

và nhỏ hơn 1

2

p −. Theo nhận xét trên thì x y= , hơn nữa

1 1

2 4

p px y x y

− −+ = ⇒ = = . (2)

Từ (1) và (2), ta có: 1

.1 .04

pS x y

−= + = .

Do đó, tổng cần tìm là 1

4

p −.

2. Do 1 (mod8)p ≡ nên tồn tại a sao cho 2 2(mod )a p≡ .

(ta cũng thấy rằng 1 (mod8) 1 (mod 4)p p≡ ⇒ ≡ ).

Ta có:

1 1 1 12 2 2 2 2 2 22 2 2 2

1 1 1 1

2 2 22

p p p p

k k k k

k k k k k k kP S

p p p p p p p

− − − −

= = = =

= = − − − = − −

∑ ∑ ∑ ∑ .

Ta cần tính: 1 1 1 1 1

2 2 2 2 2 2 2 2 22 2 2 2 2

1 1 1 1 1

2 2 2 2p p p p p

k k k k k

k k k k k k k k k

p p p p p p p p p

− − − − −

= = = = =

− = − − − = − −

∑ ∑ ∑ ∑ ∑ ,

trong đó 1 (mod8)p ≡ .

Theo nhận xét trên thì tập hợp các số dư khi chia 2 1,1

2

pk k

−≤ ≤ cho p trùng với tập hợp các số

dư khi chia 2 12 ,1

2

pk k

−≤ ≤ cho p, tức là:

12 22

1

20

p

k

k k

p p

=

− =

∑ , suy ra:

1 12 2 2 22 2

1 1

2 1

24

p p

k k

k k k p

p p p

− −

= =

−− = =

∑ ∑ .

Vậy 2 1 1 ( 1)( 5)

24 4 24

p p p pP

− − − −= − = .

Page 109: Final Version 1989 to 2010 VN TST

20

Bài 6. Một số nguyên dương được gọi là “số kim cương 2005” nếu trong biểu diễn

thập phân của nó có 2005 số 9 đứng cạnh nhau liên tiếp. Dãy ( ) , 1, 2,3,...na n = là dãy tăng

ngặt các số nguyên dương thỏa mãn na nC< (C là hằng số thực dương nào đó).

Chứng minh rằng dãy số ( ) , 1, 2,3,...na n = chứa vô hạn “số kim cương 2005”.

Trước hết, ta sẽ chứng minh các bổ đề sau:

(1) 1

1lim

n

i n=

= +∞∑ .

(2) Nếu trong hệ cơ số m ( , 1)m m∈ >ℕ : dãy số ( )na tăng và trong dãy đó không có số

hạng nào có chứa chữ số 1m− thì tổng sau 1

1n

i ia=∑ hội tụ khi n tiến tới vô cực.

*Chứng minh bổ đề (1): Ta cần chứng minh BĐT: ln( 1), 0x x x> + ∀ > . Thật vậy:

Xét hàm số: 1

( ) ln( 1), 0 ( ) 1 0, 01 1

xf x x x x f x x

x x′= − + > ⇒ = − = > ∀ >

+ +.

Do đó, hàm số ( )f x đồng biến trên (0; )+∞ . Suy ra: ( ) (0) 0 ln( 1), 0f x f x x x> = ⇒ > + ∀ > .

Trong BĐT này, thay x bởi 1

0x

> , ta cũng có:

1 1 1 1 1ln( 1) ln( ) ln( 1) ln , 0

xx x x

x x x x x

+> + ⇔ > ⇔ > + − ∀ > . Áp dụng vào tổng cần chứng minh:

[ ]1 1

1ln( 1) ln( ) ln( 1) ln1 ln( 1)

n n

i i

n n n nn= =

> + − = + − = +∑ ∑ , mà [ ]lim ln( 1)n+ = +∞ nên:

1

1lim

n

i n=

= +∞∑ . Bổ đề được chứng minh.

*Chứng minh bổ đề (2):

Đặt 1

ksn

= ∑ là tổng các số tự nhiên có chứa k chữ số viết trong hệ cơ số m và không có

chứa chữ số 1m− nào.

Giả sử một số hạng có k chữ số nào đó có dạng: 1 2 1... k kb b b b− , chữ số thứ 1 phải khác 0 và

khác 1m− nên có 2m − cách chọn, các chữ số còn lại phải khác 1m− nên có 1m− cách chọn.

Do đó, có đúng 1( 2).( 1)km m −− − số có k chữ số mà trong biểu diễn trong hệ cơ số m không có

chứa chữ số 1m− , mà mỗi số trong đó đều lớn hơn 1km − nên tổng nghịch đảo tương ứng của

chúng sẽ bé hơn 1

1

( 2).( 1)k

k

m m

m

− −.

Page 110: Final Version 1989 to 2010 VN TST

21

Hơn nữa: 1

ksn

= ∑ là tổng các số hạng có chứa k chữ số trong hệ số m và không có chứa

chữ số 1m− nào nên nó không vượt quá tổng của tất cả các số tự nhiên có cùng dạng đó mà ta

vừa đánh giá được, suy ra: 1

1

( 2).( 1)k

k k

m ms

m

− −< .

Do đó: 1

11

1 1 1 1

1 ( 2).( 1) 1 2lim lim lim lim ( 2).( ) ( 2)

11

kn n n nk

k ki k k ki

m m m ms m m m

ma m mm

−−

−= = = =

− − − −= < = − = = −

−−

∑ ∑ ∑ ∑ .

Tức là tổng này hội tụ khi n tiến tới vô cực. Bổ đề (2) được chứng minh. *Trở lại bài toán:

Đặt 200510 1m m= ⇒ − là số tự nhiên có chứa đúng 2005 số 9 liên tiếp khi viết trong hệ thập phân. Ta cần chứng minh trong dãy đã cho, có vô số số hạng chứa chữ số 1m− . Giả sử trong dãy này không có chứa số hạng nào có chữ số 1m− . Khi đó, theo bổ đề (2)

ở trên: 1

1lim

n

i ia=∑ là hữu hạn.

Hơn nữa, theo giả thiết: ,na nC n< ∀ nên 1 1 1

1 1 1 1lim lim .lim

n n n

i i iia nC C n= = =

> =∑ ∑ ∑ . Theo bổ

đề (1), giới hạn này tiến tới vô cực. Hai điều này mâu thuẫn với nhau chứng tỏ điều giả sử ở trên

là sai, tức là dãy đã cho có ít nhất một số hạng chứa chữ số 1m− , giả sử đó là: 0na .

Ta lại xét dãy con của dãy ban đầu: 0 0 01 2 3, , ,...n n na a a+ + +

Dãy này có đầy đủ tính chất của dãy đã cho nên cũng chứa ít nhất một số hạng có chứa chữ số

1m− khác với số 0na ở trên (do đây là dãy tăng).

Lập luận tương tự như thế, dãy con này có thêm một số hạng có chứa chữ số 1m− . Từ đó suy ra dãy đã cho có vô số số hạng chứa chữ số 1m− .

Vậy dãy số ( ) , 1, 2,3,...na n = chứa vô hạn “số kim cương 2005”. Đây chính là đpcm.

Page 111: Final Version 1989 to 2010 VN TST

22

LỜI GIẢI ĐỀ THI CHỌN ĐỘI TUYỂN QUỐC GIA

DỰ THI IMO 2006

Bài 1. Cho tam giác ABC có H là trực tâm. Đường phân giác ngoài của góc BHC cắt các cạnh AB, AC lần lượt tại D và E. Đường phân giác trong của góc BAC cắt đường tròn ngoại tiếp tam giác ADE tại điểm K. Chứng minh rằng đường thẳng HK đi qua trung điểm của đoạn BC. Trước hết ta sẽ chứng minh ADE∆ cân tại A.

Thật vậy: Vì HD là phân giác góc ngoài của �BHC nên:

� � � � � �01 1 1( ) (90 ) (90 )

2 2 2DHB HBC HCB ABC ACB BAC = + = − + − = .

Do đó: � � � � � �0 01 190 90

2 2ADE DBH DHB BAC BAC BAC= + = − + = − .

Tương tự, ta cũng có: � �0 190

2AED BAC= − , suy ra: � �ADE AED= , tức là tam giác ADE cân tại A.

Mặt khác AK là phân giác �DAE nên cũng là trung trực của đoạn DE, do đó AK chính là đường kính của đường tròn ngoại tiếp ADE∆ . Từ đó, ta có: KD AB⊥ , tương tự ta cũng

có: KE AC⊥ . Gọi P là giao điểm của KD và HB, Q là giao điểm của KE và HC. Ta có: , KP AB QH AB⊥ ⊥ ⇒ KP // QH.

Tương tự, ta cũng có: KQ // PH. Suy ra: KPHQ là hình bình hành, tức là HK đi qua trung điểm của PQ. Gọi BB’, CC’ là các đường cao của

tam giác ABC. Theo định lí Thalès: DP // HC’ '

PB DB

PH DC⇒ = , QE // HB’

'

QC EC

QH EB⇒ = .

Theo tính chất đường phân giác: ,' ' ' '

DB HB EC HC

DC HC EB HB= = .

Vì B, C, B’, C’ cùng thuộc đường tròn đường kính BC nên theo tính chất phương tích:

. ' . '' '

HB HCHB HB HC HC

HC HB= ⇒ = . Từ các điều này, ta được:

PB QC

PH QH= ⇒PQ // BC.

Vì HK đi qua trung điểm của PQ nên cũng đi qua trung điểm của BC. Ta có đpcm.

C'

B'

P

Q

K

D

E

H

A

B C

Page 112: Final Version 1989 to 2010 VN TST

23

Bài 2. Hãy tìm tất cả các cặp số tự nhiên (n ; k) với n là số nguyên không âm và k là số

nguyên lớn hơn 1 sao cho số : 2006 2 517 4.17 7.19n n nA = + + có thể phân tích được thành tích của k số nguyên dương liên tiếp. Trước hết ta thấy rằng tích của 4 số tự nhiên liên tiếp phải chia hết cho 8 vì trong 4 số đó có 1 số chia hết cho 4 và một số chia 4 dư 2.

Từ 2006 2 517 4.17 7.19n n nA = + + , suy ra : - Nếu n là số chẵn, ta có :

2006 2 5 2 1017 1 (mod8),4.17 4.1 (mod8),7.19 7.3 7.3 7 (mod8)n n n n≡ ≡ ≡ ≡ ≡

Suy ra : 12 4(mod8)A ≡ ≡ , tức là A không chia hết cho 8.

- Nếu n là số lẻ, cũng tương tự : 2006 2 2 517 1 (mod8), 4.17 4.1 (mod8),7.19 7.3 7.3 5(mod8)n n n≡ ≡ ≡ ≡ ≡

Suy ra : 10 2(mod8)A ≡ ≡ , tức là A cũng không chia hết cho 8.

Tức là trong mọi trường hợp luôn có A không chia hết cho 8. Suy ra nếu k thỏa mãn đề bài thì 4 {2,3}k k< ⇒ ∈ .

Xét từng trường hợp : - Nếu 2 k = : tồn tại x tự nhiên sao cho ( 1)A x x= + .

+ Nếu n = 0 thì A = 12, x = 3, thỏa mãn đề bài.

+ Nếu n > 0 thì rõ ràng 1003 2 517 4.17 7.19n n n> + . Ta thấy : 2006 2 5 2006( 1) 17 4.17 7.19 17n n n nA x x= + = + + > , suy ra 100317 nx > nhưng

2006 1003( 1) 17 17n nx x A+ > + > , mâu thuẫn.

Do đó, trong trường hợp này không có n thỏa mãn đề bài. - Nếu 3 k = : tồn tại x tự nhiên sao cho ( 1)( 1), 1A x x x x= − + ≥ ; dễ thấy x phải là số chẵn (vì

nếu ngược lại thì A chia hết cho 8, mâu thuẫn). Ta thấy :

12.( 1) 2.( 1) (mod 5)n nA ≡ − ≡ − trong khi 2( 1)( 1) ( 1) 0, 1(mod5)x x x x x− + = − ≡ ± , mâu thuẫn.

Do đó, trong trường hợp này không có n thỏa mãn đề bài. Vậy tất cả các cặp số thỏa mãn đề bài là ( ; ) (0; 2)n k = .

Page 113: Final Version 1989 to 2010 VN TST

24

Bài 3. Trong không gian cho 2006 điểm mà trong đó không có 4 điểm nào đồng phẳng. Người ta nối tất cả các điểm đó lại bởi các đoạn thẳng. Số tự nhiên m gọi là số tốt nếu ta có thể gán cho mỗi đoạn thẳng trong các đoạn thẳng đã nối bởi một số tự nhiên không vượt quá m sao cho mỗi tam giác tạo bởi ba điểm bất kì trong số các điểm đó đều có hai cạnh được gán bởi hai số bằng nhau và cạnh còn lại gán bởi số lớn hơn hai số đó. Tìm số tốt có giá trị nhỏ nhất. Do trong các điểm đã cho không có bốn điểm nào đồng phẳng nên ba điểm bất kì trong chung luôn tạo thành một tam giác. Gọi S(n) là giá trị nhỏ nhất của số tốt ứng với n điểm trong không gian (n là số tự nhiên), ta sẽ xác định giá trị của S(2006). Ta chỉ xét các giá trị 4n ≥ .

- Với n = 4 thì thử trực tiếp, ta thấy S(4) = 2. Bởi vì S(4) = 1 không thỏa mãn nên (4) 2S ≥ , ta sẽ chỉ ra rằng S(4) = 2 thỏa mãn. Cụ thể

ta có thể gán các đoạn thẳng như sau : gán 4 đoạn bất kì bởi số 1 và 2 đoạn còn lại bởi số 2, rõ ràng các tam giác tạo thành đều thỏa mãn đề bài.

- Với một giá trị n > 4 bất kì, ta sẽ chứng minh rằng :

1

( ) 12

nS n S

+ ≥ + .

Gọi a là số nhỏ nhất được gán cho các đoạn thẳng trong trường hợp có n điểm. Trong trường hợp tối thiểu, không mất tính tổng quát, ta giả sử rằng a = 1, ta gọi hai đầu mút của đoạn thẳng nào đó được gán số 1 là X và Y. Trong n – 2 điểm còn lại, nếu có một điểm được nối với X và Y bởi một đoạn thẳng gán bởi số 1 thì điểm đó cùng với X và Y sẽ tạo thành một tam giác đều không thỏa mãn đề bài. Do đó, nếu gọi A là tập hợp tất cả các điểm nối với X bởi một đoạn thẳng gán số 1 (có tính luôn điểm Y) và B là tập hợp tất cả các điểm nối với Y bởi một đoạn thẳng gán số 1 (có tính luôn

điểm X) thì giữa A và B không có phần tử nào chung hay A B n+ = .

*Ta có các nhận xét sau : - Nếu lấy một điểm bất kì trong tập A và một điểm bất kì trong B thì hai điểm đó cũng phải được nối bởi đoạn thẳng gán số 1 vì nếu không thì hai điểm đó sẽ cùng với X sẽ tạo thành một tam giác không thỏa mãn đề bài (tam giác đó hoặc không có hai số được gán trên hai cạnh bằng nhau hoặc có hai cạnh bằng nhau nhưng cạnh còn lại gán số 1 nhỏ hơn). - Hai điểm bất kì trong A được nối với nhau bởi một đoạn thẳng gán số lớn hơn 1 bởi nếu không thì khi chọn thêm một điểm trong B, ta sẽ có một tam giác không thỏa mãn đề bài (tam giác đó đều). Tương tự với tập hợp B. Tức là trong các tập A và B đều có chứa các số lớn hơn 1.

Tiếp theo, ta lại thấy trong mỗi tập A, B như vậy đều cần thêm ( ), ( )S A S B số nữa để

gán cho các đoạn thẳng. Giả sử A B≥ thì 1 1

12 2

n nA

− + ≥ + = .

2

2 1

1

1

1

Page 114: Final Version 1989 to 2010 VN TST

25

Ta hoàn toàn có thể gán các số ở tập A trùng với các số ở tập B nên các số cần có thêm

nữa là 1

2

nS +

, tính thêm số 1 nhỏ nhất đã được gán cho đoạn XY ban đầu, ta được:

1( ) 1

2

nS n S

+ ≥ + .

Từ đó, áp dụng liên tiếp kết quả này, ta có: (chú ý rằng S(4) = 2). (2006) 1 (1003) 2 (502) ... 9 (4) 11S S S A≥ + ≥ + ≥ ≥ + = .

Tiếp theo, ta sẽ chứng minh rằng giá trị 11 này thỏa mãn đề bài. * Thật vậy : Ta xây dựng cách gán các điểm từ thấp đến cao bằng cách ghép các bộ điểm ít hơn lại. Cụ thể như sau : - Đầu tiên ta xây dựng cho bộ 4 điểm. Cách gán tương tự như ở trên, nhưng trong trường hợp

này gán 4 đoạn bởi số 11 và 2 đoạn bởi số 10. - Ghép 2 bộ này lại và tách ra từ một trong hai bộ đó ra 2 điểm, gán cho đoạn thẳng nối 2 điểm

đó bởi số 10, ta đã có tất cả 8 điểm. - Tiếp tục ghép tương tự như vậy theo thứ tự như sau :

4 8 16 32 63 126 251 502 1003 2006→ → → → → → → → → (Các trường hợp từ 32 đến 63 hoặc tương tự ta phải bỏ đi 1 điểm nào đó ở một trong hai bộ ra ngoài). Mỗi lần ghép hai bộ điểm lại thì số gán trên đoạn được tách ra lại giảm đi 1 đơn vị, đến khi ghép được 2006 điểm thì số đó chính là 1.

Dễ thấy cách gán số cho các đoạn thẳng này thỏa mãn đề bài. Vậy giá trị nhỏ nhất của số tốt cần tìm là 11.

Page 115: Final Version 1989 to 2010 VN TST

26

Bài 4. Chứng minh rằng với mọi số thực [ ], , 1;2x y z∈ , ta luôn có bất đẳng thức sau :

1 1 1( )( ) 6( )

x y zx y z

x y z y z z x x y+ + + + ≥ + +

+ + +.

Hỏi đẳng thức xảy ra khi và chỉ khi nào ? Trước hết ta thấy rằng :

21 1 1 ( )( )( ) 9

x yx y z

x y z xy

−+ + + + − =∑ ,

2( )6( ) 9

( )( )

x y z x y

y z z x x y y z z x

−+ + − =

+ + + + +∑ .

Ta cần chứng minh : 2 21 1 1 ( ) 3( )

( )( ) 6( )( )( )

x y z x y x yx y z

x y z y z z x x y xy y z z x

− −+ + + + ≥ + + ⇔ ≥

+ + + + +∑ ∑

với mọi số thực x, y, z thuộc đoạn [1 ; 2].

Đặt 1 3

( )( )xSyz x y x z

= −+ +

, 1 3

( )( )ySzx y x y z

= −+ +

, 1 3

( )( )zSxy z x z y

= −+ +

.

Bất đẳng thức đã cho viết dưới dạng tương đương là: 2 2 2( ) ( ) ( ) 0x y zS y z S z x S x y− + − + − ≥ .

Không mất tính tổng quát, ta giả sử 2 1x y z≥ ≥ ≥ ≥ .

Ta sẽ chứng minh rằng , 0x yS S ≥ . Thật vậy:

20 2 0xS x xy xz yz≥ ⇔ + + − ≥ , đúng. 20 2 ( ) ( ) 0yS y yx yz zx x y z z z y x≥ ⇔ + + − ≥ − + + − ≥ (do [ ], , 1;2x y z∈ nên 0y z x+ − ≥ ).

- Nếu 0zS ≥ , ta có đpcm.

- Nếu 0zS < , ta chứng minh được rằng 2 0, 2 0x z y zS S S S+ ≥ + ≥ .

Khi đó dễ dàng thấy rằng vì: 2 2 2( ) 2 ( ) ( )x y y z z x − ≤ − + − và 0zS < nên

2 2 2 2 2( ) ( ) ( ) ( 2 )( ) ( 2 )( ) 0x y z x z y zS y z S z x S x y S S y z S S z x− + − + − ≥ + − + + − ≥

Vậy trong mọi trường hợp, ta luôn có đpcm. Đẳng thức xảy ra khi x = y = z hoặc y = z = 1, x = 2 và các hoán vị của chúng.

Page 116: Final Version 1989 to 2010 VN TST

27

Bài 5. Cho tam giác ABC là tam giác nhọn, không cân, nội tiếp trong đường tròn tâm O bán kính R. Một đường thẳng d thay đổi sao cho d luôn vuông góc với OA và luôn cắt các tia AB, AC. Gọi M, N lần lượt là giao điểm của đường thẳng d và các tia AB, AC. Giả sử các đường thẳng BN và CN cắt nhau tại K; giả sử đường thẳng AK cắt đường thẳng BC. 1. Gọi P là giao của đường thẳng AK và đường thẳng BC. Chứng minh rằng đường tròn ngoại tiếp của tam giác MNP luôn đi qua một điểm cố định khi d thay đổi. 2. Gọi H là trực tâm của tam giác AMN. Đặt BC = a và l là khoảng cách từ điểm A đến đường thẳng HK. Chứng minh rằng đường thẳng HK luôn đi qua trực tâm của tam giác ABC.

Từ đó suy ra: 2 24l R a≤ − . Đẳng thức xảy ra khi và chỉ khi nào?

1. Không mất tính tổng quát, giả sử AB < AC (trường hợp còn lại hoàn toàn tương tự).

Do tam giác ABC không cân nên AO không vuông góc với BC và MN không song song với BC,

do đó MN phải cắt đường thẳng BC tại một điểm, giả sử là Q; gọi I là trung điểm BC.

Theo định lí Menelaus cho ba điểm Q, M, N thẳng hàng: . . 1NA MB QB

NC MA QC= .

Mặt khác, theo định lí Céva cho các đoạn AP, BN, CM đồng quy, ta có: . . 1NA MB PB

NC MA PC= .

Từ đó, suy ra: PB QB

PC QC= hay Q, B, P, C là một hàng điểm điều hòa, suy ra: 2 2.IP IQ IB IC= =

Do I là trung điểm BC nên 2 2 2 2OI BC QI BI OQ OB⊥ ⇒ − = − , do đó:

2 2 2 2 2. . .QI QP QI QI PI QI IB OQ OB QB QC= − = − = − =

(do theo tính chất phương tích của Q đối với (O) thì 2 2 2 2 .OQ OB OQ R QB QC− = − = ).

Mà tứ giác BMNC cũng nội tiếp vì có � � �NCB xAB AMN= = (với Ax là tia tiếp tuyến của (O)).

Suy ra . .QM QN QB QC= .

Q IP

K

N

M

O

A

BC

Page 117: Final Version 1989 to 2010 VN TST

28

Từ đó suy ra . .QM QN QP QI= , suy ra tứ giác MNIP nội tiếp hay đường tròn ngoại tiếp tam giác

MNP luôn đi qua điểm I cố định. Ta có đpcm.

2. Gọi BD, CE là hai đường cao của tam giác ABC, L là trực tâm của tam giác ABC; gọi MF,

NG là hai đường cao của tam giác AMN, H là trực tâm của tam giác AMN. Ta cần chứng

minh rằng H, K, L thẳng hàng.

Xét đường tròn (O1) đường kính BN và (O2) đường kính CM.

Ta thấy: KM.KC = KB. KN nên K có cùng phương tích đến (O1), (O2), tức là K thuộc trục đẳng

phương của hai đường tròn này.

Đồng thời, dễ thấy rằng các điểm D, G thuộc (O1) và M, F thuộc (O2).

Do H, L là trực tâm của tam giác ABC và AMN nên LB. LD = LC. LE, HN. HG = HE. HM; tức

là H, L cùng thuộc trục đẳng phương của hai đường tròn (O1), (O2).

Từ đó suy ra H, K, L cùng thuộc trục đẳng phương của (O1), (O2) nên chúng thẳng hàng.

Từ đó suy ra l AL≤ .

Mặt khác do tam giác ABC nhọn nên 2

2 2 22 44

BCAL OI R R a= = − = − .

Do đó 2 24AL l R a= ≤ − . Đây chính là đpcm.

Đến đây, ta sẽ tìm vị trí của d sao cho đẳng thức xảy ra.

Giả sử d cắt AB, AC tại M và N thỏa mãn .AN AM

k MN k BCAB AC

= = ⇒ = .

FG

H

L

E

D

Q IP

K

N

MO

A

BC

Page 118: Final Version 1989 to 2010 VN TST

29

Gọi R, S lần lượt là trung điểm của BN và CM; suy ra R, S cũng chính là tâm của hai

đường tròn (O1), (O2).

Ta thấy khi đẳng thức xảy ra thì AL vuông góc với trục đẳng phương của (O1), (O2), tức

là AL song song với đường nối tâm RS của hai đường tròn này, mà AL vuông góc với BC nên

RS phải vuông góc với BC.

Ta có: 2RS BC NM= +���� ���� �����

, mà . 0 ( ). 0RS BC BC NM BC= ⇒ + =���� ���� ���� ����� ����

. Do góc tạo bởi MN và BC

chính là � � � � �MQB ANM ACB ABC ACB= − = − nên từ đẳng thức trên suy ra:

2 1. . .cos( )

cos( )BC BC MN BC kBC B C k

B C= = − ⇒ =

���� �����.

Vậy đẳng thức xảy ra khi và chỉ khi 1

cos( )k

B C=

−, tức là đường thẳng d cắt AB tại M,

AC tại N sao cho 1

cos( )

AN AM

AB AC B C= =

−.

Page 119: Final Version 1989 to 2010 VN TST

30

Bài 6. Cho dãy số thực ( )na được xác định bởi:

0 1

1 11, ( )

2 3n nn

a a aa+= = + với mọi n = 1, 2, 3, …

Chứng minh rằng với mọi số nguyên n, số2

3

3 1nn

Aa

=−

là một số chính phương và nó có ít

nhất n ước nguyên tố phân biệt.

Trước hết, ta sẽ chứng minh rằng 13 4 ( 3)n n nA A A+ = + với mọi n nguyên dương. (*)

Thật vậy:

Ta có: 2

2 2

933 3

3 1 3 1n

nn n

aA

a a+ = + =

− − nên

( )

2 2

22 2

9 1084 ( 3) 36 .

3 1 3 1

n nn n n

n n

a aA A A

a a+ = =

− −.

Mặt khác:

( )

2

1 22 2212

1089 93

1 13 1 3 13. ( ) 12 3

nn

n nnn

aA

a aaa

++

= = =− −+ −

.

Do đó: (*) được chứng minh, tức là 13 4 ( 3)n n nA A A+ = + với mọi n nguyên dương.

Hơn nữa, ta tính được: 9nA = nên dễ dàng thấy rằng nA chia hết cho 3 với mọi n.

Tiếp theo, ta sẽ chứng minh bằng quy nạp rằng: 13

nA+ là số chính phương với mọi n (**).

- Với n =1, 1 1 43

A+ = là một số chính phương nên (**) đúng.

- Giả sử với n = k, (**) cũng đúng, tức là tồn tại số tự nhiên p sao cho 213

kAp+ = .

Ta có: 2 2 2 21 4 ( 3)1 1 4. .(1 ) 1 4 ( 1) 1 (2 1)

3 9 3 3k k k k kA A A A A

p p p+ ++ = + = + + = − + = − cũng là

một số chính phương. Suy ra (**) cũng đúng với 1n k= + . Do đó, (**) được chứng minh.

Từ 1 13 4 ( 3) 4 .( 1)3

nn n n n n

AA A A A A+ += + ⇒ = + và (**), ta cũng dễ dàng chứng minh được bằng

quy nạp rằng nA là số chính phương với mọi n.

Cuối cùng, ta sẽ chứng minh rằng nA có ít nhất n ước nguyên tố đôi một khác nhau cũng bằng

phương pháp quy nạp. (***) - Với n = 1, rõ ràng (***) đúng.

- Giả sử (***) đúng với n = k, tức là kA có ít nhất k ước nguyên tố khác nhau.

Ta xét hai trường hợp:

Page 120: Final Version 1989 to 2010 VN TST

31

+ Nếu k có ít nhất k + 1 ước nguyên tố khác nhau thì rõ ràng (***) đúng.

+ Nếu k có đúng k ước nguyên tố đôi một khác nhau, giả sử đó là 1 2 3, , ,.., kp p p p .

Khi đó: ( , ), 1,k iA p i k= là 1 hoặc 3.

Giả sử 1kA + chỉ có đúng k ước nguyên tố đôi một khác nhau là các giá trị ở trên thì cần phải có *3 3 , , 2m

kA m m+ = ∈ ≥ℕ .

Nhưng khi đó thì 3(mo d 9)kA ≡ − không phải là số chính phương, mâu thuẫn.

Từ đó dẫn đến 1kA + phải có một ước nguyên tố nào khác k ước đã có, tức là có ít nhất k + 1 ước

nguyên tố đôi một khác nhau hay (***) đúng với 1n k= + . Do đó (***) được chứng minh.

Vậy với mọi n nguyên dương, số2

3

3 1nn

Aa

=−

là một số chính phương và nó có ít nhất n

ước nguyên tố phân biệt, bài toán được giải quyết hoàn toàn.

Page 121: Final Version 1989 to 2010 VN TST

32

LỜI GIẢI ĐỀ THI CHỌN ĐỘI TUYỂN QUỐC GIA

DỰ THI IMO 2007

Bài 1.

Cho hai tập hợp A,B là tập hợp các số nguyên dương thỏa mãn A B n= = (với n là số

nguyên dương) và có tổng các phần tử bằng nhau. Xét bảng ô vuông n n× . Chứng minh rằng ta có thể điền vào mỗi ô vuông của bảng một số nguyên không âm thỏa mãn đồng thời các điều kiện: i/ Tập hợp tổng các số ở mỗi hàng là tập A. ii/ Tập hợp tổng các số ở mỗi cột là tập B.

iii/ Có ít nhất 2( 1)n k− + số 0 trong bảng với k là số các phần tử chung của A và B.

Trước hết, ta thấy rằng nếu một giá trị k sao cho tồn tại 2 phần tử bằng nhau ở mỗi tập là

k ka b t= = thì ta điền số t vào ô vuông nằm ở hàng thứ

k và cột thứ k, các ô còn lại của hàng thứ k và cột thứ k đều điền vào số 0; như thế thì tổng các số ở hàng và cột này thỏa mãn đề bài và không ảnh hưởng đến các hàng và cột khác. Do đó, không mất tính tổng quát, ta xét trường hợp A B∩ =∅ (trường hợp có các phần tử chung thì điền thêm vào các hàng và cột theo cách tương tự như trên), tức là số phần tử chung của hai tập là 0k = .

Ta sẽ chứng minh bài toán này bằng quy nạp. Gọi Τ là tập hợp các điều kiện i/, ii/, iii/ như trên (điều kiện iii/ tương ứng với trường hợp xét số nguyên dương n).

Với n = 1, bài toán hiển nhiên đúng.

Giả sử bài toán đúng với mọi số tập hợp có n – 1 phần tử. Ta sẽ chứng minh rằng với hai tập A, B có n phần tử, ta cũng có thể xây dựng một bảng n n× thỏa mãn điều kiện Τ .

Thật vậy, xét hai tập hợp 1 2 3 1 2 3{ , , ,..., }, { , , ,..., }n nA a a a a B b b b b= = trong đó:

1 2 3 ... na a a a< < < < , 1 2 3 ... nb b b b< < < < (hai tập này không có phần tử nào chung).

Giả sử 1 1a b< . Do tổng các phần tử ở hai tập bằng nhau nên tồn tại một chỉ số i thỏa mãn

1 1 1 1 1( ) 0i ia b b a a b a> > − ⇒ − − > . Ta xét hai tập hợp A*, B* như sau:

2 3 1 1 1* { , ,..., , ,..., }i i nA a a a a b a a−= − + , 2 3* { , ,..., }nB b b b= .

0 00

0

0

000

0

0

0

0

n

i

i

2

1

n21

Page 122: Final Version 1989 to 2010 VN TST

33

Hai tập hợp này có cùng số phần tử là n – 1 nên theo giả thiết quy nạp, tồn tại một bảng có kích thước ( 1) ( 1)n n− × − thỏa mãn điều kiện Τ (trong bảng này có

ít nhất 2( 2)n − số 0).

Ta thêm vào bên trái bảng một cột và bên trên bảng một hàng nữa như hình vẽ. Ở ô góc bên trái và phía trên, ta

điền số 1a , ở hàng thứ i của bảng ban đầu (hàng có tổng

các phần tử bằng 1 1ia b a− + ), ta điền số 1 1b a− ; còn tất

cả các ô còn lại của hàng và cột vừa thêm vào, ta điền vào các số 0. Khi đó, bảng này có tổng các phần tử ở mỗi

hàng là tập A và tổng các phần tử ở mỗi cột là tập B, số các số 0 ở bảng vừa lập được không nhỏ

hơn 2 2( 2) 2( 1) 1 ( 1)n n n− + − − = − và do đó nó thỏa mãn điều kiện Τ .

Do đó, bài toán cũng đúng với mọi tập hợp có n phần tử.

Theo nguyên lí quy tạp, bài toán này đúng với mọi số nguyên dương n.

Vậy ta có đpcm.

n - 1

i

2

1

n - 1321

Page 123: Final Version 1989 to 2010 VN TST

34

Bài 2.

Cho tam giác nhọn ABC với đường tròn nội tiếp I. Gọi ( )ak là đường tròn có tâm nằm

trên đường cao của góc A và tiếp xúc trong với đường tròn (I) tại 1A . Các điểm 1 1, B C xác

định tương tự .

1/ Chứng minh 1 1 1, , AA BB CC đồng qui tại P.

2/ Gọi ( ), ( ), ( )a b cJ J J lần lượt là các đường tròn đối xứng với đường tròn bàng tiếp các

góc A, B, C của tam giác ABC qua trung điểm BC, CA, AB. Chứng minh P là tâm đẳng phương của 3 đường tròn nói trên. 1/ Trước hết, ta sẽ chứng minh bổ đề sau: Cho tam giác ABC ngoại tiếp đường tròn (I) có D là tiếp điểm của đường tròn bàng tiếp góc A lên BC. Gọi M, N là giao điểm của AD với (I) (N nằm giữa A và M). Giả sử IM cắt đường cao AH tại K. Chứng minh rằng: KA = KM.

* Thật vậy: Gọi E là tiếp điểm của (I) lên BC. Giả sử IE cắt (I) tại điểm thứ hai là N’ khác E. Qua N’ vẽ đường thẳng song song với BC cắt AB và AC lần lượt tại B’ và C’. Dễ thấy tồn tại một phép vị tự biến tam giác AB’C’ thành tam giác ABC. Phép vị tự đó cũng biến tiếp điểm N’ của đường tròn bàng tiếp (I) của ∆ AB’C’ lên B’C’ thành tiếp điểm D của đường tròn bàng tiếp (J) của ∆ ABC lên BC. Suy ra A, N’, D thẳng hàng hay N’ trùng với N. Khi đó, tam giác IMN đồng dạng với ∆ KMA (do IN // AK), mà ∆ IMN cân tại I nên ∆ KAM cân tại K hay KA = KM. Ta có đpcm. Từ đây suy ra: đường tròn có tâm thuộc đường cao góc A, đi qua A và tiếp xúc với (I) tại M thì M thuộc AD.

Dễ thấy đường tròn đó là duy nhất. *Trở lại bài toán: Gọi D, E, F lần lượt là tiếp điểm của đường tròn bàng tiếp các góc A, B, C của tam giác

ABC lên các cạnh BC, CA, AB. Theo bổ đề trên, ta thấy: 1 1 1, ,A AD B CF C BE∈ ∈ ∈ .

Suy ra: 1 1 1, , AA BB CC đồng quy khi và chỉ khi AD, BE, CF đồng quy. (1)

C'B'N

K

H

M

ED

J

I

A

B C

Page 124: Final Version 1989 to 2010 VN TST

35

Mặt khác: nếu ta đặt , , ,

2

BC a CA b AB c

AB BC CAp

= = =

+ +=

thì có thể dễ dàng tính được: DB EC p c= = −DC AF p b= = −AE BF p a= = − .

Suy ra: . . 1DB EC FA

DC EA FB= ,

theo định lí Ceva đảo, ta có AD, BE, CF đồng quy. (2) Từ (1) và (2), ta có

1 1 1, , AA BB CC đồng quy.

Ta có đpcm.

2/ Gọi A’, B’ lần lượt là trung điểm của BC, CA; A2, B2, C2 lần lượt là tâm đường tròn bàng tiếp các góc A, B, C của ABC∆ .

Gọi D’, E’ lần lượt là tiếp điểm của (I) lên BC, CA. Dễ thấy D đối xứng với D’ qua trung điểm A’ của BC, A2 đối xứng với Ja qua A’ nên JaD’ // A2D, mà

2 'aA D BC J D BC⊥ ⇒ ⊥ .

Do đó: (Ja) tiếp xúc với BC tại D’. Hoàn toàn tương tự: (Jb) tiếp xúc với CA tại E’. Ta có:

' 'CD CE= nên phương tích từ C đến (Ja) và (Jb) bằng nhau, tức là C thuộc trục đẳng phương của hai đường tròn này.

C1

B1

E

C2

I

A1

P

F

D

A2

A

B C

B2

A'

B'

E'C2

Jb

B2

Ja

I P

F

A2

D'

A

B C

Page 125: Final Version 1989 to 2010 VN TST

36

Ta sẽ chứng minh rằng CP, cũng chính là CF, vuông góc với đoạn nối tâm JaJb của hai đường tròn (Ja), (Jb). Theo cách xác định các điểm Ja, Jb, ta thấy A’ là trung điểm của A2Ja, B’ là trung điểm của B2J b.

Do đó: 2 22 ' ' a bA B A B J J= +������ ������ �����

hay 2 22 ' 'a bJ J A B A B= − =����� ������ ������

2 2BA A B−���� ������

. Ta cũng có:

2 2CF CC C F= +���� ����� �����

.

Ta có: 2 2 2 2 2 2 2 2 2 2 2 2. ( )( ) . . . .a bJ J CF BA A B CC C F BACC BAC F A B CC A B C F= − + = + − − =����� ���� ���� ������ ����� ����� ���� ����� ���� ����� ������ ����� ������ �����

2 2 2 2. .BACC A B C F= −���� ����� ������ �����

(do C2F vuông góc với AB, A2B2 vuông góc với CC2). (1)

Mặt khác, ta thấy A, B, C chính là chân các đường cao của tam giác A2B2C2 nên rõ ràng:

2 2 2 2C AB C B A∆ ∆∼ , mà C2F là đường cao của 2C AB∆ , C2C là đường cao của 2 2 2C B A∆ nên:

2 2

2 2

C F C C

AB A B= 2 2 2 2. .C F A B AB CC⇒ = . Cũng từ hai tam giác 2C AB∆ , 2 2 2C B A∆ đồng dạng; ta có:

2 2 2 2( , ) ( , )BA CC A B C F=���� ����� ������ �����

. Do đó: 2 2 2 2. .BACC A B C F=���� ����� ������ �����

. (2)

Từ (1) và (2), suy ra: 2. 0a bJ J C F =����� �����

hay 2 a bC F J J⊥ .

Do đó C2F chính là trục đẳng phương của hai đường tròn (Ja), (Jb), tức là P thuộc trục đẳng phương của hai đường tròn (Ja), (Jb). Hoàn toàn tương tự, ta cũng có P thuộc trục đẳng phương của hai đường tròn (Jc), (Jb). Từ đó suy ra P chính là tâm đẳng phương của (Ja), (Jb), (Jc). Đây chính là đpcm.

Page 126: Final Version 1989 to 2010 VN TST

37

Bài 3. Cho tam giác ABC. Tìm giá trị nhỏ nhất của biểu thức sau:

2 2 2 2 2 2

2 2 2

cos cos cos cos cos cos2 2 2 2 2 2

cos cos cos2 2 2

A B B C C A

SC A B

= + +

* Trước hết, ta sẽ chứng minh bổ đề:

“Với mọi a, b, c không âm và không đồng thời bằng 0, ta có:

2 2 2

1 1 1 9

( ) ( ) ( ) 4( )a b b c c a ab bc ca+ + ≥

+ + + + +.

Dấu đẳng thức xảy ra khi a = b = c hoặc a = b, c = 0 và các hoán vị.”

* Chứng minh:

Quy đổng và khai triển BĐT trên, ta cần chứng minh rằng:

5 4 2 3 3 4 3 2 2 2 2(4 3 ) ( 2 ) 0sym sym

a b a b a b a bc a b c a b c− − + − + ≥∑ ∑ (*)

Theo BĐT Schur cho các số không âm a, b, c, ta có:

( )( ) 0sym

a a b a c− − ≥∑ , nhân vào hai vế cho số abc không âm và khai triển, ta có:

4 3 2 2 2 2( 2 ) 0sym

a bc a b c a b c− + ≥∑ . (1)

Hơn nữa, theo BĐT Cauchy, ta có:

5 4 2 3 3 5 4 2 5 3 3(4 3 ) ( ) 3( ) 0sym sym sym

a b a b a b a b a b a b a b− − = − + − ≥∑ ∑ ∑. (2)

Cộng từng vế các BĐT (1) và (2) lại, ta có BĐT (*).

Đẳng thức xảy ra trong (*) khi và chỉ khi đẳng thức xảy ra trong (1) và (2), tức là khi

a = b = c hoặc a = b, c = 0 và các hoán vị. Bổ đề được chứng minh.

* Trở lại bài toán:

Ta có: 2

2

1cos

2 tan 12

AA

=+

. Tương tự với 2 2cos ,cos2 2

B C.

Page 127: Final Version 1989 to 2010 VN TST

38

Thay các biến đổi này vào biểu thức đã cho, ta được:

2

2 2

1

( 1)( 1)

xS

y z

+=

+ +∑ , trong đó tan , tan , tan ; , , 02 2 2

A B Cx y z x y z= = = > .

(Ở đây kí hiệu x∑ là tổng đối xứng lấy theo các biến x, y, z).

Mặt khác, trong tam giác ABC, ta luôn có:

tan tan tan tan tan tan 1 12 2 2 2 2 2

A B B C C Axy yz zx+ + = ⇒ + + = .

Do đó:

22

2 2 2 2

( )1 ( )( )

( 1)( 1) ( )( ) ( )( )( )( )

x xyx x y x zS

y z y xy z xy y z y x z x z y

++ + += = = =

+ + + + + + + +∑∑ ∑ ∑∑ ∑

2

1

( )x y=

+∑ .

Áp dụng bổ đề ở trên, ta được: 9 9

4( ) 4S

xy yz zx≥ =

+ +.

Vậy giá trị nhỏ nhất của S là 9

4 đạt được khi và chỉ khi x y z= = hay ABC là tam giác đều.

Page 128: Final Version 1989 to 2010 VN TST

39

Bài 4. Tìm tất cả các hàm số liên tục :f →ℝ ℝ thỏa mãn:

2 1( ) ( )

3 9

xf x f x= + + với mọi x∈ℝ .

Ta có: 2 2 21 1 1 1 1( ) ( ) (( ) ),

3 9 6 12 6 12

xx x f x f x x+ + = + + ⇒ = + + ∀ ∈ℝ .

Đặt 1 1

6 6y x x y= + ⇔ = − . Thay vào giả thiết đã cho, ta có: 21 1

( ) ( )6 12

f y f y− = + . (*)

Xét hàm số: ( ) :g x →ℝ ℝ thỏa mãn: 1 1

( ) ( ) ( ) ( ),6 6

g x f x f x g x x+ = ⇔ − = ∀ ∈ℝ . (1)

Từ (*), ta được: 2 1( ) ( ),

4g x g x x= + ∀ ∈ℝ , rõ ràng ( )g x cũng liên tục.

Ta sẽ xác định hàm số ( )g x thỏa mãn điều kiện trên.

Ta thấy: 2 21 1, ( ) ( ) (( ) ) ( )

4 4x g x g x g x g x∀ ∈ = + = − + = −ℝ nên ( )g x là hàm số chẵn.

Ta chỉ cần xét 0x ≥ . Ta có hai trường hợp:

- Với 0

1

2x ≤ : Xét dãy số: 2

1 0 1

1, , 1

4n nu x u u n+= = + ≥ . Khi đó: 21

1( ) 0

2n n nu u u+ − = − ≥ nên

dãy ( )nu tăng. Mặt khác, bằng quy nạp, ta chứng minh được 1

,2nu n≤ ∀ , tức là dãy này bị chặn

trên. Từ đó suy ra nó có giới hạn. Gọi t là giới hạn đó thì 2 1 1

4 2t t t= + ⇔ = .

Do đó: 0

1 1( ) ( ), 0;

2 2g x g x = ∀ ∈

.

- Với 0

1

2x > : Tương tự như trên, xét dãy số 1 0 1

1, 0, 1

4n nv x v v n+= = − ≥ ≥ .

Khi đó:

2

1

1( )1 2 0

4 1

4

n

n n n n

n n

vv v v v

v v+

− −− = − − = <

− +

nên dãy đã cho là dãy giảm. Cũng bằng quy

nạp, ta chứng minh được 1

2nv > , tức là dãy đã cho bị chặn dưới, suy ra nó có giới hạn.

Gọi k là giới hạn đó thì 1 1

4 2k k k= − ⇔ = .

Do đó: 0

1 1( ) ( ), ( ; )

2 2g x g x= ∀ ∈ +∞ . Đặt

1( )2

g a= ∈ℝ .

Từ đó suy ra: ( ) , 0g x a x= ∀ ≥ hay ( ) , g x a x= ∀ ∈ℝ . (2)

Từ (1) và (2), ta có: ( ) , f x a x= ∀ ∈ℝ . Đây là tất cả hàm số cần tìm.

Page 129: Final Version 1989 to 2010 VN TST

40

Bài 5.

Cho A là tập con chứa 2007 phần tử của tập:{ }1, 2,3,..., 4013, 4014 thỏa mãn với mọi

,a b A∈ thì a không chia hết cho b. Gọi mA là phần tử nhỏ nhất của A.

Tìm giá trị nhỏ nhất của mA với A thỏa mãn các điều kiện trên.

Chia tập hợp {1, 2,3,..., 4013, 4014} thành 2007 phần 1 2 2007...P P P∪ ∪ ∪ (mỗi tập hợp

chứa ít nhất một phần tử của { }1, 2,3,..., 4013, 4014 ) thỏa mãn tập hợp aP chứa tất cả các số

nguyên dương có dạng 2 (2 1)n a − , trong đó n là một số không âm. Khi đó, tập hợp con A của

{ }1, 2,3,..., 4013, 4014 không thể chứa hai phần tử cùng thuộc một trong 2007 tập hợp đó vì nếu

không thì rõ ràng có một số sẽ chia hết cho số còn lại, mâu thuẫn. Mặt khác, A lại có đúng 2007

phần tử nên A chứa đúng 1 phần tử của mỗi tập aP nói trên.

Gọi iα là một phần tử của A với , 1, 2007i iP iα ∈ = . Xét các phần tử 1 2 5 1094, , ,...α α α α lần

lượt có các dạng 2 72 ,3.2 ,3 .2 ,..., ,3 .2n n n n ; rõ ràng mỗi phần tử đó chỉ có hai ước nguyên tố là 2 và

3. Ta cũng thấy rằng lũy thừa lớn nhất của 2 trong 1α phải lớn hơn lũy thừa lớn nhất của 2 trong

2α vì nếu ngược lại thì 2 1α α⋮ , mâu thuẫn.

Hoàn toàn tương tự với các phần tử khác trong dãy 1 2 5 1094, , ,...α α α α , tức là nếu i j< thì

lũy thừa của 2 trong iα phải lớn hơn lũy thừa của 2 trong jα . Do đó, lũy thừa của 2 trong dãy

1 2 5 1094, , ,...α α α α là một dãy giảm thực sự. Do có 8 phần tử trong dãy trên (tương ứng với lũy

thừa của 3 thay đổi từ 0 đến 7) nên giá trị của 1α ít nhất là 72 128= . Hơn nữa, các phần tử của

dãy có giá trị của lũy thừa 3 tăng dần (nếu ngược lại, giá trị của lũy thừa 3 giảm mà lũy thừa 2 cũng giảm nên có một số chia hết cho số khác, mâu thuẫn).

Suy ra: 1α chính là giá trị nhỏ nhất trong dãy trên.

Hoàn toàn tương tự với các số nguyên tố khác, ta cũng xét số có dạng 2 .3 ...i i kn n ni kpα = ,

trong đó 2, 3, …, pk là các ước nguyên tố không vượt quá 4014. Từ đó suy ra 1α cũng chính là

phần tử nhỏ nhất trong các phần tử của tập A. Do đó phần tử nhỏ nhất mA của tập A chính là 1α ,

như chứng minh ở trên 1 128α ≥ hay 128Am ≥ .

Ta sẽ chứng minh rằng 128 chính là giá trị nhỏ nhất của mA bằng cách chỉ ra một tập hợp

con A của { }1, 2,3,..., 4013, 4014 thỏa mãn điều kiện đề bài.

Xét tập hợp: { }( ) ( )2 (2 1) |1338 3 .(2 1) 4014f i f iiA i iα= = − ≤ − ≤ .

Page 130: Final Version 1989 to 2010 VN TST

41

Rõ ràng tập hợp này có đúng 2007 phần tử thuộc { }1, 2,3,..., 4013, 4014 .

Ta sẽ chứng minh rằng xα không chia hết cho yα , với x y> (tức là tập hợp này thỏa mãn đề bài).

Thật vậy, giả sử ngược lại tồn tại x, y thỏa mãn x yα α⋮ , khi đó: ( ) ( )2 (2 1) 2 (2 1)f x f yx y− −⋮ ,

tức là ( ) ( )f x f y≥ và (2 1) (2 1)x y− −⋮ .

Từ cách xác định các giá trị u, v; ta có: (2 1) (2 1) 2 1 3(2 1)x y x y− − ⇒ − ≥ −⋮ , đồng thời ( ) 1 ( ) ( ) 13 (2 1) 4014 3 (2 1) 3 (2 1) ( ) ( )f y f x f xy x y f y f x+ +− ≥ > − ≥ − ⇒ > . Mâu thuẫn này chứng tỏ tất

cả các phần tử của A đều thỏa mãn với mọi ,a b A∈ thì a không chia hết cho b.

Vậy 128 chính là giá trị nhỏ nhất của mA cần tìm.

Page 131: Final Version 1989 to 2010 VN TST

42

Bài 6. Cho đa giác 9 cạnh đều (H). Xét ba tam giác với các đỉnh là các đỉnh của đa giác (H) đã cho sao cho không có hai tam giác nào có chung đỉnh. Chứng minh rằng có thể chọn được từ mỗi tam giác 1 cạnh sao cho 3 cạnh này bằng nhau.

Kí hiệu hình (H) đã cho là đa giác

1 2 3 8 9...A A A A A như hình vẽ. Trước hết, ta thấy

rằng độ dài các cạnh và các đường chéo của hình (H) chỉ thuộc 4 giá trị khác nhau (nếu gọi R là bán kính đường tròn ngoại tiếp của (H) thì ta dễ dàng tính được các giá trị đó là

2 .sin9

,2

2 sin9

,3

2 sin9

,4

2 sin9

)

ta đặt chúng là 1 2 3 4, , ,a a a a theo thứ tự tăng

dần của độ dài. Rõ ràng các tam giác có đỉnh thuộc các đỉnh của (H) sẽ có cạnh có độ dài thuộc 1 trong 6 dạng sau:

1 1 2 2 2 4 1 3 4

3 3 3 2 3 4 4 4 1

( , , ), ( , , ), ( , , ),

( , , ), ( , , ), ( , , )

a a a a a a a a a

a a a a a a a a a.

Giả sử 3 tam giác được lấy ra là 1 2 3, ,∆ ∆ ∆ .

Xét các trường hợp sau:

- Nếu trong các tam giác đó có một tam giác đều, rõ ràng, tam giác này phải có độ dài các

cạnh là 3

2 .sin9

; không mất tính tổng quát, giả sử đó là tam giác 1 4 7A A A . Do các tam giác

1 2 3, ,∆ ∆ ∆ không có hai đỉnh nào trùng nhau nên ta sẽ lập một tam giác có các đỉnh là một trong

hai đỉnh của các tập hợp 2 3 4 5 7 8{ , },{ , },{ , }A A A A A A . Ta sẽ chứng minh rằng tam giác đó phải có ít

nhất 1 cạnh có độ dài là 3

2 .sin9

, tức là hai đỉnh có chỉ số có cùng số dư khi chia cho 3. Giả sử

ngược lại, trong hai tam giác cần lập, không có tam giác nào có cạnh là 3

2 .sin9

, khi đó đỉnh

A2 phải nối với A4 và A4 phải nối với A8, nhưng khi đó A8 được nối với A2 là hai đỉnh có chỉ số chia cho 3 cùng dư là 2, mâu thuẫn. Do đó, trong hai tam giác lập được, luôn có một cạnh có độ

dài là 3

2 .sin9

. Suy ra trường hợp này luôn có tam giác thỏa mãn đề bài.

a1

a2

a3

a4

9

8

7

6

5

4

3

21

A

A

A

A

A

A

A

A A

Page 132: Final Version 1989 to 2010 VN TST

43

- Nếu trong các tam giác đó, không có tam giác nào đều . Khi đó các tam giác được xét

không có ba đỉnh cùng thuộc một trong ba tập hợp sau: 1 1 4 7{ , , }A A Aα = , 2 2 5 8{ , , }A A Aα = ,

3 3 6 9{ , , }A A Aα = . Ta thấy một đoạn thẳng nối hai điểm bất kì thuộc hai tập khác nhau sẽ nhận 1

trong 3 giá trị là 1 2 4, ,a a a . Hơn nữa, không có tam giác nào có độ dài 3 cạnh là 1 2 4( , , )a a a nên ta

có hai nhận xét:

(1) Một tam giác có các đỉnh thuộc cả ba tập 1 2 3, ,α α α nói trên thì sẽ có hai cạnh nào đó có độ

dài bằng nhau (các cạnh của nó có thể là 1 1 2( , , )a a a , 2 2 4( , , )a a a , 4 4 1( , , )a a a ) tức là nó phải cân.

(2) Một tam giác có hai trong ba đỉnh thuộc cùng một tập thì tam giác đó các cạnh có độ dài

là 2 3 4( , , )a a a hoặc là 1 3 4( , , )a a a , tức là tam giác đó không cân.

* Ta xét tiếp các trường hợp (các tam giác xét dưới đây là cân nhưng không đều):

+ Có hai tam giác cân và một tam giác không cân: khi đó theo nhận xét (1), hai tam giác cân đó

phải có đỉnh thuộc các tập hợp khác nhau trong ba tập 1 2 3, ,α α α ; khi đó, rõ ràng tam giác còn lại

cũng phải có đỉnh thuộc các tập hợp khác nhau, tức là nó phải cân, mâu thuẫn.

Vậy trường hợp này không tồn tại.

+ Có một tam giác cân và hai tam giác không cân: khi đó theo nhận xét (2), hai tam giác không cân đó phải có hai đỉnh thuộc cùng một tập hợp và đỉnh còn lại thuộc tập hợp khác, giả sử một

tam giác có hai đỉnh thuộc 1α và một đỉnh thuộc 2α ; rõ ràng tam giác không cân còn lại phải có

hai đỉnh thuộc 2α , một đỉnh thuộc 3α , suy ra tam giác còn lại có hai đỉnh thuộc 3α , một đỉnh

thuộc 1α nên nó là tam giác cân, mâu thuẫn.

Vậy tương tự như trên, trường hợp này không tồn tại.

+ Cả ba tam giác đều không cân: khi đó theo nhận xét (2), tam giác đó thuộc một trong hai dạng

2 3 4( , , )a a a hoặc là 1 3 4( , , )a a a , tức là các tam giác này luôn chứa 1 cạnh có độ dài là a3.

Trong trường hợp này, bài toán được giải quyết.

+ Cả ba tam giác đều cân: khi đó, các tam giác có độ dài là 1 1 2( , , )a a a , 2 2 4( , , )a a a , 4 4 1( , , )a a a .

Rõ ràng không tồn tại trường hợp có độ dài các cạnh lần lượt nhận cả ba giá trị như ba bộ trên nên phải có hai bộ trùng nhau, tức là có ít nhất hai tam giác cân bằng nhau và một tam giác cân nhận một trong ba giá trị thuộc một trong các bộ trên làm cạnh, khi đó luôn có thể chọn từ tam giác này một cạnh bằng với cạnh đáy hoặc cạnh bên của hai tam giác cân bằng nhau kia. Trong trường hợp này, bài toán cũng được giải quyết. Vậy trong mọi trường hợp, ta luôn có đpcm.

Page 133: Final Version 1989 to 2010 VN TST

44

LỜI GIẢI ĐỀ THI CHỌN ĐỘI TUYỂN QUỐC GIA DỰ THI IMO 2008

Bài 1. Trong mặt phẳng cho góc xOy. Gọi M, N lần lượt là hai điểm lần lượt nằm trên các tia Ox, Oy. Gọi d là đường phân giác góc ngoài của góc xOy và I là giao điểm của trung trực MN với đường thẳng d. Gọi P, Q là hai điểm phân biệt nằm trên đường thẳng d sao cho IM IN IP IQ= = = , giả sử K là giao điểm của MQ và NP.

1. Chứng minh rằng K nằm trên một đường thẳng cố định. 2. Gọi d1 là đường thẳng vuông góc với IM tại M và d2 là đường thẳng vuông góc với IN tại

N. Giả sử các đường thẳng d1, d2 cắt đường thẳng d tại E, F. Chứng minh rằng các đường thẳng EN, FM và OK đồng quy.

1. Xét trường hợp các điểm M, Q và N, P nằm cùng phía với nhau so với trung trực của MN. Khi đó giao điểm K của MP và NQ thuộc các đoạn này.: Gọi I’ là giao điểm của d với đường tròn ngoại tiếp MON∆ . Do d là phân giác ngoài của �MON nên I’ chính là trung điểm của cung MON , do đó: I’M = I’N hay I’ chính là giao điểm của trung trực MN với d. Từ đó, suy ra: 'I I≡ hay tứ giác MION nội tiếp.

Ta được: � �NIO NMO= . Mặt khác: do IM = IN = IP = IQ nên tứ giác MNPQ nội tiếp trong đường tròn tâm I, đường kính

PQ � �2PIN PMN⇒ = (góc nội tiếp và góc ở tâm cùng chắn cung �PN ).

Từ các điều trên, ta có: � �2NMO PMN= ⇒ MP là phân giác trong của �OMN .

x

y

d

J

F

E

K

P

Q

I

O

M

N

Page 134: Final Version 1989 to 2010 VN TST

45

Tương tự, ta cũng có: NQ là phân giác trong của �ONM . Do K là giao điểm của MP và NQ nên K chính là tâm đường tròn nội tiếp của MON∆ , suy ra K

thuộc phân giác trong của �xOy , tức là K thuộc một đường thẳng cố định (đpcm).

- Nếu giao điểm K nằm ngoài các đoạn MP và NQ: ta cũng có lập luận tương tự và có

được K là tâm đường tròn bàng tiếp �MON của tam giác MON∆ , tức là K cũng thuộc phân giác

trong của �xOy , là một đường thẳng cố định.

Trong mọi trường hợp, ta luôn có đpcm. 2. Gọi J là giao điểm của d1 và d2. Ta thấy tứ giác MINJ nội tiếp trong đường tròn đường kính IJ. Hơn nữa: MION cũng là tứ giác nội tiếp nên 5 điểm M, N, I, J, O cùng thuộc một đường

tròn. Do đó: phân giác trong góc �MON đi qua trung điểm của cung MJN . Rõ ràng M, N đối xứng nhau qua trung trực của MN nên JM = JN, tức là J cũng là trung

điểm của cung �MON .

Từ đó suy ra: J thuộc phân giác trong của �MON hay O, K, J thẳng hàng.

Ta cần chứng minh các đoạn JO, EN và MF trong JEF∆ đồng quy.

Thật vậy: . .sin sin

.. .sin sin

OEJ

OFJ

SOE JO JE OJE JE OJE

OF S JO JF OJF JF OJF= = = .

Trong JEF∆ và MON∆ , ta có : sin sin

,sin sin

JE JFE OM ONM

JF JEF ON OMN= = .

Mặt khác : � � �� � � sin sin,

sin sin

OJE ONMOJE OJN ONM OJF OJM OMN

OJF OMN= = = = ⇒ = .

Kết hợp lại, ta được :sin sin sin

. . .sin sin sin

OE JFE OM OFN OM OFN OM

OF JEF ON OEM ON OEM ON= = =

sin .sin

..sin sin

OFN OM MOE ME

ON NOF OEM NF= = .

Do đó : . 1 . . 1OE FN OE NF MJ

OF EM OF NJ ME= ⇒ = .

Theo định lí Ceva đảo, ta có OI, EN và MF đồng quy. Đây chính là đpcm.

Page 135: Final Version 1989 to 2010 VN TST

46

Bài 2. Hãy xác định tất cả các số nguyên dương m sao cho tồn tại các đa thức với hệ số thực ( ), ( ), ( , )P x Q x R x y thỏa mãn điều kiện:

Với mọi số thực a, b mà 2 0ma b− = , ta luôn có ( ( , ))P R a b a= và ( ( , ))Q R a b b= .

Với m là một số nguyên dương, ta xét các trường hợp :

- Nếu m là số chẵn, đặt 2 , *m k k= ∈ℕ . Suy ra : 2 0m ka b b a− = ⇔ = ± . Khi đó cần tìm k

sao cho các đa thức ( ) ( ) ( ), , , P x Q x R x y thỏa mãn cả hai điều kiện :

(1) ( ( , )) , ( ( , )) ,k k kP R x x x Q R x x x x= = ∀ ∈ℝ .

(2) ( ( , )) , ( ( , )) ,k k kP R x x x Q R x x x x− = − = − ∀ ∈ℝ .

Xét đa thức một biến T(x) thỏa mãn : ( , ) ( ),kR x x T x x= ∀ . Theo giả thiết thì

( ( )) ( ( , )) ,kP T x P R x x x x= = ∀ . Từ đó suy ra : deg ( ).deg ( ) 1P x T x = hay deg ( ) deg ( ) 1P x T x= = .

Giả sử ( ) , , , 0T x ux v u v u= + ∈ ≠ℝ , ( ) '. ', ', ' , ' 0P x u x v u v u= + ∈ ≠ℝ thì

'( ) ' , . ' 1, ' ' 0u ux v v x x u u u v v+ + = ∀ ⇒ = + = hay 1

' , ' ( )v x v

u v P xu u u

−= = − ⇒ = .

Mặt khác : ( ( , )) ( ( )) ( ) ( ) ( ),k

k k kx uQ R x x Q T x Q ux v x Q x P x x

u

− = = + = ⇒ = = ∀

.

Suy ra : ( ( , )) ( ( , )) , ,k kQ R a b P R a b b a a b= ⇒ = ∀ thỏa 2 0ma b− = . Nhưng theo điều kiện ban

đầu thì b cũng có thể là ka− . Mâu thuẫn này cho thấy các giá trị m trong trường hợp này không thỏa mãn đề bài.

- Nếu m là số lẻ. Đặt 2( , ) ( )mP x x S x= . Suy ra : 2( ( )) , deg ( ).deg ( ) 2P S x x x P x S x= ∀ ∈ ⇒ =ℝ .

Nếu như degS(x) = 2 thì deg ( ( ))Q S x là số chẵn, trong khi đó : deg ( ( )) deg mQ S x x m= =

với m là số lẻ, mâu thuẫn. Suy ra : degS(x) = 1, degP(x) = 2.

Mặt khác, trong đa thức 2( , )mR x x , bậc của nó có thể đạt giá trị nhỏ nhất là min(2, )m mà 2deg ( , ) deg ( ) 1mR x x S x= = nên m = 1.

Ta sẽ chứng minh rằng giá trị m = 1 này thỏa mãn đề bài bằng cách chỉ ra các đa thức P(x), Q(x), R(x, y) thỏa mãn đề bài. Thật vậy :

Xét các đa thức 2( ) , ( ) , ( , )P x x Q x x R x y y= = = .

Khi đó với m = 1 thì ta có quan hệ của a với b chính là : 2a b= . Suy ra :

( , )R a b b= , 2( ( , )) ( )P R a b P b b a= = = , ( ( , )) ( )Q R a b P b b= = , thỏa mãn đề bài.

Vậy tất cả các giá trị m thỏa mãn đề bài là m = 1.

Page 136: Final Version 1989 to 2010 VN TST

47

Bài 3. Cho số nguyên 3n > . Kí hiệu T là tập hợp gồm n số nguyên dương đầu tiên. Một tập con S của T được gọi là tập khuyết trong T nếu S có tính chất: Tồn tại số nguyên

dương c không vượt quá 2

n sao cho với 1 2,s s là hai số bất kì thuộc S ta luôn có 1 2s s c− ≠ .

Hỏi tập khuyết trong T có thể có tối đa bao nhiêu phần tử ? Trước hết ta thấy rằng: Nếu S là tập khuyết trong T thì tập ' { | }S n x x S= − ∈ cũng là một

tập khuyết trong T. Thật vậy: Giả sử ngược lại S’ không phải là tập khuyết, khi đó tồn tại hai số nguyên

dương 1 2' , ' 's s S∈ sao cho 1 2| ' ' |s s c− = với c là một số nguyên dương nào đó không vượt quá 2

n,

Khi đó xét tương ứng hai phần tử 1 1 2 2' , 's n s s n s= − = − thì rõ ràng 1 2,s s S∈ và

1 2 1 2 1 2| | | ( ' ) ( ' ) | | ' ' |s s n s n s s s c− = − − − = − = , tức là tồn tại hai phần tử 1 2,s s S∈ và

1 2| |2

ns s c− = ≤ trong khi S là tập khuyết. Mâu thuẫn này suy ra nhận xét trên được chứng minh.

Hơn nữa, do | | | ' |S S= nên khi S có số các phần tử là lớn nhất thì tương ứng cũng có tập

S’ có số phần tử lớn nhất bằng với S.

Từ đó, ta thấy có thể xét các tập khuyết S có số các số phần tử không vượt quá 2

n không

ít hơn số các số phần tử lớn hơn 2

n. Xét hai tập hợp sau:

{ | , }2

nA x x S x= ∈ ≤ , { | , }

2

nB x x S x= ∈ > thì ,A B A B S∩ = ∅ ∪ = và theo cách xác định S

như trên thì A B≥ .

Khi đó với c là một số nguyên dương nào đó không vượt quá 2

n, ta xét tập hợp:

{ | }C x c x A= + ∈ . Ta có: | | | |A C= . Do A S⊂ nên A cũng là một tập khuyết và khi đó rõ ràng

,A C B C∩ = ∅ ∩ = ∅ (vì nếu ngược lại thì tồn tại hai phần tử thuộc S mà hiệu của chúng là c,

mâu thuẫn). Suy ra tất cả các phần tử thuộc tập A hoặc B hoặc C đều là một số nguyên dương không vượt quá n, tức là ( ) | | | | | | | |A B C T A B C T n∪ ∪ ⊂ ⇒ + + ≤ = .

Kết hợp các điều này lại, ta được: 2 | | | |A B n+ ≤ . Do đó: 4 | | 2 | | 2

| | | |3 3

A B nA B

++ ≤ ≤ .

Hơn nữa: ,A B A B S∩ = ∅ ∪ = và | |S là số nguyên nên 2

| | | | | |3

nS A B = + ≤

.

Do đó số phần tử của tập khuyết S trong T luôn không vượt quá 2

3

n

.

Page 137: Final Version 1989 to 2010 VN TST

48

*Ta sẽ chỉ ra một tập khuyết thỏa mãn đề bài có đúng 2

3

n

phần tử.

Thật vậy, xét hai tập hợp A, B như sau:

11,2,3,...,

3

nA

+ = , 1, 2,...,

3 3

n nB n n n

= − + − + và S A B= ∪ .

Chọn 1

3 2

n nc

+ = ≤ . Ta thấy:

- Hiệu hai phần tử bất kì trong A không vượt quá 1 1

13 3

n n+ + − < .

- Hiệu hai phần tử bất kì trong B không vượt quá 1

( 1) 13 3 3

n n nn n

+ − − + = − < .

- Hiệu một phần tử bất kì thuộc B với một phần tử bất kì thuộc A không nhỏ hơn:

1 1 2 1( 1) 1

3 3 3 3 3 3

n n n n n nn n

+ + + + − + − ≥ − − + = > .

Khi đó, rõ ràng S A B= ∪ là một tập khuyết trong T ứng với giá trị 1

3 2

n nc

+ = ≤ .

Ta sẽ chứng minh rằng 2

| |3

nS =

. Từ cách xác định A, B, ta có:1

| | ,3 3

n nA B

+ = = .

Ta cần có: 1 2

,3 3 3

n n nn

+ + = ∀ ∈ ℕ (*).

*Xét các trường hợp: - Nếu n chia hết cho 3, tức là n có dạng 3 ,m m∈ℕ . Suy ra:

1 3 1 3 6 22

3 3 3 3 3 3

n n m m m nm m m

+ + + = + = + = = = .

- Nếu n chia 3 dư 1, tức là n có dạng 3 1,m m+ ∈ℕ . Suy ra:

1 3 2 3 1 6 6 2 22

3 3 3 3 3 3 3

n n m m m m nm m m

+ + + + + = + = + = = = = .

- Nếu n chia 3 dư 2, tức là n có dạng 3 2,m m+ ∈ℕ . Suy ra:

1 3 3 3 2 6 3 6 4 21

3 3 3 3 3 3 3

n n m m m m nm m

+ + + + + + = + = + + = = = .

Từ đó suy ra (*) được chứng minh hay tập hợp S đã cho là tập khuyết có 2

3

n

.

Vậy giá trị lớn nhất của số phần tử của tập khuyết S trong T là 2

3

n

.

Page 138: Final Version 1989 to 2010 VN TST

49

Bài 4. Cho m, n là các số nguyên dương.

Chứng minh rằng (2 3) 1nm + + chia hết cho 6m khi và chỉ khi 3 1n + chia hết cho 4m.

Theo khai triển nhị thức Newton thì:

1

1

(2 3) (2 ) 3 .(2 ) .3 (2 ) 3 (mod 6m)n

n n n k n k k n nn

k

m m C m m−

=

+ = + + ≡ +∑ .

Do đó, 6 | (2 3) 1nm m + + 6 | (2 ) 3 1n nm m⇔ + + 2 | (3 1)nm⇔ + và 3 | (2 ) 1nm + .

Cần chứng minh rằng: 2 | (3 1)nm + và 3 | (2 ) 1nm + (1) 4 | 3 1 (2)nm⇔ + .

Xét các trường hợp: * Nếu m là số chẵn: - Xét điều kiện (2):

3 1n + không chia hết cho 4m vì 3 1 2(mod8)n + ≡ hoặc 3 1 4(mod8)n + ≡ , trong khi 4 8m⋮ , tức là

không thể có điều kiện (2).

- Xét điều kiện (1): từ m là số chẵn, suy ra 3 1 4n n+ ⇒⋮ là số lẻ. Ta biết rằng: số có dạng

3n + 1 chỉ có ước nguyên tố lẻ đồng dư với 1 modun 4. Từ đó, suy ra m thỏa mãn: 2 | (3 1)nm +

phải có dạng 2(3 1),m k k= + ∈ℤ . Suy ra: (2 ) 1 2 .2 .(3 1) 1 2(mod 3) (2 ) 1n n nm k m+ = + + ≡ ⇒ +

không chia hết cho 3, tức là điều kiện (1) cũng không tồn tại. * Nếu m là số lẻ:

- Xét điều kiện (1): từ 2 | (3 1)nm + suy ra m không chia hết cho 3, từ 3 | (2 ) 1nm + suy ra n

phải là số lẻ vì nếu ngược lại thì (2 ) 1 2(mod 3)nm + ≡ , mâu thuẫn. Mà n là số lẻ thì 3 1 4n + ⋮ ,

kết hợp với ( , 4) 1m = , ta được 4 | 3 1nm + , đây chính là điều kiện (2). Do đó: (1) (2)⇒ .

- Xét điều kiện (2): từ 4 | 3 1 (4 | 3 1) ( | 3 1)n n nm m+ ⇒ + ∧ + suy ra n là số lẻ và m có dạng

3 1,k k+ ∈ℤ . Suy ra: (2 ) 1 2 . 1 1 1 0(mod 3) 3 | (2 ) 1n n n nm m m+ = + ≡ − + ≡ ⇒ + ; từ (2) ta cũng trực

tiếp có 2 | (3 1)nm + . Do đó: (2) (1)⇒ .

Kết hợp các điều trên lại, ta được: (1) (2)⇔ .

Vậy (2 3) 1nm + + chia hết cho 6m khi và chỉ khi 3 1n + chia hết cho 4m.

Đây chính là đpcm.

Page 139: Final Version 1989 to 2010 VN TST

50

Bài 5. Cho tam giác ABC nhọn, không cân có O là tâm đường tròn ngoại tiếp. Gọi AD, BE, CF lần lượt là các đường phân giác trong của tam giác. Trên các đường thẳng

AD, BE, CF lần lượt lấy các điểm L, M, N sao cho AL BM CN

kAD BE CF

= = = (k là hằng số dương).

Gọi (O1), (O2), (O3) lần lượt là các đường tròn đi qua L, tiếp xúc với OA tại A ; đi qua M tiếp xúc với OB tại B và đi qua N tiếp xúc với OC tại C.

1. Chứng minh rằng với 1

2k = , ba đường tròn (O1), (O2), (O3) có đúng hai điểm chung và

đường thẳng nối hai điểm đó đi qua trọng tâm tam giác ABC. 2. Tìm tất cả các giá trị k sao cho 3 đường tròn (O1), (O2), (O3) có đúng hai điểm chung. Trước hết, ta nêu 4 bổ đề sau: (1) Cho ba đường thẳng đôi một phân biệt a, b, c và hai đường thẳng phân biệt d, d’. Các

đường thẳng d, d’ theo thứ tự cắt a, b, c tại 1 1 1 2 2 2, , ; , ,A B C A B C thỏa mãn điều kiện:

1 1 2 2

1 1 2 2

A B A Bk

AC A C= = . Các điểm A3, B3, C3 thuộc a, b, c sao cho: 1 2 1 2 1 2

1 3 1 3 1 3

A A B B C C

A A B B C C= = .

Khi đó, A3, B3, C3 thẳng hàng và 3 3

3 3

A Bk

A C= .

(2) Cho ba đường thẳng phân biệt a, b, c và ba đường thẳng phân biệt khác a’, b’, c’ . Các

đường thẳng a’, b’, c’ theo thứ tự cắt a, b, c tại 1 1 1 2 2 2 3 3 3, , ; , , ; , ,A B C A B C A B C (các điểm này đôi

một phân biệt). Khi đó nếu 3 31 1 2 2

1 1 2 2 3 3

A BA B A B

AC A C A C= = thì hoặc 1 2 1 2 1 2

1 3 1 3 1 3

A A B B C C

A A B B C C= = hoặc a, b, c đôi

một song song. (3) Cho tam giác ABC và M bất kì. Các tia AM, BM, CM lần lượt cắt BC, CA, AB ở A1, B1, C1. Các đường thẳng A1B1, B1C1, C1A1 cắt các đường thẳng AB, BC, CA lần lượt ở A2, B2, C2. Các điểm A3, B3, C3 theo thứ tự nằm trên các đường thẳng BC, CA, AB sao cho

1 3 1 3 1 3

1 2 1 2 1 2

, 0A A B B C C

k kA A B B C C

= = = ≠ . Khi đó, A3, B3, C3 thẳng hàng khi và chỉ khi 1k = hoặc 1

2k = .

(4) Cho tam giác ABC không cân ngoại tiếp đường tròn (I). Đường tròn (I) tiếp xúc với các cạnh BC, CA, AB lần lượt tại D, E, F. Đường thẳng EF cắt BC tại M, đường thẳng AD cắt (I) tại N (khác D). Chứng minh rằng: MN tiếp xúc với (I). Các bổ đề (1), (2) có thể chứng minh dễ dàng bằng các biểu diễn theo vectơ. Dưới đây trình bày các chứng minh cho bổ đề (3), (4).

Page 140: Final Version 1989 to 2010 VN TST

51

*Chứng minh bổ đề (3). + Điều kiện đủ:

- Với k = 1, ta có A3, B3, C3 theo thứ tự trùng với A2, B2, C2. Vì AA1, BB1, CC1 đồng quy nên theo định lí

Menelaus thì 1 1 1

1 1 1

. . 1A B B C C A

AC B A C B= − . Vì A2, B1, C1 thẳng

hàng nên 2 1 1

2 1 1

. . 1A B B C C A

A C B A C B= . Suy ra: 1 2

1 2

A B A B

AC A C= − .

Tương tự: 1 2 1 2

1 2 1 2

,B C B C C A C A

B A B A C B C B= − = − .

Nhân từng vế các đẳng thức trên,

2 2 2 1 1 1

2 2 2 1 1 1

. . . . 1A B B C C A A B B C C A

A C B A C B AC B A C B

= − − − = −

.

Tức là A2, B2, C2 thẳng hàng hay A3, B3, C3 thẳng hàng.

- Với 1

2k = , A3, B3, C3 theo thứ tự là trung điểm của

A1A2, B1B2, C1C2. Theo chứng minh trên, ta đã có:

1 2

1 2

A B A B

AC A C= − . Theo tính chất tỉ lệ thức thì:

31 2 1 2 1 2 1 2 1

1 2 1 2 1 2 1 2 1 3

2

2

A BA B A B A B A B A B A B A B A A

AC A C AC A C AC A C AC A A A C

+ −= − = = ⇒ = =

− +

Suy ra:

2

3 3 2 1 1

3 2 1 3 1

2.2

A B A B A A A B

A C A A A C AC

= =

. Tương tự:

2 2

3 31 1

3 1 3 1

,B C C AB C C A

B A B A C B C B

= =

.

Nhân từng vế các đẳng thức trên, ta được: 2 2 2

3 3 3 1 1 1

3 3 3 1 1 1

. . . . 1A B B C C A A B B C C A

A C B A C B AC B A C B

= =

. Do đó, A3, B3, C3 th ẳng hàng.

+ Điều kiện cần: Khi 1k ≠ , ta kí hiệu 3( ) 3( ) 3( ), ,k k kA B C thay cho A3, B3, C3. Giả sử tồn tại số k

đồng thời khác 1 và 1

2mà 3( ) 3( ) 3( ), ,k k kA B C thẳng hàng. Khi đó, các điểm: 3( ) 3( ) 3( ), ,k k kA B C và

3(1/2) 3(1/2) 3(1/2), ,A B C đôi một khác nhau. Dễ thấy: 2 3(1/2) 2 3(1/2) 2 3(1/2)

2 3( ) 2 3( ) 2 3( )

1/ 2 1

1k k k

A A B B C C

kA A B B C C

−= = =

−.

B3

A3

C3

C2

B2

A2A1

C1

B1

A

B C

M

C2

B2

A2A1

C1

B1

A

B C

M

Page 141: Final Version 1989 to 2010 VN TST

52

Theo chứng minh ở điều kiện đủ thì hai bộ điểm A2, B2, C2 và 3(1/2) 3(1/2) 3(1/2), ,A B C thẳng

hàng, mà theo điều giả sử ở trên thì 3( ) 3( ) 3( ), ,k k kA B C cũng thẳng hàng nên theo bổ đề (2), hoặc

đường thẳng A2B2C2 và 3(1/2) 3(1/2) 3(1/2)A B C song song hoặc 3(1/2) 3(1/2)2 2

2 2 3(1/2) 3(1/2)

A BA B

A C A C= .

+ Nếu 3(1/2) 3(1/2)2 2

2 2 3(1/2) 3(1/2)

A BA B

A C A C= thì chú ý rằng: 1 3(1/2) 1 3(1/2) 1 3(1/2)

1 2 1 2 1 2

A A B B C C

A A B B C C= = , theo bổ đề (1) thì

A1, B1, C1 thẳng hàng, mâu thuẫn.

+ Nếu A2B2C2 và 3(1/2) 3(1/2) 3(1/2)A B C song

song với nhau thì chú ý rằng

3(1/2) 3(1/2) 3(1/2), ,A B C theo thứ tự là trung

điểm của 1 2 1 2 1 2, ,A A B B C C . Ta có:

3(1/2) 3(1/2) 1 1 2 2

1( )

2A B A B A B= +������������ ����� ������

,

3(1/2) 3(1/2) 1 1 2 2

1( )

2A C A C A C= +������������ ����� ������

. Suy ra:

A1B1 song song với A2B2 và 3(1/2) 3(1/2)A B ,

A1C1 song song với A2C2 và 3(1/2) 3(1/2)A C .

Từ đó suy ra, A1, B1, C1 cũng thẳng hàng, mâu thuẫn.

Do đó chỉ có 1k = và 1

2k = thỏa mãn.

Vậy bổ đề (3) được chứng minh. *Chứng minh bổ đề (4). Gọi H là giao điểm của EF và AI. Ta thấy: IA EF⊥ . Tam giác AIF vuông tại F có đường cao FH nên :

2 2. .IF IH IA ID IH IA= ⇒ = . Suy ra: ( . . )IDH IAD c g c∆ ∆∼ .

Do đó: � �IHD IDA= . Mặt khác: tam giác IDN cân tại

I nên � � �IND IDN IDA= = . Từ

đó, ta được: � �IND IHD= .

⇒Tứ giác IDNH nội tiếp. Hơn nữa, tứ giác IDMH cũng nội tiếp vì có � � 090IDM IHM= = . Do đó: 5 điểm, I, D, M, N, H cùng thuộc một đường tròn. Suy ra: IMNH nội tiếp hay � � 090INM IHM MN IN= = ⇒ ⊥ . Vậy MN là tiếp tuyến của (I). Bổ đề (4) được chứng minh.

A3(1/2)

A3(k)

C2

B2

A2A1

C1

B1

A

B C

M

C3(1/2)

B3(1/2)

B3(k)

C3(k)

I

H

M

N

E

F

D

A

B C

Page 142: Final Version 1989 to 2010 VN TST

53

*Trở lại bài toán.

1. Khi 1

2k = thì L, M, N lần lượt là trung điểm của các đoạn AD, BE, CF.

Gọi H là trực tâm của ABC∆ vàδ là phương tích của H đối với đường tròn Euler đi qua chân 3 đường cao của ABC∆ . Gọi K là giao điểm của đường thẳng AO1 với đường thẳng BC. Ta sẽ chứng minh rằng K nằm trên (O1). Thật vậy: Do ABC∆ là tam giác nhọn nên O nằm trong tam giác. Ta có: � � � �02 90AOB ACB OAB ACB= ⇒ = − .

Không mất tính tổng quát, giả sử tia AD nằm giữa hai tia AO và AB. Khi đó:

� � � ��

� � ��

0 090 902 2

BAC BACOAD OAB DAB ACB KAD OAD ACB= − = − − ⇒ = − = +

.

Mặt khác: � � � � �1

2ADB DAC DCA BAC ACB= + = + nên � �KAD KDA= .

Ta cũng có 1 1 1O A O L AO L= ⇒ ∆ cân tại O1 nên � �1 1O AL O LA= .

Từ đó suy ra: � �1O LA KDA= hay O1L // KD, mà L là trung điểm của AD nên O1 là trung điểm của

AK hay K thuộc đường tròn (O1). Do đó (O1) cắt BC tại chân đường cao của ABC∆ . Từ đó suy ra phương tích của H đối với đường tròn (O1) chính là δ . Hoàn toàn tương tự với các đường tròn (O2), (O3).

1

H

K

O

O

NM

L

D

E

F

A

B

C

Page 143: Final Version 1989 to 2010 VN TST

54

Do H có cùng phương tích đến các đường tròn (O1), (O2), (O3) nên H chính là tâm đẳng phương của 3 đường tròn (O1), (O2), (O3) . Hơn nữa: do OA là tiếp tuyến của (O1) tại A nên phương tích của O đối với (O1) chính là

2OA . Tương tự như vậy, phương tích của O đối với đường tròn (O2) và (O3) lần lượt là 2OB , 2OC , mà O là tâm đường tròn ngoại tiếp của ABC∆ nên OA = OB = OC hay O có cùng

phương tích đến các đường tròn (O1), (O2), (O3), suy ra: O cũng là tâm đẳng phương của 3 đường tròn (O1), (O2), (O3). Giả sử của 3 đường tròn (O1), (O2), (O3) có 3 trục đẳng phương khác nhau thì chúng phải đồng quy tại tâm đẳng phương, mà O và H cùng là tâm đẳng phương của chúng nên O phải trùng với H hay ABC∆ đều, mâu thuẫn với giả thiết ABC∆ không cân. Do đó, điều giả sử trên là sai và 3 đường tròn đã cho phải có 1 trục đẳng phương chung, trục đẳng phương đó chính là đường thẳng đi qua O và H. Ta cũng thấy rằng O nằm ngoài cả 3 đường tròn, H thì nằm giữa các đường cao của ABC∆ nên nó nằm trong cả 3 đường tròn. Suy ra đường thẳng OH cắt cả 3 đường tròn tại 2 điểm nào đó. Vậy 3 đường tròn (O1), (O2), (O3) có đúng 2 điểm chung, hơn nữa, đường thẳng đi qua hai điểm chung đó chính là đường thẳng OH và do đó, nó cũng sẽ đi qua trọng tâm của tam giác (đường thẳng Euler). Ta có đpcm.

2. Ta sẽ chứng minh rằng ba đường tròn (O1), (O2), (O3) có đúng hai điểm chung khi và chỉ

khi 0k = hoặc 1

2k = . Thật vậy:

*Điều kiện đủ:

- Khi 1

2k = , khẳng định đã chứng minh ở câu 1/.

- Ta sẽ tiếp tục chứng minh rằng với k = 1, ba đường tròn (O1), (O2), (O3) lần lượt đi qua L, tiếp xúc với OA tại A ; đi qua M tiếp xúc với OB tại B và đi qua N tiếp xúc với OC tại C cũng có đúng hai điểm chung. Thật vậy: - Khi k = 1, các điểm L, M, N tương ứng trùng với các điểm D, E, F. Theo chứng minh ở câu 1/, đường tròn (K, KA) đi qua D và tiếp xúc với OA tại A nên chính là đường tròn (O1) đang được xét. Gọi d1, d2, d3 là tiếp tuyến của đường tròn (O) lần lượt

tại A, B, C. Gọi X, Y, Z theo thứ tự là giao điểm của 2 3 3 1 1 2, ; , ; , d d d d d d .

Vì O1 thuộc đường thẳng BC và OA tiếp xúc với (O1) tại A nên O1 thuộc d1, từ đó suy ra O1 chính là giao điểm của BC và d1. Tương tự: O2, O3 lần lượt chính là giao điểm của CA và d2, AB và d3.

Qua các điểm O1, O2, O3 vẽ các tiếp tuyến tới đường tròn (O) lần lượt là 1 1 2 2 3 3, ,O T O T O T

(trong đó T1, T2, T3 là các tiếp điểm).

Ta có: 1 1 1 2 2 2 3 3 3, , O T O A O T O B O T O C= = = , tức là T1, T2, T3 cũng tương ứng thuộc các đường

tròn (O1), (O2), (O3).

Page 144: Final Version 1989 to 2010 VN TST

55

Theo bổ đề (4) ở trên, (xét tam giác XYZ có (O) là đường tròn nội tiếp) các đường thẳng AT1, BT2, CT3 tương ứng trùng với các đường thẳng AX, BY, CZ. Hơn nữa, XB = XC, YC = YA, ZA = ZA nên:

. . 1AY BZ CX

AZ BX CY= − ⇒ AX, BY, CZ đồng quy (theo định lí Ceva đảo trong tam giác XYZ).

Do đó: AT1, BT2, CT3 đồng quy. Đặt điểm chung của ba đường thẳng đó là S, rõ ràng S nằm trong (O). Do T1, T2, T3 nằm trên (O) nên theo tính chất phương tích:

1 2 321 3 /( ) /( ) /( ). . . S O S O S OSA ST SB ST SC ST P P P= = ⇒ = =

.

Tương tự câu 1/, ta có: 1 2 3/( ) /( ) /( )O O O O O OP P P= = , tức là OS là trục đẳng phương chung của ba

đường tròn O1), (O2), (O3). Mặt khác, S nằm trong cả ba đường tròn, O nằm ngoài cả ba đường tròn nên đường thẳng OS cắt cả ba đường tròn tại hai điểm, tức là (O1), (O2), (O3) có đúng hai điểm chung. Vậy trong trường hợp k = 1, ba đường tròn (O1), (O2), (O3) cũng có đúng hai điểm chung. Điều kiện đủ của khẳng định trên được chứng minh.

S

T3

T2

T1

O2

O3

O1

Z

X

Y

O

CB

A

Page 145: Final Version 1989 to 2010 VN TST

56

*Điều kiện cần:

Với một giá trị 0, 1k k> ≠ , gọi 1( ) 2( ) 3( ), ,k k kO O O lần lượt là tâm của các đường tròn đi qua

L, tiếp xúc với (O) tại A; đi qua M, tiếp xúc với (O) tại B, đi qua N, tiếp xúc với (O) tại N. Giả sử các đường tròn

( ) ( ) ( )1( ) 2( ) 3( ), ,k k kO O O nói trên có

đúng hai điểm chung, tức là ba

tâm của chúng là 1( ) 2( ) 3( ), ,k k kO O O

thẳng hàng. (1) Gọi d1, d2, d3 là tiếp tuyến của đường tròn (O) lần lượt tại A, B, C. Gọi X, Y, Z theo thứ tự là giao điểm của d2, d3; d3, d1; d1, d2. Chứng minh tương tự như trên, AX, BY, CZ đồng quy. (2) Đặt O1, O2, O3 là giao điểm của BC với YZ, CA với ZX, AB với XY. Dễ thấy rằng:

1( ) 2( ) 3( ), ,k k kO O O lần lượt thuộc các

đoạn thẳng 1 2 3, ,AO BO CO

và 1( ) 2( )

1 2

, k kAO BOAL BM

AD BEAO BO= = ,

3( )

3

kCO CN

CFCO= .

Suy ra:

1( ) 2( ) 3( )

1 2 3

k k kAO BO COk

AO BO CO= = = . (3)

Từ (1), (2), (3), áp dụng bổ đề 3, ta có 1k = hoặc 1

2k = .

Do đó, nếu các đường tròn (O1), (O2), (O3) có đúng hai điểm chung thì 1k = hoặc 1

2k = .

Điều kiện cần của khẳng định được chứng minh.

Vậy tất cả các giá trị k cần tìm là 1k = và 1

2k = .

Bài toán được giải quyết hoàn toàn.

O1(k)

S

O2

O3

O1

Z

X

Y

O

CB

A

O3(k)

O2(k)

Page 146: Final Version 1989 to 2010 VN TST

57

Bài 6. Kí hiệu M là tập hợp gồm 2008 số nguyên dương đầu tiên. Tô tất cả các số thuộc M bởi ba màu xanh, vàng, đỏ sao cho mỗi số được tô bởi một màu và mỗi màu đều được dùng để tô ít nhất một số. Xét các tập hợp sau:

31 {( , , ) ,S x y z M= ∈ trong đó x, y, z có cùng màu và ( ) 0 (mod 2008)}x y z+ + ≡ ;

31 {( , , ) ,S x y z M= ∈ trong đó x, y, z đôi một khác màu và ( ) 0 (mod 2008)}x y z+ + ≡ .

Chứng minh rằng 1 22 S S> . (Kí hiệu 3M là tích Đề các M M M× × ) .

*Trước hết ta sẽ chứng minh bổ đề sau: “Với n là số nguyên dương, xét tập hợp M = {1, 2, 3, …, n}. Tô màu các phần tử của S bởi màu xanh hoặc đỏ. Xét các tập hợp sau:

31 {( , , ) ,S x y z M= ∈ trong đó x, y, z cùng màu và ( ) 0 (mod )}x y z n+ + ≡ ;

32 {( , , ) ,S x y z M= ∈ trong đó x, y, z khác màu và ( ) 0 (mod )}x y z n+ + ≡ .

Giả sử rằng trong M có a số được tô màu đỏ và b số được tô màu xanh (với a + b = n) thì 2 2

1| |S a ab b= − + , 2| | 3S ab= .”

Thật vậy: Ta chọn một số x được tô màu đỏ, một số y được tô màu xanh, x y≠ . Giả sử z là một số thuộc S mà | ( )n x y z+ + , rõ ràng z tồn tại và duy nhất (z có thể trùng với x hoặ y). Không mất tính tổng quát, giả sử z được tô màu đỏ, cùng màu với x. Xét hai trường hợp: - Nếu z khác cả x và y. Khi đó cả ba số x, y, z là phân biệt. Khi đó, ta có tất cả 6 bộ ba thuộc tập S2 chứa cả x và y là: ( , , ); ( , , ); ( , , ); ( , , ); ( , , ); ( , , )x y z x z y y x z y z x z x y z y x . Nếu không tính đến thứ tự của x và y thì chỉ có 3 bộ trong các bộ trên thuộc S2. Thật vậy: khi xét x đứng trước z trong các bộ trên, ta có 3 bộ ba là: ( , , ); ( , , ); ( , , )x y z x z y y x z . Tương tự, lại xét các bộ không tính đến thứ tự của z và y (chú ý rằng ở trêm, ta giả sử x và x cùng màu), xét x đứng sau z trong các bộ này, ta cũng có 3 bộ ba nữa là: ( , , ); ( , , ); ( , , )z x y z y x y z x . Do đó, trong trường hợp này, ta có tất cả 3 bộ thuộc S2. - Nếu z bằng x hoặc z bằng y. Không mất tính tổng quát, giả sử z = x (trường hợp z = y hoàn toàn tương tự, không quan tâm đến màu của chúng nữa). Khi đó, ta cũng có các bộ 3 thuộc tập S2 chứa cả x và y là: ( , , ); ( , , ); ( , , )x x y x y x y x x (chỉ xét các bộ không tính đến thứ tự của x, y). Do đó, trong cả hai trường hợp, mỗi bộ không tính đến thứ tự (x, y) với x, y khác màu nhau cho ta đúng 3 phần tử trong tập T2 và mỗi phần tử như vậy xuất hiện đúng một lần. Suy ra giá trị của |S2| bằng 3 lần số bộ không tính đến thứ tự (x, y) đã nêu. Mặt khác: có a số được tô màu đỏ, b số được tô màu xanh nên số bộ (x, y) nói trên chính là ab, từ đó ta được: 2| | 3S ab= . Với x, y cho trước thì số z thỏa mãn | ( )n x y z+ + là duy nhất.

Cả x và y được chọn đúng n lần nên 2 21 2| | ( )S S n a b∪ = = + , hơn nữa: 1 2S S∩ = ∅ nên

2 2 2 21 2 1| | | | ( ) | | ( ) 3S S a b S a b ab a ab b+ = + ⇒ = + − = − + .

Bổ đề được chứng minh.

Page 147: Final Version 1989 to 2010 VN TST

58

*Trở lại bài toán: Giả sử số các số được tô màu xanh, vàng, đỏ lần lượt là a, b, c thì : 2008; , , *a b c a b c+ + = ∈ℕ .

- Xét tập hợp: 3{( , , )A x y z M= ∈ | x, y, z được tô cùng màu xanh và 0 (mod 2008)}x y z+ + ≡ . Các tập B, C định nghĩa tương tự, ứng với các màu vàng và đỏ.

- Xét tập hợp: 3{( , , )AB x y z M= ∈ | x, y, z được tô bởi hai màu xanh, vàng và 0 (mod 2008)}x y z+ + ≡ Các tập BC, CA được định nghĩa tương tự, ứng với các cặp màu vàng, đỏ và đỏ, xanh. - Xét tập hợp :

3{( , , )ABC x y z M= ∈ |x, y, z được tô bởi cả ba màu xanh, vàng, đỏ và 0 (mod 2008)}x y z+ + ≡ Tiếp theo, ta sẽ dùng bổ đề trên đánh giá số phần tử của các tập hợp trên: Gọi c là màu đại diện cho hai màu xanh và vàng, khi đó: số bộ ba được tô cùng màu chính là: A B C AB∪ ∪ ∪ và số bộ ba tô khác màu chính là: ABC BC CA∪ ∪ . Ta có: 2 2| | | | | | | | ( ) ( ) ,| | | | | | 3 ( )A B C AB a b c a b c ABC BC CA c a b+ + + = + − + + + + = + . Hoàn toàn tương tự, ta có:

2 2| | | | | | | | ( ) ( ) ,| | | | | | 3( ( )A B C BC b c a b c a ABC CA AB a b c+ + + = + − + + + + = + . 2 2| | | | | | | | ( ) ( ) ,| | | | | | 3 ( )A B C CA c a b c a b ABC AB BC b c a+ + + = + − + + + + = + .

Theo cách xác định như trên thì:

1 1| | | | | | | |S A B C S A B C= ∪ ∪ ⇒ = + + , 2 2| | | |S ABC S ABC= ⇒ =

Cộng từng vế tương ứng của nhóm thứ nhất rồi nhân với hai, ta được: 2 2 26(| | | | | |) 2(| | | | | |) 3( )A B C AB BC CA a b c+ + + + + = + + .

Cộng từng vế tương ứng của nhóm thứ hai, ta được: 3 | | 2(| | | | | |) 6( )ABC AB BC CA ab bc ca+ + + = + + .

Suy ra: 2 2 26(| | | | | |) 3 | | 3 ( ) ( ) ( ) 0A B C ABC a b b c c a + + − = − + − + − ≥ .

Do đó: 1 22 | | | | 0S S− ≥ .

Đẳng thức không xảy ra vì không tồn tại a b c= = nguyên dương và 2008a b c+ + = . Vậy bất đẳng thức ở trên là thực sự, tức là 1 2 1 22 | | | | 0 2 | | | |S S S S− > ⇔ > .

Đây chính là đpcm.

Page 148: Final Version 1989 to 2010 VN TST

59

LỜI GIẢI ĐỀ THI CHỌN ĐỘI TUYỂN QUỐC GIA

DỰ THI IMO 2009

Bài 1. Cho tam giác nhọn ABC nội tiếp đường tròn (O). Gọi 1 1 1, , A B C và 2 2 2, , A B C

lần lượt là các chân đường cao của tam giác ABC hạ từ các đỉnh , , A B C và các điểm đối

xứng với 1 1 1, , A B C qua trung điểm của các cạnh , , BC CA AB . Gọi 3 3 3, , A B C lần lượt là các

giao điểm của đường tròn ngoại tiếp các tam giác 2 2 2 2 2 2, , AB C BC A CA B với (O).

Chứng minh rằng: 1 3 1 3 1 3, , A A B B C C đồng quy.

Ta sẽ chứng minh các đường thẳng

1 3 1 3 1 3, , A A B B C C cùng đi qua trọng tâm của tam giác

ABC. Thật vậy: Gọi M là trung trực của BC, A’ là điểm đối xứng với A qua trung trực của BC. Ta sẽ chứng minh rằng A’ trùng với A3 hay đường tròn (AB2C2) cắt (O) tại A’.

Ta có: A, A’ đối xứng nhau qua trung trực của BC nên: ’ , ’AB A C AC A B= = . Do A, B và C1, C2

cùng đối xứng với nhau qua trung điểm của AB nên

2 1 BC AC= . Tương tự: 2 1 CB AB= . Suy ra:

2 1

2 1

'

'

BC AC AC A B

CB AB AB A C= = = .

Kết hợp với � �3 3' 'C BA B CA= ( cùng chắn cung AA’)

ta được: � � � �2 2 2 2 2 2' ' ( . . ) ' ' ' 'C BA B CA c g c BC A CB A AC A AB A∆ ∆ ⇒ = ⇒ = ∼ .

Do đó, tứ giác AC2B2A’ là tứ giác nội tiếp hay A’ trùng với A3. Gọi G là giao điểm của

trung tuyến AM với A1A3. Do AA3 // A1M nên: 3

1

2AAAG

GM A M= = ⇒G là trọng tâm của tam giác

ABC hay đường thẳng A1A3 đi qua trọng tâm G của tam giác ABC.

Tương tự: 1 3 1 3, B B C C cũng đi qua G.

Vậy các đường thẳng 1 3 1 3 1 3, , A A B B C C đồng quy. Ta có điều phải chứng minh.

3

3

3

2

2

2

1 1

1

O

B

A

C

C

C

B

AA

B

A

B C

1

2

2 1

1

G

A'

B

C

C

B

MA

A

B C

Page 149: Final Version 1989 to 2010 VN TST

60

Bài 2.

Cho đa thức 3 2( ) 1P x rx qx px= + + + trong đó , , p q r là các số thực và 0r > .

Xét dãy số sau: 2

1 2 3

3 2 1

1, ,

. . . , 0 n n n n

a a p a p q

a p a q a r a n+ + +

= = − = −

= − − − ≥

Chứng minh rằng: nếu đa thức ( )P x có một nghiệm thực duy nhất và không có

nghiệm bội thì dãy số ( )na có vô số số âm.

* Giả sử k là một nghiệm (thực hoặc phức) của đa thức:

3 2( )Q x x px qx r= + + + , do r > 0 nên 3 20 0k k pk qk r≠ ⇒ + + + = (*)

Theo giả thiết, đa thức 3 2( ) 1P x rx qx px= + + + có đúng một nghiệm thực nên nó còn có thêm

hai nghiệm phức liên hợp nữa, đồng thời 1

k chính là nghiệm của P(x) do:

3 2 2 3

3

1 1 1 1( ) 1 0

r qk pk kP r q p

k k k k k

+ + + = + + + = =

.

Xét dãy số (un) xác định bởi công thức:

1 3 2 1( )n n n n

ru a p k a a

k+ + + += + + − (**)

Mặt khác, theo giả thiết: 3 2 1 , 0,1, 2,... n n n na pa qa ra n+ + += − − − =

Ta có:

1 2 1 2 1 2 1

2 12

( ) .

( . )

n n n n n n n n n

n n n

r kq ru pa qa ra p k a a ka a ra

k kkq r r

k a a ak k

+ + + + + + +

+ +

+= − − − + + − = − −

+= − −

Từ (*) 2 32

( )kq r

kq r pk k p kk

+⇒ − + = + ⇒ − = + , do đó:

1 2 1( ( ) ) , 0,1,2,... n n n n n

ru k a p k a a ku n

k+ + += + + − = =

Trong (**), cho n = -1, ta có: 2 3

2 20 2 1 0( ) ( )

r r pk qk r ku a p k a a p q p k p k

k k k k

+ += + + − = − − + − = − = =

Suy ra: 2 22 1( ) , 0,1, 2,... n n

n n n n

ru k a p k a a k n

k+ +

+ += ⇒ + + − = = (***)

Page 150: Final Version 1989 to 2010 VN TST

61

Giả sử z là nghiệm phức của phương trình P(x) = 0 và ,ρ θ lần lượt là modun và argument

của z trong đó: , , 0ρ θ ρ∈ >ℝ .

Ta có: (cos sin )iz e iθρ ρ θ θ= = + và ( ) [ ]P x x∈ℝ nên: ( ) 0 ( ) 0 ( ) 0P z P z P z= ⇒ = ⇒ = do đó:

(cos sin )iz e iθρ ρ θ θ−= = − cũng là nghiệm của P(x).

Từ (***), ta được: 2 22 1 2 1( ) ( ) , ( ) ( ) n n

n n n n n n

r ra p z a a z a p z a a z

z z+ +

+ + + ++ + − = + + − = .

Theo công thức Moavre, ta có: (cos sin ) (cos sin )n nz i z n i nρ θ θ ρ θ θ= + ⇒ = + nên:

( ) [ ] [ ]

[ ]( ) [ ] [ ]

22 2 2

22 2

2 1

cos( 2) sin( 2) cos( 2) sin( 2)

2 . sin ( 2) sin ( 2)2 . sin ( 2) .

2 . sin sin

nn n n

nn n

n n

z z n i n n i n

z z i n ni n

iz z

ρ θ θ ρ θ θ

ρ θ θρ θ ρ

ρ θ θ

++ + +

++ +

+ +

− = + + + − + − + =

− + += + ⇒ = =

Trừ từng vế hai đẳng thức: 2 21

1 1( ) ( ) ( ) ( )n n

n nz z a r a z zz z

+ ++− − − = −

[ ]

2 22 2

1 1 2

11 2

( )( ) ( ) ( )

. ( )

sin ( 2).

sin

n nn n

n n n n

nn n

z z r z zz z a r a z z a a

z z z z

nra a

ρ

θρ

ρ θ

+ ++ +

+ +

++

− −⇔ − − = − ⇔ + =

+⇔ + =

Do 0ρ > nên xét n0 là một giá trị nguyên dương sao cho:

[ ] [ ]0

0 0

0 011 2

sin ( 2) sin ( 2)0 . 0 0

sin sinn

n n

n n ra a

θ θρ

θ θ ρ+

+

+ +< ⇒ < ⇒ + < .

Vì 2

0r

ρ> nên

0 01,n na a+ trái dấu với nhau. Do đó, trong hai giá trị này có một số âm.

Ta thấy khi n tiến tới vô cực, tồn tại vô số giá trị n0 sao cho [ ]0sin ( 2)

0sin

n θθ+

< , mà ứng với

mỗi giá trị n0 như thế ta lại tìm được một số hạng âm của dãy đã cho, tức là dãy (an) có vô số số âm. Đây chính là điều phải chứng minh.

Page 151: Final Version 1989 to 2010 VN TST

62

Bài 3. Cho các số nguyên dương a, b sao cho , a b và .a b đều không là số chính phương.

Chứng minh rằng trong hai phương trình sau:

2 2

2 2

1

1

ax by

ax by

− =

− = −

có ít nhất một phương trình không có nghiệm nguyên dương. * Trước hết ta sẽ chứng minh bổ đề sau: Cho A, B là các số nguyên dương và A, B, AB đều không là các số chính phương.

Khi đó: nếu gọi (a, b) là nghiệm nguyên dương nhỏ nhất của phương trình Pell 2 2 1x ABy− =

(do AB không là số chính phương nên phương trình Pell này luôn có nghiệm nguyên dương, nghĩa là (a, b) tồn tại) và (x0, y0) là nghiệm nguyên dương nhỏ nhất của phương trình

2 2 1Ax By− = thì ta luôn có hệ thức liên hệ sau: 2 20 0

0 02

a Ax By

b x y

= +

=

*Chứng minh:

Do (x0,y0) là nghiệm của 2 2 1Ax By− = nên 2 20 0 1Ax By− = .

Đặt 2 20 0 0 0, 2u Ax By v x y= + = ; khi đó, ta có:

( ) ( ) ( )2 222 2 2 2 2 20 0 0 0 0 0. 2 1u ABv Ax By AB x y Ax By− = + − = − = .

Do đó, (u; v) là một nghiệm của 2 2 1x ABy− = .

Mà (a, b) là nghiệm nguyên dương nhỏ nhất của phương trình Pell 2 2 1x ABy− = nên ,u a v b≥ ≥ .

Ta sẽ chứng minh rằng u = a, v = b. Thật vậy, giả sử ngược lại, u > a, v > b. Ta có:

( )( )( )( )( )

2 2

0 0 0 0

0 0 0 0 0 0

1

( )

a b AB a b AB a b AB a ABb

a b AB Ax B y Ax B y

ax Bby A ay Abx B Ax B y

− < − + = − =

⇒ − + < +

⇔ − + − < +

Mặt khác: ( )22 20 0 0 0 0 02a b AB u v AB Ax By x y AB Ax B y+ < + = + + = +

Do đó:

( ) ( ) ( )( ) ( ) ( )( ) ( )

0 0 0 0 0 0

22 2

0 0 0 0 0 0 0 0 0 0

( )ax Bby A ay Abx B a b AB x A y B

x A y B x A y B Ax By x A y B x A y B

− − − = + − <

− + = − + = +

Đặt 0 0 0 0,s ax Bby t ay Abx= − = − , ta được: 0 0

0 0

(1)

(2)

s A t B x A y B

s A t B x A y B

+ < +

− < +

Page 152: Final Version 1989 to 2010 VN TST

63

Hơn nữa: ( ) ( ) ( )( )2 22 2 2 2 2 20 0 0 0 0 0 1.1 1As Bt A ax Bby B ay Abx a ABb Ax By− = − − − = − − = =

Ta thấy s > 0 vì: 2 2 2 2 2 2 2 2 20 0 0 0 0 0 0 00 0 ( 1)s ax Bby ax Bby a x B b y a x Bb Ax> ⇔ − > ⇔ > ⇔ > ⇔ > −

2 2 2 2 2 20 0( )a ABb x Bb x Bb⇔ − > − ⇔ > − , đúng do B > 0.

Ta cũng thấy 0t ≠ bởi vì nếu t = 0 thì: 2 2 2 2 2 2 2 2 2 2 2

0 0 0 0 0 0 0 0 00 ( 1) ( 1)ay Abx ay Abx a y A b x y ABb Ab By y Ab− = ⇔ = ⇔ = ⇔ + = + ⇔ =

Điều này mâu thuẫn do A không là số chính phương.

-Nếu t > 0 thì (s; t) là một nghiệm dương của phương trình 2 2 1Ax By− = , từ đó:

,s a t b≥ ≥ , suy ra: 0 0s A t B x A y B+ > + , điều này mâu thuẫn với (1).

-Tương tự, nếu t < 0 thì (s; -t) là một nghiệm dương của phương trình 2 2 1Ax By− = , từ đó:

,s a t b≥ − ≥ , suy ra: 0 0s A t B x A y B− > + , điều này mâu thuẫn với (2).

Do đó, điều giả sử là sai, nghĩa là u = a, v = b. Bổ đề được chứng minh. * Trở lại bài toán: Giả sử ngược lại, cả hai phương trình:

2 2

2 2

1 (*)

1 (**)

ax by

ax by

− =

− = −

đều có nghiệm nguyên dương. Gọi (m, n) là nghiệm nguyên dương nhỏ nhất của phương trình 2 2 1x aby− = ; (x1; y1) là nghiệm nguyên dương nhỏ nhất của (*); (x2; y2) là nghiệm nguyên

dương nhỏ nhất của (**). Theo bổ đề trên, ta có các hệ thức sau:

2 21 1

1 12

m ax by

n x y

= +

= và

2 22 2

2 22

m bx ay

n x y

= +

=

Từ (*) 2 21 1 1ax by⇒ = + , từ (**) 2 2

2 2 1ay bx⇒ = − , so sánh các đẳng thức ở trên, ta cũng có:

( )2 2 2 2 2 2 2 21 1 2 2 1 2 1 22 1 2 1 1ax by bx ay by bx b y x+ = + ⇒ + = − ⇔ − = .

Nhưng do b là số nguyên dương, không phải là số chính phương nên b > 1, nghĩa là đẳng thức trên không thể xảy ra. Suy ra điều giả sử ở trên là sai. Vậy trong hai phương trình (*) và (**) đã cho có ít nhất một phương trình không có nghiệm nguyên dương. Đây chính là điều phải chứng minh.

Page 153: Final Version 1989 to 2010 VN TST

64

Bài 4. Tìm tất cả các số thực r sao cho bất đẳng thức sau đúng với mọi số , ,a b c dương:

31

2

a b cr r r r

b c c a a b + + + ≥ + + + +

Ta sẽ xét điều kiện cần và đủ để tìm các giá trị r thỏa đề bài.

* Điều kiện cần: Xét trường hợp 0a b= > . Đặt 0c

ta

= > . Ta có:

3

2

2 3 3

1

2

1 1 1. .

2 2 2 21

a b cr r r r

b c c a a b

a c cr r r r r r

ca c a aa

+ + + ≥ + + + +

⇔ + + ≥ + ⇔ + + ≥ + + +

2 3 2 32

22

1 1 2 1 1. .

1 2 2 1 1 2 2

2 3 1 2 3 1. . 0

1 2 2 ( 1) 1 4 1 8

t r tr r r r r r

t t t

t t tr r

t t t t

⇔ + + ≥ + ⇔ + + + ≥ + + + +

⇔ + − + + − + − ≥ + + + +

Cho 0t → , từ bất đẳng thức trên, suy ra: 2

5 1

44 2 1 0 (*)5 1

4

rr r

r

−≥

+ − ≥ ⇔ − −

* Điều kiện đủ: Ta sẽ chứng minh rằng với giá trị r thỏa mãn (*) thì bất đẳng thức đã cho đúng với mọi số dương

a, b, c. Thật vậy: Đặt: , , , , , 0a b c

x y z x y zb c c a a b

= = = >+ + +

. Ta có:

2 . . . . .

( ) ( ) ( ) 2 ( )( )( )1

( )( )( ) ( )( )( )

a b b c c a a b cxy yz zx xyz

b c c a c a a b a b b c b c c a a bab a b bc b c ca c a abc a b b c c a

a b b c c a a b b c c a

+ + + = + + + =+ + + + + + + + +

+ + + + + + + + += =

+ + + + + +

Ta sẽ chứng minh các bất đẳng thức sau: (1) 2( )

2( )( ) ( )( ) ( )( )

( )( ) ( )( ) ( )( ) 2 ( ) 2 ( ) 2 ( )

( )( )( ) ( )( )( )

x y z xy yz zx

a b c ab bc ca

b c c a a b a c b c b a c a c b a b

a a b a c b b c b a c c a c b ab a b bc b c ca c a

a b b c c a a b b c c a

+ + ≥ + +

⇔ + + ≥ + + + + + + + + + + +

+ + + + + + + + + + + + +⇔ ≥

+ + + + + +

Page 154: Final Version 1989 to 2010 VN TST

65

3 3 3

3 3 3

2 2 2

( ) ( ) ( ) 3 2 ( ) 2 ( ) 2 ( )

3 ( ) ( ) ( )

( ) ( ) ( ) 0

( )( ) ( )( ) ( )( ) 0

a b c ab a b bc b c ca c a abc ab a b bc b c ca c a

a b c abc ab a b bc b c ca c a

a a bc ab ac b b ca ba bc c c ab ca cb

a a b a c b b a b c c c a c b

⇔ + + + + + + + + + ≥ + + + + +

⇔ + + + ≥ + + + + +

⇔ + − − + + − − + + − − ≥

⇔ − − + − − + − − ≥

Bất đẳng thức này chính là bất đẳng thức Schur với các số dương a, b, c.

[ ] [ ]

3(2)

43

( )( ) ( )( ) ( )( ) 4

3( ) ( ) ( ) ( )( )( )

44 ( ) ( ) ( ) 3 ( ) ( ) ( ) 6

( ) ( ) (

xy yz zx

ab bc ca

a c b c b a c a c b a b

ab a b bc b c ca c a a b b c c a

ab a b bc b c ca c a ab a b bc b c ca c a abc

ab a b bc b c ca c

+ + ≥

⇔ + + ≥+ + + + + +

⇔ + + + + + ≥ + + +

⇔ + + + + + ≥ + + + + + +

⇔ + + + + +2 2 2

) 6

( ) ( ) ( ) 0

a abc

a b c b c a c a b

⇔ − + − + − ≥

Bất đẳng thức cuối đúng nên (2) đúng với mọi số dương a, b, c.

Đặt 3

4 t xy yz zx t= + + ⇒ ≥ . Bất đẳng thức đã cho chính là:

3

3 2 3 2

2

1( )( )( ) ( )

23 3 1

( ) ( )2 4 8

3 3 1( ) ( ) 0

2 4 8

r x r y r z r

r x y z r xy yz zx r xyz r r r

x y z r xy yz zx r xyz

+ + + ≥ +

⇔ + + + + + + + ≥ + + +

⇔ + + − + + + − + − ≥

Do 2( )x y z xy yz zx+ + ≥ + + và 1

2 1 (1 )2

xy yz zx xyz xyz xy yz zx+ + + = ⇔ = − − − nên để chứng

minh bất đẳng thức trên, ta chứng minh bất đẳng thức sau:

2

2

2 2

2 22

3 3 1 12( ) ( ) (1 ) 0

2 4 2 8

3 3 1 1(2 ) ( ) (1 ) 0

2 4 2 81 3 3 3

(2 )2 2 4 8

4 2 1 3(4 2 1) 3. (4 2 1) 0

2 8 4

xy yz zx r xy yz zx r xy yz zx

t r t r t

t r r r r

r r r rt r r t

⇔ + + − + + + − + − − − − ≥

⇔ − + − + − − ≥

⇔ + − ≥ + −

+ − + − ⇔ ≥ ⇔ + − − ≥

Bất đẳng thức cuối đúng do (*) và (2) nên bất đẳng thức đã cho là đúng.

Vậy điều kiện của r cần tìm là 5 1

4r

−≥ hoặc

5 1

4r

− −≤ .

Page 155: Final Version 1989 to 2010 VN TST

66

Bài 5. Cho đường tròn (O) có đường kính AB và M là một điểm bất kì nằm trong (O), M

không nằm trên AB. Gọi N là giao điểm của phân giác trong góc M của tam giác AMB với đường tròn (O). Đường phân giác ngoài góc AMB cắt các đường thẳng NA, NB lần lượt tại P, Q. Đường thẳng MA cắt đường tròn đường kính NQ tại R, đường thẳng MB cắt đường tròn đường kính NP tại S và R, S khác M.

Chứng minh rằng: đường trung tuyến ứng với đỉnh N của tam giác NRS luôn đi qua một điểm cố định khi M di động phía trong đường tròn.

* Qua R kẻ đường thẳng song song với PQ cắt NA tại C, qua S kẻ đường thẳng song song với PQ cắt NB tại D. Gọi I là trung điểm của CD . Ta sẽ chứng minh rằng CD // AB.

Thật vậy, do N nằm trên đường tròn đường kính AB nên: � 090ANB AN BN= ⇒ ⊥ , suy ra BN là tiếp tuyến của đường tròn đường kính PN.

Do đó: ( . )BMN BNS g g∆ ∆∼

Vì PQ là đường phân giác góc ngoài của AMN nên: � � � �SMP AMP QMR BMQ= = = .

Mặt khác: � �SMP SNP= (góc nội tiếp cùng chắn cung PS của đường tròn đường kính PN),

� �QMR QNR= ( góc nội tiếp cùng chắn cung QR của đường tròn đường kính QN).

Do đó: � � � � � � � �SNP QNR SNP SNR QNR SNR CNR SNB= ⇒ + = + ⇒ = .

Xét hai tam giác ∆ BNS và ∆ RNC có: � �CNR SNB= và � � � �RCN MPN NSM NSB= = = nên: ( . )BNS RNC g g∆ ∆∼ .

Suy ra các tam giác đồng dạng: BMN BNS RNC∆ ∆ ∆∼ ∼ .

CD

I

S

R

MP

Q

N

BOA

Page 156: Final Version 1989 to 2010 VN TST

67

Tương tự, ta cũng có: DSN RAN NAM∆ ∆ ∆∼ ∼ .

* Ta thấy, từ: . .NB NS

BNS RNC NB NC NR NSNR NC

∆ ∆ ⇒ = ⇒ =∼

. .NS ND

DSN RAN NA ND NR NSNA NR

∆ ∆ ⇒ = ⇒ =∼ .

Suy ra: . .NA NC

NA ND NB NCNB ND

= ⇒ = ⇒ AB // CD

⇒Trung điểm của AB, trung điểm của CD và N là ba điểm thẳng hàng. Tức là N, O, I thẳng hàng. (1)

* Hơn nữa: .MN BN NB NC

BMN RNC RCNC RC MN

∆ ∆ ⇒ = ⇒ =∼ .

.DN DS NA ND

DSN NAM DSMN NA MN

∆ ∆ ⇒ = ⇒ =∼ .

Kết hợp các điều trên, ta được: RC = DS, mà RC // DS (cùng song song với PQ) nên tứ giác RCSD là hình bình hành. Do đó, hai đường chéo CD và RS của tứ giác cắt nhau tại trung điểm của mỗi đường. Suy ra I là trung điểm của CD cũng là trung điểm của RS. Khi đó: NI chính là đường trung tuyến của tam giác NRS. (2) Từ (1) và (2), suy ra: trung tuyến NI của tam giác NRS luôn đi qua O. Vậy trung tuyến ứng với đỉnh N của tam giác NRS luôn đi qua I là điểm cố định khi M di động khắp phía trong đường tròn (O). Đây chính là điều phải chứng minh.

Page 157: Final Version 1989 to 2010 VN TST

68

Bài 6. Một hội nghị toán học có tất cả 6 4n + nhà toán học phải họp với nhau đúng 2 1n + lần ( )1n ≥ . Mỗi lần họp, họ ngồi quanh một cái bàn 4 chỗ và n cái bàn 6 chỗ, các vị trí ngồi

chia đều khắp mỗi cái bàn. Biết rằng hai nhà toán học đã ngồi cạnh hoặc đối diện nhau ở một cuộc họp này thì sẽ không được ngồi cạnh hoặc đối diện nhau ở một cuộc họp khác. a/ Chứng minh rằng Ban tổ chức có thể xếp được chỗ ngồi nếu 1n = . b/ Hỏi rằng Ban tổ chức có thể sắp xếp được chỗ ngồi được hay không với mọi 1n > ? a. Với 1n = , ta có bài toán như sau: một hội nghị toán học có 10 nhà toán học, họ phải họp với nhau đúng 3 lần và trong mỗi lần họp như thế, họ phải ngồi quanh một cái bàn 4 chỗ và một cái bàn 6 chỗ, các vị trí ngồi chia đều khắp bàn; đồng thời, hai người đã ngồi kề nhau hoặc đối diện nhau trong cuộc họp này thì không được ngồi kề nhau hoặc đối diện nhau trong một cuộc họp khác. Đánh số thứ tự cho 10 nhà toán học đang xét là (1), (2), (3), …(10). Ta sẽ chỉ ra một cách sắp xếp thỏa mãn đề bài trong trường hợp này. Ta có các sơ đồ sau: -Buổi họp thứ 1:

-Buổi họp thứ 2: -Buổi họp thứ 3:

7

8

9

6 10

42

1

3

5

3

8

4

6 10

95

1

7

2

7

3

9

2 4

106

1

8

5

Page 158: Final Version 1989 to 2010 VN TST

69

b. Ta sẽ chứng minh rằng trong trường hợp tổng quát, Ban tổ chức luôn có thể xếp chỗ ngồi cho các nhà toán học trong các cuộc họp. Ta chia 6 4n + nhà toán học thành 2 2n + “nhóm”. Một “nhóm” chỉ gồm 1 người luôn ngồi ở một vị trí cố định tại bàn 4 chỗ, đặt người này là X0; 2 1n + “nhóm” còn lại chia ra từ 6 3n + nhà toán học, mỗi “nhóm” có 3 nhà toán học.

Đặt các “nhóm” đó là 1 2 3 2 1, , ,..., nX X X X + . Các “nhóm” này sẽ lần lượt ngồi vào các vị trí còn lại

của bàn 4 chỗ cùng với X0, mỗi “nhóm” ngồi đúng một lần. * Với các bàn 6 chỗ, ta có cách sắp xếp như sau:

Ở bước thứ , 1 2 1k k n≤ ≤ + , với hai “nhóm” bất kì , i jX X trong đó: (mod 2 1), i j k n+ ≡ +

1 , 2 1, i j n i j≤ ≤ + ≠ thì các nhà toán học thuộc hai “nhóm” Xi, Xj sẽ ngồi vào họp cùng nhau ở

một bàn 6 chỗ nào đó; đồng thời, những nhà toán học thuộc cùng một “nhóm” thì ngồi ở các vị trí tạo thành một tam giác đều trên các bàn 6 chỗ, nghĩa là họ sẽ không rơi vào trường hợp ngồi đối diện nhau hoặc ngồi cạnh nhau.

* Với bàn 4 chỗ, ta có cách sắp xếp như sau:

- Nếu k là số chẵn thì ở bước này, chỉ có một “nhóm” có chỉ số 2

k là không được ngồi

chung bàn 6 chỗ với nhóm nào, như thế “nhóm” này sẽ ngồi vào bàn 4 chỗ cùng với X0.

- Nếu k là số lẻ, ở bước này; tương tự trên, chỉ có một “nhóm” có chỉ số là 2 1

2

k n+ +

không được chung bàn 6 chỗ với “nhóm” nào, “nhóm” này sẽ ngồi vào bàn 4 chỗ cùng với X0. Dễ dàng thấy rằng cách sắp xếp như thế thỏa mãn mọi yêu cầu của bài toán. Vậy Ban tổ chức có thể sắp xếp được chỗ ngồi mọi 1n > .

j

i

X

X

Page 159: Final Version 1989 to 2010 VN TST

70

LỜI GIẢI ĐỀ THI CHỌN ĐỘI TUYỂN QUỐC GIA

DỰ THI IMO 2010

Bài 1. Cho tam giác ABC không vuông tại A có đường trung tuyến AM. Gọi D là một điểm di

động trên đường thẳng AM. Gọi ( ) ( )1 2,O O là các đường tròn đi qua D, tiếp xúc với BC lần

lượt tại B và C. Gọi P, Q lần lượt là giao điểm của đường thẳng AB với đường tròn (O1), đường thẳng AC với đường tròn (O2). Chứng minh rằng:

1. Tiếp tuyến tại P của 1( )O và tiếp tuyến tại Q của 2( )O phải cắt nhau tại một điểm.

Gọi giao điểm đó là S. 2. Điểm S luôn di chuyển trên một đường thẳng cố định khi D di động trên AM.

1. Vì M là trung điểm của BC nên 2 2MA MB= , suy ra M có cùng phương tích đến hai

đường tròn ( ) ( )1 2, O O hay M thuộc trục đẳng phương của .

Hơn nữa, hai đường tròn cắt nhau tại D nên D cũng nằm trên trục đẳng phương của chúng.

Từ đó, suy ra DM chính là trục đẳng phương của ( ) ( )1 2,O O , mà A thuộc đường thẳng

DM nên A có cùng phương tích đến hai đường tròn ( ) ( )1 2,O O .

Suy ra: . .AP AB AQ AC= hay tứ giác BPQC nội tiếp.

Từ hệ thức trên ta cũng thấy rằng nếu P trùng với A thì Q cũng trùng với A, nếu P thuộc đoạn AB thì Q cũng thuộc đoạn AC và ngược lại.

x

S

Q

P

O2

O1

M

A

B C

D

Page 160: Final Version 1989 to 2010 VN TST

71

Không mất tính tổng quát, giả sử góc�ABC nhọn, gọi Px là tia tiếp tuyến của đường tròn

(O1) sao cho góc �xPB nhọn.

Theo tính chất tiếp tuyến, ta có: � �xPB PBC= , mà � �PBC AQP= nên � �xPB AQB= .

⇒Px cũng là tiếp tuyến của đường tròn (APQ). Do đó: (O1) tiếp xúc với đường tròn (APQ). Hoàn toàn tương tự: ta cũng có (O2) cũng tiếp xúc với đường tròn (APQ).

Suy ra: tiếp tuyến tại P của 1( )O và tiếp tuyến tại Q của 2( )O cũng chính là hai tiếp tuyến của

(APQ) tại các điểm P, Q.

Hơn nữa, theo giả thiết: � 090PAQ ≠ nên hai tiếp tuyến đó không song song và do đó

chúng phải cắt nhau (đpcm). 2. Theo chứng minh ở trên, ta có: S thuộc tiếp tuyến của (O1) và (O2), SP SQ= nên S có

cùng phương tích đến hai đường tròn ( ) ( )1 2,O O nên S thuộc trục đẳng phương của hai đường

tròn này, nghĩa là S nằm trên AM. Vậy khi D thay đổi trên AM thì S cũng di chuyển trên AM là đường thẳng cố định. Ta có đpcm.

Page 161: Final Version 1989 to 2010 VN TST

72

Bài 2. Với mỗi số n nguyên dương, xét tập hợp sau :

{ }11( ) 10( ) |1 , 10k hnT k h n n k h= + + + ≤ ≤ .

Tìm tất cả giá trị của n sao cho không tồn tại , ; na b T a b∈ ≠ sao cho ( )a b− chia hết cho 110.

Đặt ( , , ) 11( ) 10( ), , ,k hf k h n k h n n k h n= + + + ∈ℕ .

Ta có: ( , , ) ( , , )f k h n f h k n= nên không mất tính tổng quát, ta giả sử h k≥ .

Nếu (mod11)m n≡ thì :

( , , ) ( , , ) 11( ) 10( ) 11( ) 10( )

10 ( ) ( ) 110 ( , , ) ( , , ) (mod110)

k h k h

k k h h

f k h m f k h n k h m m k h n n

m n m n f k h m f k h n

− = + + + − + + + =

= − + − ⇒ ≡ ⋮

Từ đó, ta chỉ cần xét các giá trị n thỏa 1 11n≤ ≤ .

Xét hiệu: 5(6,6, ) (1,1, ) 110 20 .( 1)f n f n n n− = + − . Nếu 520 .( 1)n n − chia hết cho 110 thì

giá trị n tương ứng sẽ không thỏa. Từ đó, ta loại đi các giá trị 1,3,4,5,9,11n = .

Ta cũng có 8 2 6 4 2 2 2 2(8,2, ) (6,4, ) 10( ) 10 ( 1) ( 1)f n f n n n n n n n n− = + − − = − + , với 10n =

thì 2 2 2 210 ( 1) ( 1) 110n n n− + ⋮ nên giá trị này cũng không thỏa.

Ta sẽ chứng minh rằng các giá trị 2,6,7,8n = thỏa mãn. Thật vậy:

Tính toán trực tiếp, ta thấy rằng với 2,6,7,8n = thì (mod11)k hn n≠ với (mod11)k h≠ .

Giả sử ngược lại, với các giá trị n nêu trên, tồn tại hai bộ ( , ) ( ', ')k h k h≠ (giả sử 'k k> ) sao cho:

( , , ) ( ', ', )f k h n f k h n≠ . Khi đó: ' '11( ' ') 10( ) 0 (mod110)k h k hk h k h n n n n+ − − + + − − ≡ .

Suy ra: ' ' ' ' '' ' (mod10), (mod11) ( 1) ( 1) (mod11)k h k h k k k h h hk k h h n n n n n n n n− −− ≡ − + ≡ + ⇔ − ≡ − .

Do 11 là số nguyên tố nên theo định lí Fermat nhỏ thì: ' ' (mod11)k k h hn n− −≡ .

Dễ thấy: ' '1 1 0 (mod11)k k h hn n− −− ≡ − ≠ nên từ đẳng thức trên, suy ra: ' (mod11) 'k hn n k h≡ ⇒ = .

Do đó: 'k h= hay ( , ) ( ', ')k h k h≡ , mâu thuẫn.

Suy ra các giá trị 2,6,7,8n = đều thỏa mãn yêu cầu của đề bài.

Vậy tất cả các giá trị n cần tìm là: 2,6,7,8 (mod11)n ≡ .

Page 162: Final Version 1989 to 2010 VN TST

73

Bài 3. Gọi một hình chữ nhật có kích thước 1 2× là hình chữ nhật đơn và một hình chữ nhật có kích thước 2 3× , bỏ đi 2 ô ở góc chéo nhau (tức có có 4 ô vuông con) là hình chữ nhật kép. Người ta ghép khít các hình chữ nhật đơn và hình chữ nhật kép này lại với nhau được một bảng hình chữ nhật có kích thước là 2008 2010× . Tìm số nhỏ nhất các hình chữ nhật đơn có thể dùng để ghép.

Ta sẽ chứng minh rằng số hình chữ nhật đơn nhỏ nhất thỏa mãn đề bài là 1006. Thật vậy: *Điều kiện cần: Ta xét một cách phủ hình chữ nhật 2008 2010× thỏa mãn đề bài (chú ý rằng 2008 chỉ số hàng và 2010 chỉ số cột). Gọi , , ,x y z t là số các hình chữ nhật 1 2, 2 1, 2 3, 3 2× × × × trong cách

phủ đó (ở đây thực ra các hình chữ nhật 1 2, 2 1× × đều là các hình chữ nhật đơn của đề bài, chỉ

phân biệt ở cách phủ dọc hay ngang, trong đó 1 2× được đặt ngang, 2 1× được đặt dọc; tương tự với cách phân biệt 2 3, 3 2× × ).

Tô màu trắng cho các hàng lẻ, tô màu đen cho các hàng chẵn. Ở tất cả các ô của hàng thứ ,1 2010i i≤ ≤ , ta đánh số tương ứng các số tự nhiên i.

Ta sẽ chứng minh các nhận xét sau: - Nhận xét 1: ta luôn có đẳng thức: 2( ) 4( ) 2008.2010x y z t+ + + = .

Mỗi hình chữ nhật 1 2× hoặc 2 1× có chứa hai ô vuông, mỗi hình chữ nhật 2 3× hoặc 3 2× có chứa bốn ô vuông. Tổng các ô vuông này bằng số ô vuông của cả hình chữ nhật lớn nên 2( ) 4( ) 2008.2010x y z t+ + + = .

- Nhận xét 2: Giá trị ,x y là chẵn.

Ta thấy trên toàn bảng, các hình chữ nhật 2 3, 3 2× × đều có số ô trắng bằng số ô đen; các

hình chữ nhật 2 1× được đặt dọc nên cũng có số ô trắng bằng số ô đen. Suy ra, số hình chữ nhật 1 2× ở các hàng được tô màu trắng bằng số hình chữ nhật ở các hàng được tô đen. Hơn nữa, tổng số các hàng là 2010 là chẵn nên giá trị x phải là chẵn. Từ nhận xét 1, ta thấy y cũng phải chẵn. Trở lại bài toán, ta xét tương ứng Φ đi từ tập hợp các hình chữ nhật đang xét đến các số nguyên là hiệu giữa tổng các số ở ô vuông được tô đen với tổng các số ở ô vuông được tô trắng ghi trên nó. Dễ dàng thấy rằng: (3 2) 0; (2 3) 2; (2 1) 1Φ × = Φ × = ± Φ × = ± .

Từ đó suy ra: (3 2) 0; (2 3) 2 ; (2 1)z yΦ × = Φ × ≤ Φ × ≤∑ ∑ ∑ .

(kí hiệu (3 2)Φ ×∑ là tổng tính trên tất cả các hình chữ nhật 3 2× được dùng, định nghĩa tương

tự với các hình chữ nhật khác).

Ta cũng thấy rằng, tổng các số ghi trên x hình chữ nhật 1 2× là một số chẵn thuộc [ ]2; 2.2008 ,

mà x là số chẵn (nhận xét 2), ta có đánh giá sau: ( )(1 2) 2.2008 22

xΦ × ≤ −∑ .

Page 163: Final Version 1989 to 2010 VN TST

74

Ta có: [ ]1004 1004

1 1

(2008 2010) 2010. 2 (2 1) 2010. 2010.1004i i

i i i= =

Φ × = − − = =∑ ∑ .

Mặt khác: (2008 2010) (2 1) (1 2) (3 2) (2 3)Φ × = Φ × + Φ × + Φ × + Φ ×∑ ∑ ∑ ∑ .

Từ các điều trên, suy ra: 2010.1004 .(2.2008 2) 2 2010.1004 2007 22

xy z x y z≤ − + + ⇔ ≤ + + .

Tiếp theo, ta xét hình chữ nhật 2010 2008× (tương tự như trên nhưng có 2010 hàng và 2008 cột), bắt đầu lại các lập luận về số các hình chữ nhật 1 2, 2 1, 2 3, 3 2× × × × được dùng.

Ta xây dựng được bất đẳng thức sau: 2008.1005 2009 2y x t≤ + + .

Cộng hai bất đẳng thức này lại, ta có: 2008.1005 2010.1004 (2009 2 ) (2007 2 ) 2008 2010 2( )y x t x y z x y z t+ ≤ + + + + + = + + + .

Hơn nữa, theo nhận xét 1 thì: 2010.1004 ( ) 2( )x y z t= + + + .

Từ đó ta được: 2008.1005 2007 2009 2009( )x y x y≤ + ≤ + .

Suy ra: 2008

1005. 10042009

x y+ ≥ > , mà x y+ là số chẵn nên 1006x y+ ≥ .

Do đó, tổng các hình chữ nhật đơn cần dùng ít nhất là 1006. Điều kiện cần được chứng minh. *Điều kiện đủ: Ta sẽ chỉ ra một cách ghép hình chữ nhật dùng đúng 1006 hình chữ nhật đơn.

Khối 4Khối 3Khối 2Khối 1

Page 164: Final Version 1989 to 2010 VN TST

75

Hình trên mô tả cách ghép một hình chữ nhật 10 16× , trong đó: các hình chữ nhật khuyết được tô bằng 5 màu khác nhau (đỏ, hồng, xanh lam, xanh lá cây, xanh đậm) để dễ dàng phân biệt; trên hình các khối được tô màu xanh lá mạ là các hình chữ nhật đơn chắc chắn phải dùng, các khối màu vàng thì tùy trường hợp, có thể là hình chữ nhật đơn mà cũng có thể là hình chữ nhật khuyết. * Hình chữ nhật 2010 2008× có thể được tạo thành từ hình trên bằng quy tắc sau:

- Thêm các dòng bằng cách chèn vào giữa mỗi khối ở trên các hình có dạng:

Mỗi lần ghép như thế thì ta có thêm được hai hàng mới, do 2010 chia hết cho 2 nên khi thực hiện việc này liên tiếp một cách thích hợp thì khối này sẽ tăng về chiều dài, tạo thành các khối mới có kích thước 2010 4× và ở mỗi khối như vậy, ta chỉ dùng đúng 2 hình chữ nhật màu xanh lá mạ. - Thêm cột bằng cách lặp lại các khối 1, 2 , 3, 4 ở trên hình (chú ý tính tuần hoàn giữa các khối: (1) tương ứng với (3), (2) tương ứng với (4)). Như thế thì ta cần phải có tất cả 502 khối dành cho 2008 cột. Đồng thời, ở khối đầu tiên và khối cuối cùng, ta cần dùng thêm một hình chữ nhật đơn màu vàng, các khối ở giữa thì dùng các hình chữ nhật khuyết màu vàng. Tức là: ở hai khối đầu tiên và cuối cùng, ta cần dùng 3 hình chữ nhật đơn, các khối ở giữa chỉ cần dùng 2 hình chữ nhật đơn thôi. Khi đó, tổng số hình chữ nhật đơn cần dùng là: 500.2 2.3 1006+ = . Xoay hình chữ nhật 2010 2008× lại, ta được hình chữ nhật 2008 2010× cần phải ghép, hình chữ nhật đó có đúng 1006 hình chữ nhật đơn thỏa mãn đề bài. Do đó, điều kiện đủ được chứng minh. Vậy giá trị nhỏ nhất các hình chữ nhật đơn cần dùng là 1006. Bài toán được giải quyết hoàn toàn.

Page 165: Final Version 1989 to 2010 VN TST

76

Bài 4.

Cho , ,a b c là các số thực dương thỏa mãn điều kiện: 1 1 1

16( )a b ca b c

+ + ≥ + + .

Chứng minh rằng:

3 3 3

1 1 1 8

9( 2( )) ( 2( )) ( 2( ))a b a c b c b a c a c b+ + ≤

+ + + + + + + + +.

Hỏi đẳng thức xảy ra khi nào? Theo bất đẳng thức Cauchy, ta có:

2

33( )( )

2( ) ( ) 3 ( ). 32 2 2 2

a c a c a c a b a ca b a c a b a b

+ + + + ++ + + = + + + ≥ + =

( )3

1 2

27( )( )2( ) a b a ca b a c⇒ ≤

+ ++ + +.

Tương tự với hai biểu thức còn lại. Do đó:

( )3

1 2 4( )

27( )( ) 27( )( )( )2( )cyc sym

a b c

a b a c a b b c c aa b a c

+ +≤ =

+ + + + ++ + +∑ ∑ .

Hơn nữa, ta thấy với mọi a, b, c dương:

29( )( )( ) 8( )( ) ( ) 0

8( )( )( ) ( )( )

9

sym

a b b c c a a b c ab bc ca a b c

a b b c c a a b c ab bc ca

+ + + − + + + + = − ≥

⇒ + + + ≥ + + + +

Do đó:

( )3

1 1

6( )2( )cyc ab bc caa b a c≤

+ ++ + +∑ . (1).

Mặt khác, ta cũng có: 2( ) 3 ( )ab ca ca abc a b c+ + ≥ + + nên theo giả thiết:

1 1 1 3( ) 316( )

16

ab bc ca a b ca b c ab bc ca

a b c abc ab bc ca

+ + + ++ + ≥ + + = ≥ ⇒ + + ≥

+ +. (2)

Từ (1) và (2), suy ra: 3 3 3

1 1 1 8

9( 2( )) ( 2( )) ( 2( ))a b a c b c b a c a c b+ + ≤

+ + + + + + + + +.

Ta có đpcm. Đẳng thức xảy ra khi dấu bằng ở tất cả các bất đẳng thức trên xảy ra hay:

, , 01

41 1 1

16( )

a b c

a b c a b c

a b ca b c

>

= = ⇒ = = = + + = + +

.

Page 166: Final Version 1989 to 2010 VN TST

77

Bài 5.

Trong một hội nghị có n nước tham gia, mỗi nước có k đại diện ( 1n k> > ). Người ta chia n.k người này thành n nhóm, mỗi nhóm có k người sao cho không có hai người nào cùng nhóm đến từ cùng một nước. Chứng minh rằng có thể chọn ra một nhóm gồm n người sao cho họ thuộc các nhóm khác nhau và đến từ các nước khác nhau. Ta gọi một nước X và một nhóm Y nào đó có liên hệ với nhau nếu trong nhóm Y có người của nước X. Khi đó, một nước X bất kì có k người đại diện nên có liên hệ với k nước và một nhóm Y bất kì có chứa k người đại diện khác nhau của các nước khác nhau nên có liên hệ với đúng k nước. Do đó, một tập hợp bất kì m nước nào đó trong n nước đã cho sẽ có liên hệ với

ít nhất .m k

mk

= nhóm khác nhau.

Gọi , 1,ia i n= là n nhóm đã cho và ,1,iX n là tập hợp các nhóm có liên hệ với nước thứ i.

Theo điều vừa chứng minh ở trên, ta thấy với mọi:

1 2 3, , ,..., {1,2,3,..., }ki i i i n∈ , 1 k n≤ ≤ thì: 1

j

k

ij

X k=

≥∪ . (*)

Ta sẽ chứng minh rằng tồn tại 1 2 3 1 2 3( , , ,..., ) ...n na a a a X X X X∈ × × × × và ,i ja a i j≠ ≠ .

Thật vậy, ta bắt đầu bỏ đi phần tử thuộc mỗi tập Xi sao cho (*) vẫn được thỏa mãn. Cuối

cùng thu được các tập hợp mới 1 2 3' , ' , ' ,..., 'nX X X X (với ' )i iX X⊂ vẫn thỏa điều kiện (*) có số

phần tử nhỏ nhất mà nếu bỏ đi thêm bất cứ phần tử thuộc tập hợp ' ,1,iX n thì điều kiện (*) sẽ

không còn được thỏa.

Ta chứng minh rằng ' 1, 1,iX i n= = .

Thật vậy, không mất tính tổng quát giả sử 1'X có chứa phần tử khác nhau là ,α β . Do nếu bỏ

thêm một trong hai phần tử α hoặc β thì điều kiện (*) không còn thỏa mãn nên sẽ có hai tập chỉ

số P, Q sao cho:

Với 1( ' \{ }) 'ii P

M X Xα∈

= ∪∪ , 1( ' \{ }) 'ii Q

N X Xβ∈

= ∪∪ không thỏa mãn điều kiện (*), tức là:

1, 1 ,M P N Q M P N Q< + < + ⇒ ≤ ≤ .

Ta có:

1 1 1(( ' \{ }) ( ' \{ })) ( ' ' ) ' 'i i ii P i Q i P Q

M N X X X X X Xα β∈ ∈ ∈ ∪

∪ = ∪ ∪ ∪ = ∪∪ ∪ ∪

'ii P Q

X M N∈ ∩

⊆ ∩∪

Page 167: Final Version 1989 to 2010 VN TST

78

Từ hai điều này suy ra: 1M N P Q∪ ≥ ∪ + , M N P Q∩ ≥ ∩

Theo nguyên lí bù trừ, ta có:

1 1P Q M N M N M N P Q P Q P Q+ ≥ + = ∪ + ∩ ≥ ∩ + ∪ + = + +

Điều vô lí này dẫn đến khẳng định ' 1, 1,iX i n= = .

Rõ ràng các tập hợp này không giao nhau vì nếu tồn tại

, ( ' ' ) ' ' 1 2i j i ji j X X X X≠ ∩ ≠ ∅ ⇒ ∩ = < , mâu thuẫn với điều kiện (*).

Giả sử mỗi phần tử của 'iX chính là 'ia thì tập hợp sau: 1 2 3( ' , ' , ' ,..., ' )na a a a thỏa mãn

' 'i i ia X X∈ ⊂ và ,i ja a i j≠ ≠ .

Vậy ta đã chỉ ra rằng tồn tại n nhóm khác nhau tương ứng liên hệ với n nước khác nhau, mỗi nhóm đó liên hệ với đúng một nước và mỗi nước liên hệ đúng một nhóm nên n người đại diện mà mỗi nước liên hệ với mỗi nhóm tương ứng rõ ràng thỏa mãn đề bài. Ta có đpcm.

Page 168: Final Version 1989 to 2010 VN TST

79

Bài 6.

Gọi nS là tổng bình phương các hệ số trong khai triển của nhị thức (1 )nx+ , trong đó

n là số nguyên dương; x là số thực bất kì.

Chứng minh rằng: 2 1nS + không chia hết cho 3 với mọi n.

Ta sẽ chứng minh bổ đề sau (định lí Lucas):

“Cho m, n là hai số tự nhiên và p là một số nguyên tố. Giả sử:

1 21 2 1 0. . ... .k k

k km m p m p m p m p m−−= + + + + +

1 21 2 1 0. . ... .k k

k kn n p n p n p n p n−−= + + + + +

Khi đó: 0

(mod )i

i

kmm

n ni

C C p=

≡∏ (quy ước rằng 0, abC a b= > ).

*Chứng minh:

Không mất tính tổng quát, giả sử m n> (nếu m n= thì bổ đề hiển nhiên đúng).

Trước hết, ta thấy rằng: ( , ) 1, 1, 1p i i p= = − nên ! ( 1)!

.!( )! !( )!

kp

p pC p p

k p k k p k

−= =

− −⋮ , tức là:

0 (mod ), 1, 1kpC p k p≡ = − .

Ta có: 1

1

( 1) 1 . 1(mod )p

p p i p i pp

i

x x C x x p−

=

+ = + + ≡ +∑ . (*)

Ta sẽ chứng minh nhận xét: *( 1) 1(mod ),j jp px x p j+ ≡ + ∀ ∈ℕ bằng quy nạp. Thật vậy:

- Với 1j = , nhận xét đúng theo (*).

- Giả sử nhận xét này đúng với 1j h= ≥ . Ta sẽ chứng minh rằng nó cũng đúng với 1j h= + .

Ta có: ( 1) 1 (mod )h hp px x p+ ≡ + .

Suy ra: ( ) ( ) 1 1

( 1) 1 (mod ) ( 1) 1 (mod )h h h hh h

p p p px x p x x p+ +

+ ≡ + ⇒ + ≡ + .

Do đó nhận xét đúng với 1j h= + . Theo nguyên lí quy nạp, nhận xét được chứng minh.

Ta xét khai triển sau:

Page 169: Final Version 1989 to 2010 VN TST

80

0

..

00 0

(1 ) (1 ) (1 ) (mod )

ki

ii i ii i

i

mk km pmm p j j p

mji i

x x x C x p=

== =

∑+ = + ≡ + ≡ ∑∏ ∏ .

Hệ số của nx ở vế (1 )mx+ là nmC ; do biểu diễn 1 2

1 2 1 0. . ... .k kk kn n p n p n p n p n−

−= + + + + + là duy

nhất nên hệ số của nx ở vế .

00

i i

i

mkj j p

mji

C x==∑∏ là

0

i

i

kmn

i

C=∏ .

Từ đó ta được: 0

(mod )i

i

kmm

n ni

C C p=

≡∏ .Bổ đề được chứng minh.

*Trở lại bài toán: Ta có: 2

2 22

1 0 0

(1 ) (1 ) .( 1) . . . .nn n n

n n n i n i i n i n i in n n

i i i

x x n C x C x C x− − −

= = =

+ = + + ⇔ =

∑ ∑ ∑ .

Đồng nhất hệ số của 2nx ở hai vế, ta có: 22

0 0

. ( )n n

n i n i in n n n

i i

C C C C−

= =

= =∑ ∑ .

Do đó, với mọi n tự nhiên thì 2n

n nS C= .

Như thế ta cần chứng minh rằng: 24 1n

nC + không chia hết cho 3 với mọi n.

Giả sử: 0

2 .3 , , 1,k

ii i

i

n a a i k=

= ∈ =∑ ℕ . Xét hai trường hợp:

- Nếu {0;1}, 1,ia i k∈ ∀ = thì 2 {0;2}, 1,ia i k∈ ∀ = và tổng {0;1}, 1,ia i k∈ ∀ = là số chẵn, đặt

0 2

0

2 , 2 2 4 1(mod3)

k

ii

k at t

ii

a t t =

=

∑= ∈ ⇒ = = ≡∑ ℕ ; ta cũng có:

0

4 2 .3 , , 1,k

ii i

i

n a a i k=

= ∈ =∑ ℕ .

Theo bổ đề trên thì 024 2

0 0

1 1 2 1 2 1 2(mod 3)

k

ii i i

i

k k aa an

n ai i

C C =

= =

∑+ ≡ + ≡ + ≡ + ≡∏ ∏ .

- Nếu tồn tại một giá trị 2ja = ; không mất tính tổng quát, giả sử đây là số nhỏ nhất trong

tập hợp , 0,ia i k= . Khi đó: hệ số tương ứng tại vị trí j ở khai triển theo lũy thừa 3 của 4n là 1.

Mà 21 0C = nên 2 2

4 2 40

0 (mod3) 1 1 (mod3)i

i

kan n

n a ni

C C C=

≡ ≡ ⇒ + ≡∏ .

Vậy trong mọi trường hợp, ta đều có 2 1nS + không chia hết cho 3. Đây chính là đpcm.

Page 170: Final Version 1989 to 2010 VN TST

81

PHẦN 3 *****

HÌNH ẢNH CỦA

ĐỘI TUYỂN

QUA CÁC NĂM

Page 171: Final Version 1989 to 2010 VN TST

82

*Năm 2005.

*Đội tuyển Việt Nam thi IMO 2005:

1.Trần Trọng Đan

2.Phạm Kim Hùng

3.Nguyễn Nguyên Hùng

4.Đỗ Quốc Khánh

5.Trần Chiêu Minh

6.Nguyễn Trường Thọ

Page 172: Final Version 1989 to 2010 VN TST

83

*Năm 2006.

*Đội tuyển Việt Nam dự thi

IMO 2006:

1. Đặng Bảo Đức

2. Hoàng Mạnh Hùng

3. Nguyễn Duy Mạnh

4. Lê Hồng Quý

5. Nguyễn Xuân Thọ

6. Lê Nam Trường

Page 173: Final Version 1989 to 2010 VN TST

84

*Năm 2007:

*Đội tuyển Việt Nam dự thi IMO 2007:

1. Đỗ Xuân Bách

2. Nguyễn Xuân Chương

3. Lê Ngọc Sơn

4. Phạm Thành Thái

5. Đỗ Ngọc Thanh

6. Phạm Duy Tùng

Page 174: Final Version 1989 to 2010 VN TST

85

*Năm 2008:

*Đội tuyển Việt Nam dự thi IMO 2008:

1. Lê Ngọc Anh

2. Nguyễn Phạm Đạt

3. Dương Trọng Hoàng

4. Đỗ Thị Thu Thảo

5. Đặng Trần Tiến Vinh

6. Hoàng Đức Ý

Page 175: Final Version 1989 to 2010 VN TST

86

*Năm 2009:

*Đội tuyển Việt Nam thi IMO 2009:

1. Hà Khương Duy

2. Nguyễn Xuân Cương

3. Nguyễn Hoàng Hải

4. Phạm Hy Hiếu

5. Phạm Đức Hùng

6. Tạ Đức Thành

Page 176: Final Version 1989 to 2010 VN TST

87

*Năm 2010:

*Đội tuyển Việt Nam thi IMO 2010:

1. Phạm Việt Cường

2. Nguyễn Kiều Hiếu

3. Nguyễn Minh Hiếu

4. Trần Thái Hưng

5. Vũ Đình Long

6. Nguyễn Ngọc Trung

Page 177: Final Version 1989 to 2010 VN TST